Elsevier Jan

157
Q 1. Which of the following tracts is concerned with pain and temperature? (a) Pyramidal tract (b) Anterior spinothalamic tract (c) Lateral spinothalamic tract (d) Dorsal spinocerebellar tract Answer : Unattempted Explanation : Pain and temperature are carried by the spinothalamic tract (STT) towards the VPL (ventro-postero-lateral) nucleus of thalamus.Pain is carried by the lateral STTwhereas anterior STT carries the crude touch sensations.Spinothalamic tracts carry sensations contralaterally hence the area of sensation loss is to the opposite side of the lesion (Brown-Séquard syndrome).Spinocerebellar tracts carry the unconscious proprioception, mainly from the lower limbs, towards the cerebellum and form the mossy fibers. They contribute archi-cerebellar fibers to the cerebellum.Dorsal spinocerebellar fibers carry the information ipsilaterally and enter the inferior cerebellar peduncle to reach the cerebellum.Ventral spinocerebellar tract carries the information contralaterally and enters the cerebellum via superior cerebellar peduncle.Unconscious proprioception of upper limb is carried by the Cuneocerebellar tract.Conscious proprioception is carried by the dorsal columns (Fasciculus gracilis and cuneatus).Pyramidal tract is a motor tract and is concerned with control of fine and skilled voluntary motor activity.References1 Chaurasia's Human Anatomy, (4th Ed.) Pg. 3182 Gray's Anatomy, (39th Ed.) Pg. 3163 Review of Medical Physiology, Ganong, (22nd Ed.) Pg. 1384 Textbook of Medical Physiology, Guyton and Hall, (11th Ed.) Pg. 588 Q 2. Regarding long bones, all of the following statements are true except (a) They are ossified in membrane (b) They provide levers for movement (c) They receive blood supply from one or more nutrient arteries, periosteal arteries and periarticular arteries (d) During growth they show structurally different regions like diaphysis, metaphysis and epiphysis #Num# (e) Their growth in length stops when the epiphyseal plates become ossified Answer : Unattempted Explanation : The long bones are ossified in cartilage, i.e. they show the endochondral mode of ossification, in which bone is laid over a cartilage model (see a standard text).

Transcript of Elsevier Jan

Page 1: Elsevier Jan

Q 1.  Which of the following tracts is concerned with pain and temperature?

(a) Pyramidal tract

(b) Anterior spinothalamic tract

(c) Lateral spinothalamic tract

(d) Dorsal spinocerebellar tract

 

Answer : Unattempted

Explanation :

Pain and temperature are carried by the spinothalamic tract (STT) towards the VPL (ventro-postero-

lateral) nucleus of thalamus.Pain is carried by the lateral STTwhereas anterior STT carries the crude

touch sensations.Spinothalamic tracts carry sensations contralaterally hence the area of sensation loss

is to the opposite side of the lesion (Brown-Séquard syndrome).Spinocerebellar tracts carry the

unconscious proprioception, mainly from the lower limbs, towards the cerebellum and form the mossy

fibers. They contribute archi-cerebellar fibers to the cerebellum.Dorsal spinocerebellar fibers carry the

information ipsilaterally and enter the inferior cerebellar peduncle to reach the cerebellum.Ventral

spinocerebellar tract carries the information contralaterally and enters the cerebellum via superior

cerebellar peduncle.Unconscious proprioception of upper limb is carried by the Cuneocerebellar

tract.Conscious proprioception is carried by the dorsal columns (Fasciculus gracilis and

cuneatus).Pyramidal tract is a motor tract and is concerned with control of fine and skilled voluntary

motor activity.References1 Chaurasia's Human Anatomy, (4th Ed.) Pg. 3182 Gray's Anatomy, (39th Ed.)

Pg. 3163 Review of Medical Physiology, Ganong, (22nd Ed.) Pg. 1384 Textbook of Medical Physiology,

Guyton and Hall, (11th Ed.) Pg. 588

Q 2.  Regarding long bones, all of the following statements are true except

(a) They are ossified in membrane

(b) They provide levers for movement

(c) They receive blood supply from one or more nutrient arteries, periosteal arteries and periarticular arteries

(d) During growth they show structurally different regions like diaphysis, metaphysis and epiphysis

#Num# (e)  Their growth in length stops when the epiphyseal plates become ossified

Answer : Unattempted

Explanation :

The long bones are ossified in cartilage, i.e. they show the endochondral mode of ossification, in which

bone is laid over a cartilage model (see a standard text).

Q 3.  Regarding sesamoid bones, all of the following statements are true except

(a) They develop within a tendon

(b) They stabilize a joint

(c) They help to change the direction of pull of the tendon

(d) They help to reduce friction on the tendon

Page 2: Elsevier Jan

#Num# (e)  They prevent obstruction of blood supply to the tendon

 

Answer : Unattempted

Explanation :

Sesamoid bones develop within tendons and so are quite mobile when the muscle is contracting or

relaxing. Hence they cannot contribute to the stability of the joint.

Q 4.  All of the following are examples of synovial joints except

(a) Hinge

(b) Syndesmosis

(c) Pivot

(d) Saddle

Answer : Unattempted

Explanation :

A syndesmosis, by definition, is a fibrous variety of joint and not synovial (Syn—together, desmos—

fibrous tissue); and a joint made with fibrous tissue.

Q 5.  The major and the most significant factor controlling joint stability, in most cases, is

(a) Fibrous capsule

(b) Articular surfaces

(c) Ligaments

(d) Atmospheric pressure

#Num# (e)  Muscle tone of surrounding muscles

 

Answer : Unattempted

Explanation :

The muscle tone of the surrounding short muscles is the main factor in stability, especially if the joint

surfaces are not congruent and matching. The best example of this is the glenohumeral or shoulder joint

where the muscles of the so-called ‘rotator cuff’ stabilize the joint.

Q 6.  Which of the following types of joint are present between ear ossicles?

(a) Primary cartilaginous joints

(b) Secondary cartilaginous joints

(c) Synovial joints

(d) Fibrous joints

Answer : Unattempted

Explanation :

Ear ossicles have multiaxial synovial joints between them.Malleus and incus form saddle variety of

synovial joint, whereas, incudostapedial joint is a ball and socket type. All the three ossicles develop in

Page 3: Elsevier Jan

cartilage and are cartilaginous bones.Malleus and incus develop in the cartilage of the 1st pharyngeal

arch, whereas, stapes develops in the 2nd arch with the styloid process and upper hyoid. Since the

joints are between cartilaginous bones, the articular cartilage will be of hyaline variety.Fibrous joints are

present in the skull as sutures and gomphosis. Syndesmosis is also a fibrous joint as observed between

radius and ulna and the posterior part of sacroiliac joint.Secondary cartilaginous joints (symphysis) are

usually in the midline, e.g., Manubriosternal and xiphisternal joints along with the pubis

symphysis.Primary cartilaginous joints (synchondrosis) is seen between the sphenoid and occipital

bone (at clivus) and also in the growing bones (Epiphysio-diaphyseal joints).Malleus is the most lateral

of the ossicles.Incus is the largest of the three ossicles.Stapes is the smallest ossicles and is most

medial.First rib has got two anterior synchondrosis as compared to the other true ribs (which have only

one).Note: Synovial joints formed by the Membranous bones are atypical and are lined by fibro-cartilage

(as articular cartilage).For example:Temporomandibular joint (condylar synovial) and sternoclavicular

joint (saddle synovial), their articular cartilage is not hyaline but fibro-cartilage.References1Chaurasia's

Human Anatomy, Vol. 3 (4th Ed.) Pg. 2622 Gray's Anatomy, (39th Ed.) Pg. 103, 6583 Gray's Anatomy,

(38 Ed.) Pg. 485, 617, 1275

Q 7.  The characteristics of synovial (diarthrodial) joints are all of the following except

(a) Articular surfaces of bones are covered by articular cartilage

(b) They permit either very limited movement or no movement at all

(c) A fibrous capsule reinforces and supports the joint

(d) A synovial membrane lines the fibrous capsule and covers the intracapsular parts of bone which are not covered by articular cartilage

#Num# (e)  The synovial fluid lubricates the articular surfaces and nourishes articular cartilage

 

Answer : Unattempted

Explanation :

Synovial joints have a cavity due to which movements necessarily occur. Small, plain synovial joints

show limited gliding or sliding. All others have great freedom of movement.

Q 8.  All of the following statements regarding the early development of the heart are true except

(a) The primitive cardiogenic area arises caudal to the stomodeum

(b) The heart undergoes positional changes and morphogenesis during flexion

(c) The heart begins to beat before five weeks of development

(d) Endothelium lining the heart is continuous with one layer lining the blood vessels and of same embryonic origin

#Num# (e)  Angiogenesis and cardiogenesis are fundamentally similar

Answer : Unattempted

Explanation :

Primitive cardiogenic area in the early embryo, before folding, lies cephalic to the stomodeum, but

caudal to the mesoderm of the septum transversum, the precursor of the diaphragm.

Page 4: Elsevier Jan

Q 9.  Which of the following statements concerning the development of teeth is correct?

(a) Primary (deciduous) teeth lack cementum

(b) The enamel organ is a neural crest derivative

(c) Secondary teeth arise from enamel organs after the primordia of primary teeth have formed

(d) The dental lamina forms by mesenchymal proliferation

#Num# (e)  The odontoblasts are enamel organ derivatives that secrete dentin

 

Answer : Unattempted

Explanation :

Odontoblasts secrete dentin but are derived from neural crest. Enamel organ gives rise to ameloblasts

(enameloblasts) which make enamel.

Q 10.  All of the following are true statements about liver development except

(a) The hepatic sinusoidal endothelium is derived from vitelline and umbilical veins

(b) Rapid growth of liver fills much of the peritoneal cavity during early gastrointestinal development

(c) Hepatic duct epithelial cells are derived from splanchnic mesoderm

(d) The liver parenchymal cells are derived from the liver diverticulum

Answer : Unattempted

Explanation :

Epithelial cells of hepatic ducts are derived from endoderm.

Q 11.  Composite muscles include all of the following EXCEPT

(a) Pectineus

(b) Adductor magnus

(c) Rectus femoris

(d) Biceps femoris

 Answer : Unattempted

Explanation :

Muscles like adductor magnus, pectineus, biceps femoris, etc., are called as composite muscles

because of their double innervation.Composite means made up of unlike parts (Dorland's medical

dictionary).DefinitionComposite muscles are those, whose functions are regarded the same in terms of

skeletal structure and muscle arrange-ment condition, as a single muscle model.For example: Vastus

intermedius, lateralis and medialis on the knee joint for its extension; Ilio-psoas is also a composite

muscle performing flexion at the hip joint (its parts are iliacus and psoas major).Certain muscles form by

fusion with either the dorsal or ventral musculature to constitute composite muscles, which will have

more than one nerve supply.For example:Adductor magnus is a composite muscle supplied by

obturator nerve and tibial part of sciatic nerve.Note: A single muscle itself is also considered as a

composite muscle, when its different parts work together to perform a particular function. For

example:Biceps femoris has its long head and short head running parallel to each other inserting on

Page 5: Elsevier Jan

the same point and working together for knee flexion. Long head of biceps is supplied by tibial part of

sciatic nerve and the short head by the common peroneal nerve.Pectineus may have 2 strata of fibers,

one supplied by the obturator nerve and the other by femoral nerve.Adductor magnus, Biceps femoris,

pectineus are composite individual musclesQuadriceps femoris and Ilio-psoas are composite group

muscles.References1 Chaurasia's Human Anatomy Vol. 2 (4th Ed.) Pg. 642 Gray's Anatomy, (39th Ed.)

Pg.1467, 14693 Surgical and mechanical treatment of peripheral nerves by Byron Stookey, Pg. 222

Q 12.  Digastric muscles are all of the following EXCEPT

(a) Occipitofrontalis

(b) Sternocleidomastoid

(c) Omohyoid

(d) Ligament of Treitz

Answer : Unattempted

Explanation :

Definition: Digastric muscle is a muscle with two bellies (Belly - the fleshy contractile part of the

muscle).Sternocleidomastoid has two heads and one belly, like biceps brachii muscle.Muscles like

and occipitofrontalis have two bellies: superior and inferior belly of omohyoid a nd occipital and frontal

belly of occipitofrontalis.Ligament of Treitz is a fold of mesentery at the duodeno-jejunal junction. This

ligament contain a belly of skeletal muscle arising from the left crus of diaphragm and another belly of

smooth muscle arising from the muscle coat of duodeno-jejunal flexure.Note: Gastrocnemius is a

muscle with two heads and two bellies-it is also a digastric muscle!References1 Dorland's Medical

dictionary2Gray's Anatomy, (39th Ed.) Pg. 113–14, 500, 536, 538, 11653 Last Clinical Anatomy (11th

Ed.) Pg. 369, 2624 K L Moore, (5th Ed.) Pg. 32, 263

Q 13.  Which of the muscle attached to tuberosity of 5th metatarsal will have avulsion of tendon

following violent inversion of the foot

(a) Peroneus brevis

(b) Peroneus longus

(c) Peroneus tertius

(d) Extensor digitorum longus

 

Answer : Unattempted

Explanation :

The peroneus brevis tendon passes deep to the peroneal retinaculum and inserted into the base of 5th

metatarsal.ReferencesGray's Anatomy for Students, (1st Ed.), Pgs. 552, 555.

Q 14.  Blood flow to the lower limbs in postductal coarctation of the aorta in maintained by increased

blood flow through

(a) Inferior phrenic and pericardiophrenic vessels

(b) Intercostal and superior epigastric

Page 6: Elsevier Jan

(c) Subcostal and umbilical

(d) Intercostal and inferior phrenic

Answer : Unattempted

Explanation :

Coarctation of aorta is the narrowing of the aorta distal to the left subclavian artery. This narrowing can

markedly reduce blood flow to the lower body commonly the internal thoracic, superior epigastric,

musculophrenic arteries enlarge anteriorly. These arteries supply the anterior intercostals arteries that

allow the blood to flow retrogradely into the aorta.ReferencesGray's Anatomy for Students, (1st Ed.), Pg.

214.

Q 15.  An elderly male on ventilator has received atracurium infusion for 3 days. He develops epileptic

fits. Probable cause for his epilepsy is

(a) Allergy to drug

(b) Accumulation of atracurium

(c) Accumulation of laudanosine

(d) Ventilator failure

 

Answer : Unattempted

Explanation :

Laudanosine is the major metabolite of atracurium, with Hofmann elimination and ester hydrolysis. It is

associated with central nervous system excitation, which leads to precipitation of seizure. These occur in

patient receiving high total dose or have hepatic failure (laudanosine is metabolized in the

liver).References[1. Clinical Anesthesiology, Morgan, (3rd Ed.), Pg 191[2. Pharmacology and Physiology

in Anesthetic Practice, Stoelting, (3rd Ed.), Pg 207

Q 16.  RTH Laennec is known as ‘the father of chest medicine’. He invented the stethoscope in the year

(a) 1781

(b) 1804

(c) 1816

(d) 1826

Answer : Unattempted

Explanation :

Laennec (1781–1826) was born in the year 1781 in Quimper, Brittany, France, studied medicine at

Charity Hospital in Paris and qualified in the year 1804. He invented stethoscope (1816), one of the

medicine's most powerful tools.

Q 17.  Who called Nitrous oxide as laughing gas?

(a) Priestley

(b) Sir Humphrey Davy

Page 7: Elsevier Jan

(c) William Morton

(d) Sir JY Simpson

 

Answer : Unattempted

Explanation :

In 1799 Sir Humphrey Davy inhaled Nitrous oxide and noted strange effect that included euphoria and

sobbing; and he called it ‘laughing gas’.

Q 18.  ‘Train of four’ is characteristically used in concern with

(a) Malignant hyperthermia

(b) Non-depolarizing neuromuscular blockers

(c) Mechanical ventilation

(d) To check hemodynamic parameters

Answer : Unattempted

Explanation :

A peripheral nerve stimulator delivers a current of variable frequency and amplitude to peripheral motor

nerve. The evoked mechanical or electrical response of the innervated muscle is observed.Train of four

stimulation denotes four successive 200 μ sec stimuli in 2 second (2Hz). The twitches in a train of four

patterns progressively fade as relaxation increases. The ratio of the response to the first and fourth

twitches is a sensitive indicator of non depolarizing muscle relaxation. Disappearance of the forth twitch

represent a 75% block, the third twitch a 80% block, and the second twitch a 90%

block.References[Clinical Anesthesiology, Morgan, (3rd Ed.), Pg. 121, 122

Q 19.  Cocaine was used to anaesthetize a frog cornea by

(a) William Halsted

(b) Gaedickle

(c) Albert Niemann

(d) Karl Kollar

 

Answer : Unattempted

Explanation :

Karl Kollar an ophthalmologist demonstrate the use of topical cocaine for surgical anaesthesia of eye

(1884). Cocaine was isolated from coca plant by Gaedickle (1855) and it was purified by Albert Niemann

(1860), and later surgeon William Halsted demonstrated its use in intradermal infiltration and nerve block

(1884).

Q 20.  Edmonds Gaskin Boyle invented the Boyle's anaesthesia apparatus in

(a) 1901

(b) 1907

(c) 1917

Page 8: Elsevier Jan

(d) 1941

Answer : Unattempted

Explanation :

Boyle qualified in the year 1901, wrote a book Practical Anaesthetics (1907), and invent anaesthesia

machine (1907). He expired in the year 1941.

Q 21.  Curved blade laryngoscope was introduced by

(a) Alfred Kirstein

(b) Robert Miller

(c) Philip Ayre

(d) Robert Macintosh

 

Answer : Unattempted

Explanation :

Robert Miller introduced straight blade (1941) laryngoscope. Alfred Kirstein of Freiburg pioneered direct

laryngoscopey in 1895. Philip Ayre introduced Mapleson E system breathing circuit.

Q 22.  All are true about thiopentone EXCEPT

(a) NaHCO3 is a preservative

(b) Contraindicated in porphyrias

(c) Agent of choice in shock

(d) Cerebroprotective

Answer : Unattempted

Explanation :

Thiopental To prevent formation of free acid by CO2 from the atmosphere, 6% anhydrous sodium

carbonate is added to powder, which is prepared in an atmosphere of nitrogen.Cerebral blood flow,

intracranial pressure, cerebral metabolism, and O2 consumption are reduced leading to cerebral

protection.Barbiturate many precipitate lower motor neuron paralyzes and perhaps death in patients of

porphyrias.Ketamine is a drug of choice in shock.References[Lee's Synopsis of Anesthesia, (12th Ed.),

Pg. 179, 183

Q 23.  ASA classification for preoperative assessment of patient's physical status was adopted in the

year

(a) 1951

(b) 1961

(c) 1967

(d) 1971

 Answer : Unattempted

Explanation :

Page 9: Elsevier Jan

American Society of Anaesthesiologist (ASA) adopted this classification in the year 1961.

Q 24.  At which hospital on October 16, 1846 W.T.G. Morton of Boston gave Ether anaesthesia to a

patient?

(a) Middlesex hospital

(b) Massachusetts General Hospital

(c) Boston Hospital

(d) University College Hospital

Answer : Unattempted

Explanation :

W.T.G. Morton gave Ether anaesthesia to Gilbert Abbott (a printer and journalist), for the removal of a

jaw tumour by Dr John Collins Warren.

Q 25.  Enzyme responsible for complete oxidation of glucose to CO2 to water is present in

(a) Cytosol

(b) Mitochondria

(c) Lysosomes

(d) Endoplasmic reticulum

 

Answer : Unattempted

Explanation :

There are three steps of the complete glucose oxidation to H2O and CO2.Glycolysis, in cytosol, is a

process in which one molecule of glucose is converted into two molecules of pyruvate.Oxidative

phosphorylation is a process in which two molecules of pyruvate form two molecules of acetyl COA in

mitochondria.Krebs cycle, the final step, converts two molecule of acetyl CoA into CO2, H2O, and

energy. This process takes place in mitochondria.ReferencesHarper's Illustrated Biochemistry (27th

Ed.), Pg. 145.

Q 26.  Which of the following is the first substrate of Krebs cycle?

(a) Pyruvate

(b) Glycine

(c) HCl

(d) Lipoprotein

Answer : Unattempted

Explanation :

Among the given choices, pyruvate is the correct answer as it forms the acetyl CoA, which enters in the

Krebs cycle and forms citrate along with the oxaloacetate. The reaction is known as

condensation.ReferencesHarper's Illustrated Biochemistry, (27th Ed.), Pgs. 146-147, 168.

Page 10: Elsevier Jan

Q 27.  Which of the following is not an essential amino acids?

(a) Methionine

(b) Lysine

(c) Alanine

(d) Leucine

 

Answer : Unattempted

Explanation :

Essential amino acids are those which cannot be synthesized by human body and hence require

exogenous supplement. Essential amino acids are threonine, valine, tryptophan, isoleucine, leucine,

phenylalanine, and methionine (TV TILL 8 pm), while histidine and arginine are semi-essential amino

acids.ReferencesHarper's Illustrated Biochemistry, (27th Ed.), Pg. 242.

Q 28.  The long and short arms of chromosomes are designated, respectively, as

(a) p and q arms

(b) m and q arms

(c) q and p arms

(d) l and s arms

Answer : Unattempted

Explanation :

There are two arms of all human chromosomes.A short arm “p”A long arm “q”The symbol “p” was

chosen to designate the short arm because “p” stands for “petit” in French, which means small, while

letter “q” was selected to signify the long arm just because “q” is the next letter in the alphabet.Both the

arms are separated with each other with the help of centromere, which is the point of attachment of

chromosome to the spindle during the cell division.

Q 29.  Euchromatin is the region of DNA that is relatively

(a) Uncondensed

(b) Condensed

(c) Overcondensed

(d) Partially condensed

 

Answer : Unattempted

Explanation :

Euchromatin is the region of DNA that is partially condensed and transcriptionally active. Constitutive

heterochromatin is always condensed and inactive. Facultative heterochromatin may be condensed and

inactive or uncondensed and active.ReferencesHarper's Illustrated Biochemistry, (27th Ed.), pg. 324.

Q 30.  Reactive oxygen intermediates in peroxisomes are produced by

(a) NADPH oxidase

Page 11: Elsevier Jan

(b) Superoxide dismutase

(c) Catalase

(d) Glutathione peroxidase

Answer : Unattempted

Explanation :

Peroxisomes are so named because they usually contain one or more enzymes that use molecular

oxygen to remove hydrogen atoms from specific organic substrates in an oxidative reaction that

produces hydrogen peroxide (H2O2).Catalase enzyme in peroxisomes utilizes H2O2 generated by other

enzymes in the organelle to oxidize a variety of other substrates–including phenols, formic acid,

formaldehyde, and alcohol by the “peroxidative” reaction: H2O2 + R' H2→ R' + 2H2O.This type of

oxidative reaction is particularly important in liver and kidney cells, where the peroxisomes detoxify

various toxic molecules that enter the bloodstream.ReferencesTextbook of Biochemistry with Clinical

Correlations, Devlin, (5th Ed.), Pg. 22

Q 31.  Trypsin can be activated by which of the following?

(a) Alpha-I antitrypsin

(b) Alpha-I-anti proteinase

(c) Enterokinase

(d) Egg-white

 

Answer : Unattempted

Explanation :

Trypsin is a proteinase.It can hydrolyze a peptide bond formed by the carbonyl group of a lysine residue

on trypsinogen, converting the latter to trypsin and inactive hexapeptide.It can be activated (i.e.,

trypsinogen converted to trypsin) by enterokinaseThe powerful protein-splitting enzymes of the

pancreatic juice are secreted as inactive proenzymes.Trypsinogen is converted to the active enzyme

trypsin by the brush border enzyme enteropeptidase (en terokinase) when the pancreatic juice enters

the duo denum.Enteropeptidase contains 41 % polysaccharide, and this high polysaccharide content

apparently pre vents it from being digested itself before it can exert its effect.Trypsin converts

chymotrypsinogen into chymotrypsin and other pro enzymes into active enzymes.Trypsin can also

activate trypsinogen; therefore, once some trypsin is formed, there is an auto catalytic chain

reaction.Enteropeptidase deficiency oc curs as a congenital abnormality and leads to protein

malnutrition.Alpha-I anti trypsin is the principal serine protease inhibitor of human plasma. It inhibits

trypsin, elastase and certain other proteases forming complexes with them.Alpha-I anti proteinase is the

new name for alpha-I anti trypsinEnterokinase activates trypsinogen into trypsin in the presence of

CalciumEgg-white is a trypsin inhibitorOther trypsin inhibitors are:Human and bovin ColostrumRaw

SoyabeansDFP (Di Iso propyl fluoro phosphate)ReferencesHarper's Illustrated Biochemistry, (27th Ed.),

Pg. 597

Q 32.  All of the following are TRUE about denaturation of proteins EXCEPT

(a) Retention of biological activity

Page 12: Elsevier Jan

(b) Increased viscosity

(c) Increased solubility

(d) Same structure

Answer : Unattempted

Explanation :

The most common observation in denaturation of proteins is precipitation or coagulation of the

protein.Denatured proteins lead to wide range of characteristics like loss of solubility to communal

aggregation.Denaturation reaction cannot break the peptide bonds so primary structure does not change

after denaturation, but the secondary and tertiary structure lead to disruption and destruction.Tertiary

structure has hydrogen bonding, disulfide bonding, salt bridges, and nonpolar hydrophobic interactions;

all these are disrupted in denaturation reaction.Denaturation disrupts the normal α-helix and β-sheets

and uncoils them into a random shape.

Q 33.  Treatment of multiple carboxylase deficiency is

(a) Biotin

(b) Pyridoxine

(c) Thiamine

(d) Folic acid

 

Answer : Unattempted

Explanation :

Biotin is the vitamin required in fatty acid biosynthesis. The rate-limiting enzyme for this process is

acetyl-CoA carboxylase for which biotin acts as a cofactor. Biotin deficiency results in multiple

carboxylase deficiency (MCD).

Q 34.  Study of multiplication of proteins in disease process is termed as

(a) Proteomics

(b) Genomics

(c) Glycomics

(d) Nucleomics

Answer : Unattempted

Explanation :

Proteomics is a study of the entire complement of proteins elaborated by cell under normal physiological

and pathological conditions.Genomics is the study of an organism's entire genome.The identity of the

entirety of carbohydrates in an organism is thus collectively referred to as the glycome. The word “glyco”

means sugar. The study of glycome is known as glycomics.Nucleomics is the study of the behavior and

characteristics of nucleons or atomic nuclei. It is a branch of physical science that deals with nucleons or

with all phenomena of the atomic nucleus.References1 Harper's Illustrated Biochemistry, (27th Ed.),

Pgs. 28-29.2http://medical.merriam-webster.com/medical/nucleonics.3http://en.wikipedia.org/wiki/

Protein_purification.

Page 13: Elsevier Jan

Q 35.  Insulin causes increased lipogenesis by all of the following EXCEPT

(a) Inhibition of pyruvate dehydrogenase

(b) Increasing activity of acetyl carboxylase

(c) Decrease of cAMP in cell

(d) Causes glucose to move in the cell

 

Answer : Unattempted

Explanation :

Insulin stimulates lipogenesis by the following:Increasing acetyl CoA carboxylase activity.Increasing

transport of glucose into cell (adipose tissue).Converts inactive form of pyruvate dehydrogenase to

active form in adipose tissue.Insulin, by its ability to depress the level of intracellular cyclic AMP and

inhibit lipogenesis in adipose tissue, decreases the concentration of plasma FFA and therefore long-

chain acetyl CoA, which is the inhibitor of lipogenesis.ReferencesHarper's Illustrated Biochemistry, (27th

Ed.), Pg. 201–2

Q 36.  Steroid receptor has attachment site for all of the following EXCEPT

(a) Hormone responsive element

(b) Transcription repressor

(c) Transcription mediator

(d) Steroid hormone

Answer : Unattempted

Explanation :

All receptors have at least two functional domains:A recognition domain binds the hormone ligand and a

second region generates a signal that couples hormone recognition to some intracellular function.

Steroid, thyroid, and retinoid hormone receptors have several functional domains:One site binds the

hormone.Another binds to specific DNA regions.The third site is involved in the interaction with other

coregulator proteins that result in the activation (or repression) of gene transcription.The fourth site may

specifically bind to one or more of the proteins that influence the intracellular trafficking of the

receptor.ReferencesHarper's Illustrated Biochemistry, (26th Ed.), Pg. 435

Q 37.  White-blue screening is used in DNA-cloning for

(a) Identifying plasmid in DNA

(b) Cloned part of DNA

(c) Inserted DNA

(d) Protein produced

 Answer : Unattempted

Explanation :

The vector may also include a gene spanning cloning site, which creates an easily detectable phenotype

Page 14: Elsevier Jan

to allow for selection of inserts such as the LacZa gene, which is used to span a cloning site to allow for

convenient blue–white color screening for recombinant vectors, including an insert at the site within the

LacZa gene.

Q 38.  All of the following are TRUE about glutathione EXCEPT

(a) It is a tripeptide

(b) Glutathione synthesis takes place in cytosol of the cells

(c) Takes part in heme biosynthesis

(d) Its deficiency causes Hartnup's disease

Answer : Unattempted

Explanation :

Glutathione is a tripeptide (glutamic acid + cysteine + glycine)It converts Hb to metHb.Hartnup's

disease is due to the defect in renal and intestinal transport of tryptophan.Participates in decomposition

of toxic hydrogen peroxide in the reaction catalyzed by glutathione peroxidase.It is an

important intracellular reductant.Helps in transport of certain amino acids across the membrane in

kidney.ReferencesTextbook of Biochemistry, Chatterjee, Pg. 429

Q 39.  A high specificity screening test tells about

(a) True positive

(b) True negatives

(c) False positive

(d) False negatives

 

Answer : Unattempted

Explanation :

A screening test with a high specificity will have few false positives.The term “false positive” means that

patients who do not have the disease are told that they have it. In which case, normal healthy people

may be subjected to further diagnostic tests.A screening test which is very sensitive has few “false

negatives.” The lower the sensitivity, the larger will be the number of false negatives, otherwise

sensitivity tells about true positive and specificity tells about true negative.References[Textbook of

Preventive and Social Medicine, Park (10th Ed.), Pg. 111

Q 40.  Indian male reference criteria are TRUE in all of the follow ing EXCEPT

(a) 60 kg

(b) 220 kcal/day

(c) 8 hours in bed

(d) 20–39 age

Answer : Unattempted

Explanation :

Page 15: Elsevier Jan

Criteria for Indian male reference areWeight–60 kg.Spends 8 hours in bed, 4–6 hours sitting and moving

around and 2 hours in walking and in active recreation or household duties.Age 20–39 years.Free from

disease and physically fit for active work on each work ing day, employed for 8 hours that involve

moderate activity.For an Indian reference male the energy allowance is 2425 calories/day; so the choice

(b) with 2,200calories or 220 kcal/day is not fitting in the criteria.Note: Indian female criteria is on Pg.

458 of Park.References[1. Textbook of Preventive and Social Medicine, Park, (16th Ed.), Pg.

425[2. Textbook of Preventive and Social Medicine, Park, (18th Ed.), Pg. 457

Q 41.  Chronic carrier state is not seen in

(a) Measles

(b) Diphtheria

(c) Typhoid

(d) Gonorrhea

 

Answer : Unattempted

Explanation :

Carriers are defined as “An infected person or animal that harbors a specific infectious agent in absence

of clinical disease and serves as a potential source of infection.”Measles does not have long-term

chronic carriers; however, they are present in diphtheria, gonorrhea, malaria, HBV, dysentery, and

typhoid.

Q 42.  Which of the following is least common complication of measles is

(a) SSPE

(b) Pneumonia

(c) Otitis media

(d) Keratomalacia

Answer : Unattempted

Explanation :

The least common complication of measles is SSPE; it is a serious neurological complication but the

incidence is very rare (1 in 100,000). This results mainly because of immune-mediated response to

myelin proteins and not due to virus itself.Note: The most common complication of measles is measles-

associated diarrhea.

Q 43.  All of the following strategies are used for prevention of neonatal tetanus, EXCEPT

(a) Five clean practices

(b) Giving penicillin to all the newborns

(c) Two doses of TT to all pregnant women

(d) Immunizing all married women

 Answer : Unattempted

Page 16: Elsevier Jan

Explanation :

Antibiotics are indicated for the treatment of tetanus and not for its prevention. However, the prevention

of neonatal tetanus is by the following:FIVE Clean practice or safe-delivery practice if not institutional

otherwise institutional deliveries should be preferred not only for tetanus but for other reasons

also.Immunization of the mother in antenatal period.

Q 44.  All of the following are transmitted by lice, EXCEPT

(a) Q fever

(b) Trench fever

(c) Relapsing fever

(d) Epidemic typhus

Answer : Unattempted

Q 45.  Which of the following disease not transmitted by lice?

(a) Relapsing fever

(b) Trench fever

(c) Q fever

(d) Epidemic typhus

 

Answer : Unattempted

Explanation :

This particular question is a very pet question of examiners. PG aspirants are suggested to mug up all

details about rickettsial diseases particularly about Q fever.

Q 46.  Mean ≪ median

(a) Negative skewed

(b) Normal

(c) Positively skewed

(d) Cannot be specified

Answer : Unattempted

Explanation :

A normal curve is symmetrical, smooth, and bell shaped.The highest frequencies are concentrated in

the center of the curve around mean and the lowest at the two extremes; normally half of the values lie

below, i.e., on the left side of the mean and the other half lie above, i.e., on the right hand side of the

mean.All the three measures of centered tendency, mean, median, and mode, coincide at the centre of

the curve; their values are identical in a normal distribution series.So if mean ≪ median-negatively

skewed and if mean > median-positively skewed.References[1.Community Medicine, Dhaar and

Robbani[2. High-Yield Biostatics, Pg. 11, 12[3.Mahajan Biostatics, Pg. 68

Page 17: Elsevier Jan

Q 47.  Endemic ascites is associated with

(a) Pyrrolizidine

(b) Aflatoxin

(c) BOAA

(d) Sanguinarine

 

Answer : Unattempted

Explanation :

Endemic ascites is caused by pyrrolizidine alkaloid, a hepatotoxin found in weed seeds of Crotalaria

(Jhunjhunia).These weed seeds contaminate the millet seedsPanicum miliare.It causes ascites and

jaundice.Beta-oxylamino alanine (BOAA) toxin is present in seeds of pulse Lathyrus platinum (Khesari

dal) causes neurolathyrism. Sanguinarine toxin is found in Argemone mexicana seeds which causes

epidemic droopy. Argemone contaminates mustard oil.References[1.Textbook of Preventive and Social

Medicine, Park, (18th Ed.), Pg. 480[2. Textbook of Preventive and Social Medicine, Park, (19th Ed.), Pg.

525

Q 48.  Which of the following statements is not true for herd immunity?

(a) Herd structure is constant

(b) It is mostly due to subclinical infection

(c) Can be acquired by immunization

(d) Spread of epidemic is influenced by it

Answer : Unattempted

Explanation :

Herd structure is constant.Herd structure is defined as a group of immunized people that is built up by

the protection of individual members who are protective. It provides an immunological barrier for the

spread of disease. It is never constant and always changes because of addition of new unimmunized

members, migration, death, and in presence of alternative host.

Q 49.  Under eradication of congenital rubella syndrome program the first priority group for rubella

vaccination is

(a) All nonpregnant women of age 15–34 years

(b) All adolescent nonpregnant girls 15–24 years of age

(c) All female children at 1 year

(d) All nonpregnant women

 

Answer : Unattempted

Explanation :

The immunization strategies to prevent CRS have been directed first toward protecting the women of

childbearing age group (15–39 years) and then to interrupt transmission of rubella subsequently to all

children by age of 1 year.

Page 18: Elsevier Jan

Q 50.  Which of the following statement is true about BCG vaccination?

(a) Distilled water is used as diluent for BCG

(b) The site of injection should be cleaned with spirit

(c) Mantoux test becomes positive after 48 hours of vaccination

(d) WHO recommends Danish 1331 strain for vaccine production

Answer : Unattempted

Explanation :

BCG is live attenuated vaccine, hence it should never to be cleaned with spirit only; plain water swab is

used.The bacilli are derived from a bovine strain (Danish 1331 recommended by WHO).Present day

vaccines are supplied in the freeze-dried form, if it is preferably stored at a temperature below 10°C.For

reconstitution, normal saline is recommended as diluent, as distilled water may cause irritation.Dosage

of 0.1 mg in 0.1 mL volume. Dosage in a newborn (up to 4 weeks) is 0.05 mL.Vaccine is injected

intradermally using a tuberculin syringe (omega microstat syringe). The site of injection is the insertion of

deltoid of the left arm. A satisfactory injection should produce a wheel of 5 mm diameter.Time of

administration being at birth or at 6 weeks along with the other vaccines.

Q 51.  Case finding in RNTCP is based on

(a) Sputum culture

(b) Sputum microscopy

(c) X-ray chest

(d) Mantoux test/PCR

 

Answer : Unattempted

Explanation :

Case finding in RNTCP is based on sputum microscopy.Gold standard test for detection of TB is

however sputum culture (99.5% sensitive) but it takes 3–4 weeks to give the culture reports. Sputum

culture is too costly (in a private laboratory, each culture costs anything ranging between Rs 400 and

Rs 500).Three sputum smear, if done as per protocol of RNTCP (spot, overnight, and spot), will also

yield a sensitivity of same order. Hence, it is the investigation of choice.

Q 52.  All can be incinerated, EXCEPT

(a) Cytotoxic waste

(b) Sharps

(c) Anatomical waste

(d) Infectious waste

Answer : Unattempted

Explanation :

Sharp wastes, such as needles, scalpels, syringe, and blades, come under CAT-4 of biomedical waste

Page 19: Elsevier Jan

management and they are disposed by chemical treatment, autoclaving, microwaving, and mutilation

shedding.

Q 53.  According to the concept of the iceberg phenomenon of disease, the ‘waterline’ represents the

demarcation between which of the following?

(a) Undiagnosed and diagnosed

(b) Symptomatic and presymptomatic

(c) Carrier and sufferers

(d) Apparent and inapparent

 

Answer : Unattempted

Explanation :

A concept closely related to the spectrum of disease is the concept of the iceberg phenomenon of

disease; according to this concept disease in a community may be compared with an iceberg.The

floating tip of the iceberg represents what is physical seen in the community i.e., clinical case; the vast

submerged portion of the iceberg represents: the presymptomatic and undiagnosed cases, and carries

in the community; the waterline represents the demarcation between the apparent and inapparent

disease.References[Textbook of Prevent and Social Medicine, Park, (19th Ed.), Pg. 36

Q 54.  Which of the following is not a criterion for judging causality in non communicable diseases?

(a) Dose response relationship

(b) Lack of temporal association

(c) Specificity of the association

(d) Strength of association

Answer : Unattempted

Explanation :

The problemChronic non-communicable diseases are assuming increasing importance among the

adult population in both developed and developing countries.Non-communicable disease risk

factorsMost epidemiologists accept that six key sets of “risk factors” are responsible for a major share

of adult non-communicable disease morbidity and premature mortality. These are as follows:Cigarette

use and other forms of smokingAlcohol abuseFailure or inability to obtain preventive health services

(e.g., for hypertension control, cancer detection, management of diabetes)Life-style changes (e.g.,

dietary patterns, physical activity)Environmental risk factors, (e.g., occupational hazards, air and water

pollution, and possession of destructive weapons)Stress factors.Gaps in natural historyThese gaps

cause difficulties in etiological investigations and research.These are:Absence of a known

agentMultifactorial causationLong latent periodIndefinite onsetReferences[Textbook of Prevent and

Social Medicine, Park, (18th Ed.), Pg. 82–3

Q 55.  A village having which of the following features is called “NOT a problem village”?

(a) Safe water with a distance of > 1.6 km

Page 20: Elsevier Jan

(b) Depth of drinking water >15 m

(c) Excess mineral

(d) Guinea worm

 

Answer : Unattempted

Explanation :

According to the Bhore Committee and Environmental Committee for Water Supply and Sanitation, a

problem village isWhere water recourses are present > 1.6 km distance.Water at a depth of >15 m.Has

cholera or guinea worm in the proximity.Water has iron, fluoride, excess salinity, or other

minerals.References[Textbook of Preventive and Social Medicine, Park (16th Ed.), Pg. 627

Q 56.  Which of the following does not cause the thiamine deficiency?

(a) Chronic diarrhoea

(b) Chronic alcoholism

(c) Homocysteinemia

(d) Food faddist

Answer : Unattempted

Q 57.  Regarding fluorosis not a true statement is

(a) Fluorosis is the most common cause of dental caries in children

(b) Deposition occurs in the skeletal system and muscles

(c) Defluoridation is done by Nalgonda technique

(d) Genu valgum

 

Answer : Unattempted

Explanation :

Fluorosis occurs when fluoride content of water is between 3–5 mg/liter. Normal fluoride content of

drinking water should be between 0.5–0.8 mg%.Its also known as double-edged sword. Prevention

fluorosis includes change of drinking water source.Nalgonda technique (NEERI)Lime + Alum followed

by flocculation, sedimentation, and filtration.

Q 58.  Which of the following is not true in a randomized control trial (RCT)?

(a) Baseline characteristics of intervention and control groups should be similar

(b) Investigator's bias is minimized by double blinding

(c) The sample size required depends on the hypothesis

(d) The dropouts from the trial should be excluded from the analysis

Answer : Unattempted

Explanation :

In RCT, sample size does not depend upon the hypothesis to be tested.

Page 21: Elsevier Jan

Q 59.  DOC for a pregnant woman in 2nd trimester with pustular psoriasis is

(a) Prednisolone

(b) Dapsone

(c) Acitretin

(d) Methotrexate

 

Answer : Unattempted

Explanation :

Pustular psoriasis in pregnancy is called Impetigo Herpetiformis, out of all the choices dapsone is

useless and of the rest the safest is steroids.Non-pustular psoriasisChronic plaque type: Nb UVB,

PUVAAcute guttate: UVB, AntibioticsInverse, flexural: Topical SteroidsErythrodermic: Methotrexate,

retinoidsUnstable nummular: Steroids systemicSebo-psoriasis: Topical steroidsPustular

psoriasisGeneralized pustular psoriasis (von Zumbusch): Retinoids (DOC), Methotrexate (2nd

DOC)Pregnancy: Systemic steroids i.e. PrednisolonePsoriasis with arthropathy: Methotrexate (DOC),

etanerceptDOC in psoriasis areNormally-PUVA therapy is DOCIn Pustular, AIDS, Psoriatic

Erythroderma - Retinoids (DOC)In psoriatic arthropathy-Methotrexate (DOC)References1. Rook's

Textbook of Dermatology, (7th Ed.), Pg, 34.48–34.612. Comprehensive Textbook of Dermatology (with

MCQ), 2008 (in press), Vijay K Garg, Kabir Sardana

Q 60.  Where are tuberculides seen?

(a) Lupus vulgaris

(b) Scrofuloderma

(c) Lichen scrofulosorum

(d) Erythema nodosum

Answer : Unattempted

Explanation :

Erythema nodosum can also be seen as a hypersensitivity reaction to tubercular focus in the body, but

it has other causes as well, so it is not a definite tuberculide.ReferencesIllustrated Synopsis of

Dermatology and STD, Khanna N, (2nd Ed.), Pg. 122, 216

Q 61.  A 42-year-old female has palpable purpura with rash over buttocks, pain in abdomen, and

arthropathy diagnosis is

(a) Sweet syndrome

(b) HSP

(c) Purpura fulminans

(d) Meningococcemia

 Answer : Unattempted

Explanation :

Page 22: Elsevier Jan

Henoch-Schönlein purpura is a systemic disease characterized by a petechial rash on the extremities,

arthropathy, abdominal pain, and glomerulonephritis.The glomerular lesion is identical to that found in

IgA nephropathy, Henoch-Schönlein nephritis and IgA nephropathy may be part of a spectrum of

manifestations of a single disease.Nephritis is present in 80% of the patients and manifests as a

nephritic urine sediment and moderate proteinuria.Macroscopic hematuria and nephritic range

proteinuria are uncommon.Light-microscopic appearance can vary from mild mesangial proliferation

and expansion to diffuse proliferation with glo-merular crescents.The sine qua non for diagnosis is the

presence of mesangial IgA deposition on immunofluorescence microscopy.IgG and C3 are also

detected. Electron microscopy reveals mesangial immune deposits.Immune complexes may also be

present in the peripheral glomerular capillary wall and para mesangial areas.Biopsy of involved skin

reveals dermal IgA deposition and leukocytoclastic vasculitis.IgA deposition is also seen in the areas of

uninvolved skin.Since there is no proven therapy for Henoch-Schönlein nephritis, treatment is

symptomatic. Steroids and/or cytotoxic agents are often tried in patients with severe disease.The

disease typically undergoes clinical exacerbation.The prognosis is generally

excellent.ReferencesHarrison's Principles of Internal Medicine, (16th Ed.), Pg. 1682 (17th Ed.), Pg,

2128

Q 62.  Spalding's sign a characteristic feature of

(a) Drowning

(b) Maceration

(c) Mummification

(d) Starvation

Answer : Unattempted

Explanation :

Loss of alignment and overriding of skull bones due to shrinkage of the brain is called as Spalding's

sign (as seen on an X-ray). Maceration is a process of aseptic autolysis which occurs when the dead

child remains in the uterus for some period surrounded with liquor amnii, but with the exclusion of air.

The earliest sign of maceration is skin slippage, seen in 12 hours after death of the child. The

macerated fetus looks soft and flaccid, with a sweetish disagreeable smell. Skin shows purple

discoloration, large fluid-filled blebs, and peeling off of skin. Bones become flexible and detachable and

the viscera become edematous.The importance of signs of maceration is that with this one can

differentiate a “dead born” child (died in the uterus before the birth process began) from a “still born”

child which does not show these signs. A still born child is one who has completed 28 weeks of

pregnancy, died in- utero and did not show any sign of life after being completely

born.ReferencesTextbook of Forensic Medicine and Toxicology, Vij K, (3rd Ed.), Pg. 225–226.

Q 63.  Spalding's sign is characteristically seen in

(a) Drowning

(b) Mummification

(c) Adipocere

Page 23: Elsevier Jan

(d) Maceration

 Answer : Unattempted

Q 64.  Burtonian line is seen in the poisoning of

(a) Mercury

(b) Lead

(c) Arsenic

(d) Zinc

Answer : Unattempted

Explanation :

Burtonian line (blue line at gums' margin, lead sulphide-PbS, 50-70% cases, near carious/dirty teeth) is

seen in chronic lead poisoning. Chronic lead poisoning can be referred to as plumbism or saturnism.

The signs and symptoms of saturnine poisoning are as follows: facial pallor, basophilic

stippling/punctate basophilia (dark blue, pin-head spots in cytoplasm of RBCs), Pb palsy, Pb

encephalopathy, wrist drop, foot drop, colic, constipation, menstrual irregularity, sterility, nephropathy,

aminolevulinic acid (ALA) in urine, coproporphyrin in urine, and X-ray radiopaque bands at metaphyses

of long bones (signifies Pb > 0.03 mg % in blood and is a diagnostic feature).ReferencesRudra A.

Lyon's Medical Jurisprudence & Toxicology, The first Indian textbook of Forensic Medicine and

Toxicology, Dogra TD, (11th Ed.), Pg. 1164–1168.

Q 65.  Acute Yellow Fatty Liver is caused by which of the following poisonings?

(a) Aconite

(b) Arsenic

(c) Lead

(d) Mercury

 

Answer : Unattempted

Explanation :

Arsenic poisoning-Substance used are As2O3, Arsenic trioxide, Arsenious oxide, Scheele's green, Paris

green/Emerald green, Arsine gas, Orpiment/Realgar. ‘sankhya = somalkhar’Arsenic poisoning is of

three types -Fulminant-Death in 3 hrs.Gastroenteric-most common, resembles bacterial food

poisoning; constriction of throat, colicky pain, intense thirst, vomiting, uremia, circulatory

collapse.Chronic-Rash, hyperkeratosis, rain-drop pigmentation, basal cell carcinoma, cirrhosis of liver,

portal hypertension, subendocardial petechial hemorrhages of the heart.Tests for diagnosis are-

Marsh's test, Reinsch's test.ReferencesThe Essentials of Forensic Medicine and Toxicology, Reddy,

(16th Ed.), Pg. 433–36

Q 66.  Acrodynia is a feature of which poisoning?

(a) Mercury

Page 24: Elsevier Jan

(b) Phenolic acid

(c) Oxalic acid

(d) Carbolic acid poisoning

Answer : Unattempted

Explanation :

Mercury poisoning refers to the disease caused by exposure to mercuric chloride (HgCl2), which is also

referred to as the corrosive sublimate. Acid metallic test can be used for detecting mercury in the body.

Signs and symp-toms include burning pain, vomiting, circulatory collapse, Hatter's shakes(Glass

blower's shakes or generalized tremor), mercurial erethism (anxiety, depression, timidity, acrodynia),

mercurialentis (brownish deposit of mercury on anterior lens capsule, is bilateral and has no effect on

visual acuity). Acrodynia(Pink disease or Feer's disease or Swift syndrome) is characterized by painful

and pink extremities.ReferencesTextbook of Forensic Medicine and Toxicology, Vij K, (3rd Ed.), Pg.

655–666.

Q 67.  Corpus delicti correlates to

(a) Police officer

(b) Essence of crime

(c) Conduct of postmortem

(d) Death by asphyxia

 

Answer : Unattempted

Explanation :

The corpus delicti means body of offence or essence of crime. In case of a murder, it is the fact that

a person has died from unlawful violence and includes the body of the victim and other facts like bullet,

knife, or cloths.ReferencesSharma, Concise Textbook of Forensic Medicine and Toxicology, (1st Ed.),

Pg. 11.

Q 68.  A 30-year-old male from West Bengal presents to you with hyperkeratosis, and transverse nail

lines. Most likely cause is

(a) Chronic arsenic poisoning

(b) Chronic lead poisoning

(c) Chronic mercury poisoning

(d) Acute arsenic poisoning

Answer : Unattempted

Explanation :

Through the water supply in W. Bengal the arsenic comes into the system causing - rain drop

pigmentation on the back, Mees lines, Nail Leukonychia, and palmoplantar hyperkeratosis.Chronic

arsenic poisoning causes some skin signs and symptoms such as- Hyperkeratosis (mostly at distal

parts), Desquamation (of palms and soles), Raindrop pigmentation, Mees' lines (transverse white striae

of the fingernails technically called as striate leukonychia).Chronic Arsenic poisoning also causes-GIT

Page 25: Elsevier Jan

symptoms-loss of weight, malaise, loss of appetite, salivation, colicky pain abdomen,

constipation.Hematological symptoms-anemia-normocytic, normochromic and leucopenia,

thrombocytopenia.Peripheral neuropathy- sensory and motor polyneuritis (sensory symptoms

predominate), numbness and tingling.Arsenic exposure can increase the risk of internal malignancy

such as lung, bladder, kidney and prostate.Signs of previous arsenic exposure include hyperkeratotic

lesions in palms and soles, diffuse truncal hyperpigmentation and multiple squamous cell

carcinomas.References1.Essentials of Forensic Medicine and Toxicology, Reddy, (16th Ed.), Pg.

4352.Textbook of Forensic Medicine and Toxicology, Krishan Vij, (3rd Ed.), Pg. 646–47

Q 69.  Which of the following tests is done for vaginal cells collected for the investigation for rape?

(a) Lugol's iodine test

(b) Benzidine test

(c) Takayama test

(d) Florence test

 

Answer : Unattempted

Explanation :

Lugol's iodine test is done on the washing of glans penis (or from a moist blotting paper) of the accused

which is then exposed to Iodine vapors or Lugol's iodine solution. Brown color means the test is

positive (results can be elicited up to 4th day), brown color is due to the presence of glycogen in the

vaginal epithelial cells of the victim.This test is an important tool in the examination of the accused

person of a sexual offense/rape.Presence/absence of smegma should also be noted.Presence of

smegma rules out complete penetration as it gets rubbed off during sexual intercourse.It is collected

again after 24 hrs.A non-retractile prepuce is one which covers the glans penis.ReferencesThe

Essentials of Forensic Medicine and Toxicology, Reddy, (16th Ed.), Pg. 333

Q 70.  In a postoperative case of liver transplant which of the fol lowing cells are developed?

(a) B-cell

(b) T-cell

(c) Myeloid cell

(d) Null cell

Answer : Unattempted

Explanation :

Transplantation immunity is predominantly cell mediated.The first set of response is brought about

almost exclusively by T-lymphocytes.Antibodies are formed more rapidly and abundantly during a

second set of response.Cyclosporine, an immunosuppressive agent, contributed substantially to the

improvement in survival after liver transplantation. Cyclosporine inhibits early activation of T-cells and

is specific for T-cell function.ReferencesTextbook of Microbiology, Ananthnarayan, (6th Ed.), Pg.

165Robbin's Basic Pathology, (8th Ed.), Pg. 190Harrison's Principles of Internal Medicine, (15th Ed.),

Pg. 1772

Page 26: Elsevier Jan

Q 71.  In torsades de pointesthe ECG changes are

(a) Wide QRS

(b) Narrow QRS

(c) Wide QTC

(d) Narrow QTC

 

Answer : Unattempted

Explanation :

Torsade de pointes (twisting of the points)–polymorphic VT associated with long QT interval; may be

caused by triggered activity due to early after depolarization.Very prominent U-waves are markers of

increased susceptibility to the torsade de pointes type of ventricular tachycardia.It refers to VT

characterized by polymorphic QRS complexes that changes in amplitude and cycle length giving the

appearance of oscillations around the baseline. This rhythm is by definition associated with QT

prolongation.CausesElectrolyte disturbance (decrease in potassium and magnesium)Antiarrhythmic

drugs (Quinidine)PhenothiazineTricyclic antidepressantsLiquid protein dietsIntracranial

eventsBradyarrhythmias (3rd degree AV block)ReferencesPrinciples of Internal Medicine, (15th Ed.),

Pg. 1265, 1292, 1304

Q 72.  Coarctation of aorta is most commonly associated with

(a) Turner syndrome

(b) TOF

(c) Klinefelter syndrome

(d) Patau's syndrome

Answer : Unattempted

Explanation :

Turner syndromeGenotype is XOMajor cardiovascular manifestations–coarctation of aorta,

bicuspid aortic valve, aortic dilatation.Noncardiac anomalies–short female, patientbroad chest,

lymphedema, webbed neck.TOFVSD; RV Hypertrophy; obstruction to RV outflow; aortic override

(straddle) of the VSDKlinefeltersyndromeSole to pubic height more than normal; sterility;

gynecomastia; small penis; small testis; high pinched voice.Patau's syndromeCardiovascular–VSD,

RV Patent ductus arteriosus, double outlet RVNoncardiac anomalies–single midline intracerebral

ventricle with midfacial defect, polydactyly, nail changes, MRReferencesHarrison's Principles of

Internal Medicine, (15th Ed.), Pg. 1333, TableShaw's Textbook of Gynecology, (12th Ed.), Pg. 100

Q 73.  In Alzheimer disease plaque on imaging are seen in

(a) Frontal lobe

(b) Parietotemporal lobe

(c) Temporal lobe

(d) Cerebellum

Page 27: Elsevier Jan

 

Answer : Unattempted

Explanation :

Alzheimer's disease (AD) is the most common cause of dementia in western countries.Pathologically,

there is gross, diffuse atrophy of the cerebral cortex with secondary enlargement of the ventricular

system.PET has indicated that the earliest metabolic change in AD occurs in parietal cortex and the

most severe pathology usually seen is the hippocampus, temporal cortex, and nucleus basalison

autopsy.The most important microscopic findings are neuritic “sessile” plaques and cytoplasmic

neurofibrillary tangles (NFTs). The neurotic plaques contain a central core that includes a (β-amyloid,

proteo glycans, Apo E, α-antichymotrypsin, and other proteins.The most common risk factors are old

ageand positive family history.Others risk factors are:AluminumMercuryVirusesPrionsFemales >

malesLow education levelLow level of estrogen in post-menopausal women.Note: Use of NSAIDs

decreases the risk.ReferencesHarrison's Principles of Internal Medicine, (15th Ed.), Pg. 2391, 2393

Q 74.  All of the following cause high anion gap acidosis EXCEPT

(a) Starvation

(b) Diabetic ketoacidosis

(c) Methyl alcohol ingestion

(d) Glue sniffing

Answer : Unattempted

Explanation :

Causes of high anion gap metabolic acidosis are:Keto

acidosisDiabeticAlcoholicStarvationToxinsEthylene glycolMethanol (Methyl alcohol)SalicylatesRenal

failure (acute and chronic)Lactic acidosis–shock, circulatory failureMalignanciesInfection-cholera,

malariaCO or cyanidepoisoningINZ, BiguanidesJejunoileal bypass or intestinal

obstructionReferencesHarrison's Principles of Internal Medicine, (15th Ed.), Pg. 289, 285, Table 50.2

Q 75.  Rasmussen aneurysm occurs in

(a) Pulmonary artery

(b) Internal mammary artery

(c) Vertebral artery

(d) Bronchial artery

 

Answer : Unattempted

Explanation :

Massive hemoptysis may ensue as a consequence of the erosion of a fully patent vessel located in the

wall of a cavity. Hemoptysis, however, may also result from rupture of a dilated vessel in the cavity

(Rasmussen's aneurysm).ReferencesHarrison's Principles of Internal Medicine, (15th Ed.), Pg. 1027

Q 76.  Shock lung is

(a) Diffuse alveolar damage

Page 28: Elsevier Jan

(b) Usual interstitial pneumonia

(c) Acute interstitial pneumonia

(d) Organizing pneumonia

Answer : Unattempted

Explanation :

Shock is recognized as a major cause of acute lung injury and subsequent acute respiratory syndrome

(ARDS).These disorders are characterized by non-cardiogenic pulmonary edema secondary to diffuse

pulmonary capillary endothelial and alveolar epithelial injury, hypoxemia, and B/L diffuse pulmonary

infiltrates.Shock lung (ARDS) is vascular collapse caused by bacterial sepsis or

traumaReferencesHarrison's Principles of Internal Medicine, (15th Ed.), Pg. 23Robbin's Basic

Pathology, (6th Ed.), Pg. 120, 676

Q 77.  In orthostatic hypotension on standing, which of the follow ing is NOT seen?

(a) Resting tachycardia

(b) Bradycardia

(c) Silent MI

(d) Orthostatic hypotension

 

Answer : Unattempted

Explanation :

Symptoms of autonomic neuropathy (dysautonomia) are mainly negative (i.e., loss of function) and

include postural hypotension with faintness or syncope, anhydrosis, hypothermia, bladder atony,

constipation, dry mouth and dry eyes from failure of salivary and lacrimal glands to secrete, blurring of

vision from lack of pupillary, and ciliary regulation and sexual impotence in males.Positive

phenomena (hyperfunction) may also occur and include episodic hypertension, diarrhea, hyperhidrosis

and either tachycardia or bradycardia.In DM autonomic neuropathies affecting the cardiovascular

system cause a resting tachycardia and orthostatic hypotension. Reports of sudden death have also

been attributed to autonomic neuropathy.Autonomic neuropathy in DM affecting cerebrovascular

symptoms are:Postural hypotensionResting tachycardiaFixed heart rateWith all the above

References, we should go with the choice B–bradycardia.ReferencesHarrison's Principles of

Internal Medicine, (15th Ed.), Pg. 2123, 2506Davidson's Principles and Practice of Medicine, (15th

Ed.), Pg. 676

Q 78.  A patient presented in the OPD with investigational reports showing MCV-55, Hb-9, platelet

count-40,000, RBC-4.5 million, and history of blood transfusion; the diagnosis is

(a) Iron deficiency

(b) Thalassemia minor

(c) Thalassemia major

(d) B12 deficiency

Answer : Unattempted

Page 29: Elsevier Jan

Explanation :

Features of thalassemia minorProfound microcytosis and hypochromia with target cellsMCV rarely

>75 fLHematocrit rarely ≪30–33%Anemia is minimal or mildDiagnosis is clear by the picture of all

the choices.Fe deficiency anemia features:MCV 70–90 fLNormocytic normochromic anemiaSI serum

Fe ≪30TIBC (total iron binding capacity) >360Saturation ≪10%Iron stores 0Serum ferritin values are

most convenient lab test to estimate Fe stores.Features of thalassemia majorSevere

anemiaMacrocytosis (MCV increased)Increased HbF, HbA2, or bothPatient requires chronic

hypertransfusion therapy to maintain hematocrit at a level of 27–30% or so.Features of

B12 deficiencyMCV >100 fLLeucopeniaThrombocytopeniaDecreased leucocytesBut if MCV >110 fL

the likely diagnosis is megaloblastic anemia.ReferencesHarrison's Principles of Internal Medicine, (15th

Ed.), Pg. 663, 672, 674

Q 79.  A 45-year-old male presented with mediastinal tumor and microscopic features of sheets of

lymphocytes with arboriz ing pattern having keratin immunoreactivity; the diagnosis is

(a) Thymoma

(b) Thymic carcinoid

(c) Hodgkin

(d) β-cell lymphoma

 

Answer : Unattempted

Explanation :

MC Mediastinal masses inAnterior mediastinumThymomaLymphomaTeratomatous

neoplasmsThyroid massesMiddle mediastinumVascular massesLN enlargement by metastasis or

granulomatous diseasePleuropericardial cystBronchogenic cystPosterior mediastinumNeurogenic

tumorMeningoceleMeningomyeloceleGastroenteric cystEsophageal

diverticulaThymomaMicroscopically, virtually all thymomas are made up of a mixture of epithelial cells

and a variable infiltrate of non-neoplastic lymphocytes; they may have Hassall's corpuscles with

poorly formed whorls representing residual normal thymic tissue having keratinized cores.Malignant

thymoma type I: have palisading cells about blood vessels. The critical distinguishing feature of this

neoplasm is penetration of the capsule with invasion.Malignant thymoma type II (thymic

carcinoma): Cytologically malignant and majority are Squamous cell Ca either well or poorly

differentiated on lymphoepithelioma composed of cytologically anaplastic cortical type epithelial cells

scattered against a dense background of benign appearing lymphocytes.Hodgkin's lymphomaA

distinctive tumor giant cell known as theReed–Sternberg cell (RS) is considered to be the essential

neoplastic element in all forms of Hodgkin's disease and its identification is essential for the histologic

diagnosis.ReferencesRobbin's Basic Pathology, (6th Ed.), Pg. 643, 727, 1167

Q 80.  In Goodpasture's syndrome, which of the following is NOT TRUE?

(a) Presence of antibasement antibodies

(b) Collagen is affected

(c) Glomeruler nephritis

Page 30: Elsevier Jan

(d) Leukocytoclastic vasculitis

Answer : Unattempted

Explanation :

Anti-GBML (glomerular basement membrane) diseases is an autoimmune disease.The clinical

complex or anti-GBM nephritis and lung hemorrhage is referred to as Goodpasture's syndrome.The

target antigen is a component of the noncollagenous (NCL) domain of the a-3 chain of type IV

collagen, the a-3 chain being preferentially expressed in glomerular and pulmonary alveolar base ment

membrane.Binding of anti-GBM antibodies to the GBM induces activation of complement, leukocyte

recruitment, necrotizing proliferative glomerulonephritis, and disruption of glomerular capillary wall.

Anti-GBM antibodies are detected in the serum of >90% of patients.ReferencesHarrison's Principles of

Internal Medicine, (15th Ed.), Pg. 1583

Q 81.  About cystinosis NOT TRUE is

(a) Fanconi syndrome

(b) Cysteine stone

(c) Blonde hairs

(d) Increased cysteine in cells

 

Answer : Unattempted

Explanation :

Cystinosis is characterized by intra-lysosomal accumulation of free cystine in the body tissue. This

resembles the appearance of cysteine crystals in the cornea, conjunctiva, bone marrow, lymph nodes,

leukocytes, and internal organs.Three variants– Infantile (nephropathic) form–leading to Fanconi

syndrome and renal insufficiency in the first decade of life.– Juvenile (intermediate) form–in which

renal disease manifests in the second decade of life.– Adult (benign) form–characterized by deposits

of cysteine in the cornea but not in the kidney.Cystinosis must be considered in any child with vitamin-

D-resistant rickets, Fanconi syndrome, or glomerular insufficiency.Hexagonal or rectangular cysteine

crystals can be detected in the cornea.Cysteine is the least soluble AA; its overproduction leads to

renal, ureteral and bladder stones.ReferencesHarrison's Principles of Internal Medicine, (15th Ed.), Pg.

2308

Q 82.  Which of the following is the differentiating factor for insuli noma and sulphonylurea-induced

hypoglycemia?

(a) Insulin: glucose ratio

(b) Anti-insulin antibodies

(c) Plasma C-peptide level

(d) Plasma insulin level

Answer : Unattempted

Explanation :

An insulin: glucose ratio >0.3 is virtually diagnostic of hypo glycemia due to insulinoma.Though

Page 31: Elsevier Jan

quantitatively secretion of insulin is more in insulinoma, about >70% is secreted as pro-insulin and here

level of plasma insulin (the standard tests only detect plasma insulin and not total insulin which

includes plasma insulin and proinsulin) and of C-peptide may be spuriously similar in both the

hypoglycemic conditions.In both of the conditions, there is normal biochemical mechanism of secretion.

So C-peptide will be secreted with every molecules of insulin secreted whether by insulinoma or by

sulphonylurea-stimulated pancreas.As insulinoma is a slowly growing condition, sympathetic activation

maintains normoglycemia despite raised plasma insulin levels and hypoglycemia is seen intermittently.

Whenever the tumor is stimulated, stored secretions are released.

Q 83.  Rituximab is NOT USED in

(a) RA

(b) SLE

(c) PNH

(d) NHL

 

Answer : Unattempted

Explanation :

Rituximab is human monoclonal antibody.It depletes B-cellsIts half-life is 1–15 daysUsed as IV

infusionUsed inSLERANHL (B-cell type)B-cell LeukemiaOther connective tissue disorders

Q 84.  The rate of cerebral atrophy, specifically of the hippocampus and medial part of the temporal

lobe, is accelerated in the early stage of

(a) Alzheimer's disease

(b) Parkinson's disease

(c) Wilson's disease

(d) Pick's disease

Answer : Unattempted

Explanation :

Alzheimer's diseases (AD)Pathologically, there is gross, diffuse, atrophy of the cerebral cortex with

secondary enlargement of the ventricular system.Microscopic findings are neuritic sessile plaques and

cytoplasmic neurofibrillary tangles (NFTs) and earliest metabolic changes are seen in the parietal

cortex andhippocampus, temporal cortex, and nucleus basalis.Parkinson's diseaseEosinophilic

intraneural inclusion granules (Lewy bodies) are present in the basal ganglia, brainstem, spinal

cord, and sympathetic ganglia.Wilson's diseasesThe effect of the copper toxicity in the brain is seen

most in the lenticular nuclei and less commonly in the pons, medulla, thalamus,

cerebellum, andcerebral cortex.Pick's diseaseCerebral atrophy confined to frontal or fronto-parietal

lobeReferencesHarrison's Principles of Internal Medicine, (15th Ed.), Pg. 2274, 2292, 2293, 2390,

2395

Q 85.  Pancytopenia with cellular marrow is a feature of

Page 32: Elsevier Jan

(a) PNH

(b) G6PD deficiency

(c) Acquired aplastic anemia

(d) Thalassemia

 

Answer : Unattempted

Explanation :

It is the most consistent blood finding in anemia, which may range from mild to moderate to very

severe. The anemia is usually normo-macrocytic, unremarkable red cell morphology; if MCV is high, it

is usually largely accounted for by reticulocytosis, which may be quite marked (upto 20%, or upto

400,000/μL). Neutropenia and/or thrombocytopenia may or may not be present. In conjugated bilirubin

is mildly or moderately elevated, LDH is typically markedly elevated, and haptoglobin is usually

undetectable. Hemoglobin urea may be overt in a random urine sample. The bone marrow is usually

cellular with marked to massive erythroid hyperplasia, often with mild to moderate dyserythropoietic

features. At some stage of the disease the marrow may become hypocellular or even frankly

aplastic.ReferencesHarrison's Principles of Internal Medicine, (17th Ed.), pg.693

Q 86.  Which of the following is the absolute finding of bone marrow biopsy?

(a) Acute leukemia

(b) Megaloblastic anemia

(c) Hairy cell leukemia

(d) Thalassemia

Answer : Unattempted

Explanation :

In hairy cell leukemia, the malignant cells appear to have “hairy” projections on light and electron

microscopy and show a characteristic staining pattern withtartarate-resistant acid phosphatase. Bone

marrow is typically not able to be aspirated, and biopsy shows a pattern of fibrosis with diffuse

infiltration by the malignant cells.ReferencesHarrison's Principles of Internal Medicine, (17th Ed.), Pg.

640, 697

Q 87.  A 80-year-old, asymptomatic man presents with a total leucocyte count of 1 lakh, with 80%

lymphocytes and 20% PMC's. Most probably this patient is suffering from

(a) HIV

(b) CML

(c) CLL

(d) TB

 Answer : Unattempted

Explanation :

In the question, following things are the reason for the diagnosis—age, 80 years; asymptomatic;

Page 33: Elsevier Jan

gender–male; TLC–1 lakh; lymphocytes 80%; with 20% PMCs.In CLL, following important points, we

should knowMedian age—65 yearsMale percentage—53%Increased number of circulating

lymphocytes (i.e., > 4 × 109/L and usually > 10 × 109/L) and same finding in bone marrow confirm the

diagnosis.Peripheral blood smear shows many “smudge” or “basket” cells, nuclear remnants of cells

damaged by the physical shear stressTrisomy 12 is found in 25% to 30% of patientsTypically, CLL is

often found incidently when a complete blood count is done for other seasonSo, all findings are present

in question aboveReferencesHarrison's Principles of Internal Medicine, (17th Ed.), Pg. 693

Q 88.  Most common catheter-induced blood infection is due to

(a) Staphylococcus aureus

(b) Coagulase negative staphylococci

(c) Gram-negative bacilli

(d) Candida

Answer : Unattempted

Explanation :

Coagulase-negative staphylococci constitute a major component of the normal flora of the human

body. S epidermidis is the most common organism associated with stitch abscesses. It has a prediction

for growth on implanted foreign bodies, such as artificial heart valves, shunts, intravascular leading to

bacteremia.ReferencesHarrison's Principles of Internal Medicine, (17th Ed.), Pg. 819Textbook of

Microbiology, Ananthnarayan, (7th Ed.), Pg. 200

Q 89.  Which of the following is not associated with pituitary apoplexy?

(a) Hyperthyroidism

(b) Diabetes mellitus

(c) Sickle cell anemia

(d) Hypertension

 

Answer : Unattempted

Explanation :

Pituitary apoplexy acute intrapituitary hemorrhagic vascular events.CausesPituitary

adenomaPostpartum (Sheehan's syndrome)DiabetesHypertensionSickle cell anemiaAcute

shockClinical featuresIt is an endocrine emergency that may result in severe hypoglycemia,

hypotension, CNS hemorrhage, and death. Acute symptom may include severe headache with signs of

meningeal titration, B/L visual changes, ophthalmoplegia, cardiovascular collapse, and loss of

consciousness.DiagnosisCT or MRI reveals signs of intratumoral or sellar hemorrhage with deviation

of pituitary stalk and compression of pituitary tissue.TreatmentPatient without visual loss or impaired

consciousness are treated by observation and high dose of glucocorticoidsPatients with visual loss and

impairment of consciousness require urgent surgical decompression.PrognosisVisual recovery

inversely related to length of time after the acute event hypopituitarism is very

common.ReferencesHarrison's Principles of Internal Medicine, (17th Ed.), Pg. 2198

Page 34: Elsevier Jan

Q 90.  Hemolytic uremic syndrome is not associated with

(a) EHEC

(b) Shigella

(c) Campylobacter

(d) Vibrio cholera

Answer : Unattempted

Explanation :

HUS is associated with the following:Shiga toxin–producing E Coli (0157; H7), most common

(MC)Shigella dysenterial type-1Streptococcal pneumoniaeCampylobacter

JejuniHIVReferencesHarrison's Principles of Internal Medicine, (17th Ed.), Pg, 723, 1813Davidson's

Principles and Practice of Medicine, (19th Ed.), Pg. 349, 610, 611

Q 91.  Infective endocarditis due to Pseudomonas is most commonly seen with

(a) Intravenous drug abuse of pentazocine

(b) HIVpatient

(c) Chronic steroid therapy

(d) Elderly with community-acquired pneumonia

 

Answer : Unattempted

Explanation :

Infective endocarditis due to P. aeruginosa is seen mainly in IV drug users whose native values are

involved. This organism has also been reported to cause prosthetic valve

endocarditis.ReferencesHarrison's Principles of Internal Medicine, (17th Ed.), Pg. 952, 789

Q 92.  Which of the following is not a causes of raised JVP with hypotension?

(a) Cardiac tamponade

(b) Right ventricular MI

(c) Heart failure

(d) 2nd degree AV Block

Answer : Unattempted

Explanation :

A hypotensive patient with decreased intravascular and cardiac volume status may have a history

suggesting hemorrhage or other volume losses (vomiting, diarrhea, polynin). JVP is often reduced in

such a patient. A hypotensive patient with increased intravascular volume status and cardiac

dysfunction may have S3 and/or S4 gallops on cardiac examination, and increased JVP.In the early

stages of HF, the venous pressure (NP) maybe normal at rest but may become abnormally elevated

with sustained pressure of the abdomen.Increased JVP with hypotension is associated with the

following:Cardiac tamponadeConstrictive pericarditisRight HFRestrictive cardiomyopathyRight

ventricle MIReferencesHarrison's Principles of Internal Medicine, (17th Ed.), Pg. 1673, 1446, 1490,

Page 35: Elsevier Jan

1419

Q 93.  Which of the following is not a feature of AML?

(a) Retinoic acid is used in treatment

(b) 15/17 translocation may be seen

(c) CD 15/34 both seen in same cell

(d) Associated with disseminated intravascular coagulation (DIVC)

 

Answer : Unattempted

Explanation :

Tretinoin (45 mg/m2 per day orally until remission is documented) plus concurrent anthracycline

chemotherapy appears to be among the safest and most effective treatments for APL (acute

promyelocytic leukemia). Unlike patients with other types of AML, patients with this subtype benefit

from maintenance therapy with either tretinoin or chemotherapy.AML FAB M3 is now designated

acutepromyelocytic leukemia (APL), based on the presence of either the t(15;17) (q22;q12) cytogenetic

rearrangement or the PML/RAR α product of the translocation. Similar examples exist with a variety of

other balance translocations and inversions, including the t(8;21), t(9;11), t (6;9) and inv (16).Various

physical findings are as follows:Ecchymosis and oozing from IV sites (DIC, possible acute

promyelocytic leukemia)Fever and tachycardia (signs of infection)Papilledema, retinal infiltrates, cranial

nerve abnormalities (CNS leukemia) poor dentition, dental abscessesGum hypertrophy (leukemic

infiltration, most common in monocytic leukemia)Skin infiltration or nodules (leukemia infiltration MC in

monocytic leukemiaLymphadenopathy, splenomegaly, hepatomegalyBack pain, lower extremity

weakness (spinal granulocytic sarcoma, most likely in t(8;21) patientsReferencesHarrison's Principles

of Internal Medicine, (17th Ed.), Pg. 682, 678, 680

Q 94.  All of the following statements about mycosis fungoides are true EXCEPT

(a) It is the most common form of cutaneous lymphoma

(b) Pautriers microabscess

(c) Indolent course and easily amenable to treatment

(d) Erythroderma seen and spreads to peripheral circulation

Answer : Unattempted

Explanation :

Mycosis fungoides is also known as cutaneous T-cell lymphoma. This lymphoma is more often seen

by dermatologists. Median age is mid fifties, more common in males and in blacks. It is an indolent

lymphoma with patient often having several years of eczematous or dermatitic skin lesions before the

diagnosis is finally established. In advanced stages, the lymphoma can spread to lymph nodes and

visceral organs. Patients with this lymphoma may developgeneralized erythroderma and circulating

tumor cells, called Sezary's syndrome. Rare patients with localized early stage mycosis fungoides can

be cured with radiotherapy, often total skin electron beam irradiation. More advanced disease has been

treated with topical glucocorticoids, topical nitrogen mustard, phototherapy, psoralen with ultraviolet. An

electron-beam irradiation (PUVA),interferon, antibodies, fusion toxins, and systemic cytotoxic

Page 36: Elsevier Jan

therapy.ReferencesHarrison's Principles of Internal Medicine, (17th Ed.), Pg. 697

Q 95.  Which of the following is correct ECG finding in Hypokalemia?

(a) Increased PR interval with ST depression

(b) Increased PR interval with peaked T wave

(c) Prolonged QT interval with T wave inversion

(d) Decreased QT interval with ST depression

 

Answer : Unattempted

Explanation :

The ECG changes in hypokalemia are due to delayed ventricular repolarization and do not correlate

well with the plasma K+ concentration. Early changes includeflattening or inversion of the T wave have,

a prominent U wave, ST segment depression, and a prolonged QU interval. Severe K+ depletion may

result in aprolonged PR interval, decreased voltage, and widening of QRS component and an

increased risk of ventricular arrhythmias.ReferencesHarrison's Principles of Internal Medicine, (17th

Ed.), Pg. 282

Q 96.  A person with mitral regurgitation and atrial fibrillation presents with syncope. On examination

the person has a heart rate of 55. Which of the following is the most common cause?

(a) Digitalis toxicity

(b) Incomplete heart block

(c) Stroke

(d) Subarachnoid hemorrhage

Answer : Unattempted

Explanation :

In MR associated with atrial fibrillation, digitalis is indicated. Digitalis toxicity may produce almost all

types of arrhythmias— pulsus bigeminus, ventricular extrasystoles, VT, and terminally fibrillation.

Partial to complete AV block may play the role in cardiac toxicity or it may accompany other

arrhythmias. Severe bradycardia, atrial extrasystoles, and AF to AH have also been noted. Severe

arrhythmias due to digitalis are more likely to occur in patients with hearts damaged by other

disease.ReferencesDavidson's Principles and Practice of Medicine, (19th Ed.), Pg. 458

Q 97.  A 30-year-old man presents with generalized edema and hypertension. Urine examination

shows subnephrotic proteinuria (>2 gm) and microscopic hematuria. Serum complement levels

are decreased and he is positive for antihepatitis C antibodies. Which of the following is the

most probable diagnosis in this patient?

(a) Poststreptococcal glomerulonephritis (PSGN)

(b) Mixed cryoglobulinemia

(c) Membranoproliferative glomerulonephritis (MPGN)

Page 37: Elsevier Jan

(d) Focal segmental glomerular sclerosis (FSGS)

 

Answer : Unattempted

Explanation :

It is an immune-mediated glomerulonephritis, 70% of patients have hypocomplementemia. MPGN is

divided into three types type I, type II, and type III.Type I MPGN is commonly associated with

persistent hepatitis C infections,autoimmune disease like lupus or cryoglobulinemia, or neoplastic

diseases.Type II and type III MPGN are usually idiopathic. Type I MPGN on renal biopsy shows

characteristic tram tracking. Patients present with proteinuria, hematuria, pyuria (30%), systemic

symptoms of fatigue children with type I disease, or an acute nephritic picture with RPGN. Nephrotic

syndrome, hypertension, and renal insufficiency, all predict poor outcome.ReferencesHarrison's

Principles of Internal Medicine, (17th Ed.), Pg. 1789

Q 98.  A 60-year-old woman presents with generalized edema, skin ulceration, and hypertension. Urine

examination shows subnephrotic proteinuria (>2 gm) and microscopic hematuria. Serum

complement levels are decreased and she is positive for anti-hepatitis C antibodies. Most

probable diagnosis would be

(a) PSGN

(b) Essential mixed cryoglobulinemia

(c) Membranoproliferative glomerulonephritis

(d) Focal, segmental glomerulosclerosis

Answer : Unattempted

Explanation :

Skin ulcerations in 60-year-old women are features more commonly associated with essential

cryoglobinuria in association with all other features explained in previous

question.ReferencesHarrison's Principles of Internal Medicine, (17th Ed.), Pg. 1789

Q 99.  Hypophosphatemia is not seen in

(a) Acute renal failure

(b) Resolving phases of diabetic ketoacidosis

(c) Respiratory alkalosis/COPD

(d) Chronic alcoholism

 Answer : Unattempted

Explanation :

Hyperphosphatemia is an almost invariable complication of ARF. Severe hyperphosphatemia may

develop in highly catabolic patients or following rhabdomyolysis, hemolysis, or tissue ischemia.Various

causes for hypophosphatemia are as follows:HyperparathyroidismFanconi

syndromeCystinosisWilson diseaseMc-cune-Albright syndrome (fibrous dysplasia)Poorly controlled

DMAlcoholismHyperaldosteronismHypomagnesemiaAmyloidosisHUSRenal transplantation or partial

liver resectionDrugs like ethanol, acetazolamide, high dose of estrogens or glucocorticoids, heavy

Page 38: Elsevier Jan

metals (lead, cadmium), toluene, cisplatin, ifosfamide, foscarnet, calcitonin,

pamidronateReferencesHarrison's Principles of Internal Medicine, (17th Ed.), Pg 1759, 2370

Q 100.  Nocardia cannot be stained with

(a) Mucin stain

(b) Acid-fast stain

(c) Kirem's stain

(d) Alcian blue

Answer : Unattempted

Explanation :

Most Nocardia are acid fast in direct smears. If a weak acid is used for discoloration (e.g., in the

modified Kinyoun, Ziehl-Neelsen, and Fite-Faraco methods), the organism often takes up silver

stain.The question is wrongly framed and it should probably be framed as “Nocardia are stained with”.

The answer should be (b) only in such a case.Partial acid fastness is a unique characteristic

of Nocardia that is not exhibited by other actinomycetes.In paraffin section, the organism can be

demonstrated by silver stain, tissue Gram stains or Fite-Faraco acid-fast stains. (Article from Achievers

of pathology lab medicine.)ReferencesHarrison's Principles of Internal Medicine, (15th Ed.), Pg. 1007

Q 101.  Selective medium used for Vibrio cholerae is

(a) TCBS medium

(b) Thayer martin medium

(c) Skirrow's medium

(d) Loeffler's medium

 

Answer : Unattempted

Explanation :

Various media used for the V. cholerae areHolding or transport media– VR medium-pH 8.6-8.8–

Cary-Blair media-pH 8.4– Autoclaved sea waterEnrichment media– Alkaline peptone water-pH 8.6–

Mansur's taurocholate tellurite peptone water-pH 9.2Plating media– BSA-Alkaline bile salt agar-pH

8.2– GTTA-Mansur's gelatin taurocholate trypticase tellurite agar– TCBS-thiosulfate, citrate, bile salts,

and sucrose.Laboratory isolation of the organisms requires the use of a selective medium.The

best of these is TCBS agar on which the organism grown as a flat yellow colony.Choice (b): Thayer

Martin agar media is for Neisseria gonorrheaChoice (c): Skirrow's medium is used forCampylobacter

jejuniChoice (b): Loeffler's medium used for Mycobacterium tuberculosisReferences1. Textbook of

Microbiology, Ananthnarayan, (6th Ed.), Pg. 211, 282]] 2.Harrison's Principles of Internal Medicine,

(15th Ed.), Pg. 983, 2000

Q 102.  Which of the innate immunity components are active against viral cells?

Page 39: Elsevier Jan

(a) NK cells

(b) Cytotoxic T cells

(c) B cell

(d) Memory B cell

Answer : Unattempted

Explanation :

NK (Natural killer) cells possess spontaneous cytotoxicity toward various target cells, mainly malignant

and virus infected cells. Their cytotoxicity is not antibody dependent or MHC restricted. NK activity

is “Natural” or “nonimmune”. NK cells therefore form part of the innate immune set

up.References1. Textbook of Microbiology, Ananthnarayan, (6th Ed.), Pg. 118]] 2. Harrison's Textbook

of Internal Medicine, (16th Ed.), Pg. 1025

Q 103.  Which on culture shows gram positive cocci in chains and hemolytic colonies?

(a) Bile solubility

(b) Optochin sensitivity

(c) Bacitracin sensitivity

(d) Catalase positive

 

Answer : Unattempted

Explanation :

Streptococci are Gram +ve cocci arranged in chains or pairs.Streptococci are classified on the basis of

their hemolytic properties.190/P—sensitivity to bacitracin is employed as a convenient method for

differentiating Str. pyogenes from other hemolytic streptococci.ReferencesTextbook of Microbiology,

Ananthnarayan, (6th Ed.), Pg. 187, 190

Q 104.  Traveler's diarrhea is caused due to

(a) Enterotoxigenic E. coli

(b) Enterohemorrhagic E. coli

(c) Enteropathogenic E. coli

(d) Enteroinvasive E. coli

Answer : Unattempted

Explanation :

Persons from developed countries visiting endemic areas often suffer from ETEC diarrhea—a condition

known as Traveler's diarrhea.References1. Textbook of Microbiology, Ananthnarayan, (6th Ed.), Pg.

256]] 2. Harrison's Textbook of Internal Medicine, (16th Ed.), Pg. 72

Q 105.  Which of the following streptococcal components cross reacts to synovium of humans

tissues?

(a) Carbohydrate C antigen

Page 40: Elsevier Jan

(b) Polysaccharide capsule

(c) Capsular hyaluronic acid

(d) Cell wall antigen

 

Answer : Unattempted

Explanation :

Various structural components of Streptococcus pyogenes exhibit antigenic cross-reaction with

different tissues of the human body. Antigenic relationships have been demonstrated

between:Capsular hyaluronic acid and human synovial fluidCell wall protein and myocardiumGroup A

carbohydrate and cardiac valvesCytoplasmic membrane antigens and vascular intima,

andPeptidoglycan and skin antigensIt has been postulated that these antigenic cross reactions may

account for some of the manifestations of rheumatic fever and other streptococcal diseases, the tissue

damage being of an immunological nature.ReferencesTextbook of Microbiology, Ananthnarayan, (7th

Ed.), Pg. 206

Q 106.  Which of the following organisms is least susceptible to disinfectants and antiseptics?

(a) Spore

(b) Prions

(c) Protozoa

(d) Fungi

Answer : Unattempted

Explanation :

Chemical disinfectants which can be safely applied to the skin or mucus membrane and are used to

prevent infection by inhibiting the growth of bacteria are called antiseptics.Prions - These infectious

agents are unique in being protein in nature, devoid of DNA and RNA, and unusually resistant to

physical and chemical agents such as heat, irradiation and fomentation - AnanthnarayanPrions are

resistant to inactivation by heat, disinfectants, and radiation.Note:Various organisms are arranged as

follows, according to decreased resistance to antibiotics prions, bacterial spores, protozoal oocysts,

helminths eggs, mycobacteria, small nonenvolved viruses, protozoal cysts, fungal spores, Gram-

negative bacteria, vegetative fungi and algal, vegetative helminths and protozoa, large noninveloped

viruses, Gram-positive bacteria, enveloped viruses.References1. Textbook of Microbiology, P.

Chakraborty, (1st Ed.), Pg. 580]] 2. Textbook of Microbiology, Ananthnarayan, (7th Ed.), Pg. 24, 567]]3.

Antiseptics, Disinfections and Sterilization, McDonell's, (1st Ed.), Pg. 33

Q 107.  Prion protein is

(a) Helical

(b) Random

(c) α-helix

(d) Extended

 Answer : Unattempted

Page 41: Elsevier Jan

Explanation :

Creutzfeldt- Jakob disease (CJD) is a degenerative disease of the CNS that is caused by infectious

protein called prion.Prion reproduces by binding to the normal cellular isoform of the prion (PrPc) and

stimulating its conversion into the disease causing isoform PrPsc.Prion protein, PrPc is rich inα-

helix and has little β-helix while PrPsc has less a-helix and a high (3-helix content.Prions are

proteinaceous infectious particles that lack nucleic acid.Prions are composed largely if not entirely of

PrPsc molecules.They can cause scrapie in animals and related neurodegenerative diseases of

human such as CJD; Scrapie agent is a synonym.ReferencesHarrison's Principles of Internal

Medicine, (15th Ed.), Pg. 2486

Q 108.  A patient in an ICU is on central venous line for the past one week. He is on ceftazidime and

amikacin. After 8 days of antibiotics he develops a spike of fever and his blood culture is

positive for Gram-positive cocci in chains, which are catalase negative. Following this,

vancomycin was started but the culture remained positive for the same organism even after 10

days of therapy. The most likely organism causing this infection is

(a) Staphylococcus aureus

(b) Viridans streptococci

(c) Enterococcus faecalis

(d) Coagulase-negative staphylococcus

Answer : Unattempted

Explanation :

Here we have following things to reach the diagnosis Gram +ve cocci in chains, catalase −ve,

ceftazidim, amikacin and vancomycin resistant.Options A and D are staphylococcus, options B and C

are streptococcus staphylococcus-are Gram +ve cocci in cluster and catalase +ve. While in

streptococcus— viridans streptococci are generally penicillin sensitive but– Enterococcus fecalisstrains

resistant to penicillin and other antibiotics occur frequently, so it is essential to perform antibiotic

sensitivity for proper therapy. So we should go with the option (c).ReferencesTextbook of Microbiology,

Ananthnarayan, (6th Ed.), Pg. 178-79, 187, 196, 199

Q 109.  Which of the following is TRUE regarding Pneumocystis jirovecii?

(a) Occurs only in immune-compromised individuals

(b) Sputum examination is very helpful in diagnosis

(c) Usually associated with CMV infection

(d) Always associated with pneumatocele

 Answer : Unattempted

Explanation :

“Pneumocystis was previously regarded as a protozoan and included in the volume on Parasitology, in

the 9th edition of Topley and Wilson. However, in the present (10th) edition, it is considered with the

other fungi in the Medical Mycology volume”.P. jirovecii is the species found in humans, while P.

carinii is the most common species found in rats. How frequently, we read P. carinii as affecting

Page 42: Elsevier Jan

immunocompromised humans in routine microbiology books! Actually, it is P. jirovecii.“In 1999, Frenkel

formally described Pneumocystis. He retained his originally proposed name for the species found in

humans, P. jirovecii and the name for the most common organism found in rats as P. carinii”!P.

jirovecii causes opportunistic pneumonia infection in young immunocompromised adults.Pneumocystis

may be associated with CMV, but not usuallyA definitive diagnosis requires the demonstration of P.

carinii in the lung in addition to clinical signs and symptoms of the infection. Methods for obtaining

organisms include bronchoalveolar lavage, tracheal aspirates, transbronchial biopsy, bronchial

brushings, percutaneous transthoracic needle aspiration, and open lung biopsy. Induced sputum

samples are useful ifcarinii is found, but the absence of the organism does not exclude the

infection.The epidemic infantile form of P. carinii interstitial plasma cell. Pneumonitis is seen

predominantly in infants between 3 and 6 mo of age. The onset is subtle with tachypnea and 101°

fever, progressing to intercostal suprasternal, and infrasternal real flaring, and cyanosis. In the sporadic

form of P. carinii pneumonitis occurring in children and adults with underlying immunodeficiency, the

onset is usually abrupt with fever, tachypnea, dyspnea, and cough progressing to nasal flaring and

cyanosis.Note:Pneumatocele are seen in staphylococcal infection, pneumocystic disease, past

traumatic (laceration), after treatment of metastasis).Risk factors of pneumocystic disease are–

Immunocompromized patients (HIV), primary immunodeficient diseases, patients on immuno-

suppressive therapy. Premature malnourished infants (immunodeficient).References1. Topley and

Wilson's Medical Mycology, (10th Ed.), Pg. 778]] 2.Harrison's Principles of Internal Medicine, (16th

Ed.), Pg. 1194, 1195]] 3. Jawetz, (24th Ed.), Pg. 648, 649

Q 110.  All of the followings is not TRUE regarding Superantigens, EXCEPT

(a) Activate very large number of T cells irrespective of their antigenic specificity

(b) Bind to the MHC II and T cell receptors causing T cell activation

(c) Bind to the cleft in the MHC II molecules

(d) Binds directly to the lateral aspect of the TCR beta chain

Answer : Unattempted

Explanation :

Superantigens are viral or bacterial proteins that cross link the variable beta domain of a T cell receptor

to an alpha chain of a class 2 MHC.This cross linkage provides an activating signal that induces T cell

activation and proliferation in the absence of antigen specific recognition of peptides 1 MHC class 2

groove because superantigens bind outside of antigen binding cleft.They activate any clones of T cells

expressing a particular variable beta sequence and thus cause polyclonal activation of T cells, resulting

in the overproduction of IFN gamma.This in turn activates macrophages, resulting in overexpression of

proinflammatory cytokines (IL 1, IL 6 and TNF alpha).Excess amounts of these cytokines induce

systemic toxicity molecules produced during infectious processes.Molecules known to act as

superantigens include staphylococcal enterotoxins, Toxic shock syndrome toxin 1(TSST 1) and

streptococcal pyrogenic exotoxins.Conventional ‘Simple Antigen’– Bind to MHC class II molecule in the

antigen binding groove or cleft (groove of the ab heteodimer)– Bind to the T-cells at the T-cell receptor

region (TCR)– Stimulation of the T-cells is dependent on the antigenic specificity of T-cells.– Stimulate

smaller no. of T-cells (1 to 10,000 T-cells)– Release of inflammatory cytokines

Page 43: Elsevier Jan

(Menokines/Lymphokines) is less.References1. Textbook of Microbiology, Ananthnarayan, (7th Ed.),

Pg. 142, 197, 207]] 2. Harrison's Principles of Internal Medicine, (7th Ed.), Pg. 2034

Q 111.  Which one of the following is true?

(a) Agar has nutrient properties

(b) Chocolate medium is selective medium

(c) Addition of selective substances in a solid medium is called enrichment media

(d) Nutrient broth is a basal medium

 

Answer : Unattempted

Explanation :

Simple media/basal media-Example is nutrient broth. It consists of peptone, meet extract, NaCl, and

water. Nutrient agar, mode by adding 2% agar to nutrient broth is the simplest and most common

medium. If the concentration of agar is reduced to 0.2-0.5%, semisolid or sloppy agar is obtained which

enables motile organisms to spread. Increasing the concentration of agar to 6% prevents spreading or

swarming.

Q 112.  Regarding Androgen insensitivity syndrome, all are true EXCEPT

(a) Absent vagina

(b) Absent ovaries

(c) Abundant pubic hair

(d) Normal male karyotype

Answer : Unattempted

Explanation :

There are 5 variants of AIS in terms of increasing severity, and the complete AIS technically is the most

evident in appearance.It presents when the external genitalia formed are totally female like in a male

fetus, because the androgens which give are suppose to give rise to the definitive male genitalia are

not recognized by the end organs.Also there is conversion of the unused androgens to estrogens by

aromatization in the adiposities and these estrogens in turn give rise to the glandular development of

the breast.Hence these individuals with complete AIS or Testicular feminization syndrome are perfect

in appearance as females in spite of actually being males.There is an absence ovaries with

undescended testes and the vagina and uterus are also absent because of the testes producing anti-

mullerian factor which inhibits formation of the female internal genitalia.

Q 113.  Ergometrine is used in management of which of the following?

(a) CAD

(b) Stroke

(c) Migraine

(d) Raynaud's disease

Page 44: Elsevier Jan

 

Answer : Unattempted

Explanation :

Ergotamine is a vaso constrictor drug so it is contraindicated in CAD, CVA and Raynaud's

disease.Adverse effects of ergotamine are nausea, vomiting, abdominal pain, muscle cramps,

weakness, paresthesias, coronary and other vascular spasm, chest pain.The drug is contraindicated in

the presence of sepsis, ischemic heat disease, peripheral vascular disease, hypertension, pregnancy,

liver and kidney disease.References[Essentials of Medical Pharmacology, KD Tripathi, (16th Ed.), Pg.

168

Q 114.  The uterus is supported by all except one of the following

(a) Broad ligament

(b) Uterosacral ligament

(c) Mackenrodt's ligament

(d) Pubocervical ligament

Answer : Unattempted

Explanation :

Broad ligament is merely a fold of peritoneum on either side of the uterus and has no supporting value.

Q 115.  The uterus develops from

(a) Mesonephric duct

(b) Paramesonephric duct

(c) Gartner duct

(d) Metanephros

 

Answer : Unattempted

Explanation :

The uterus develops from paramesonephric duct. Mesonephric and Gartner duct are the remnants of

male organs lying in the broad ligament.

Q 116.  Vagina is developed from

(a) Mullerian duct

(b) Sinovaginal bulb

(c) Mesonephric duct

(d) a + b

#Num# (e)  b + c

Answer : Unattempted

Q 117.  Sensory nerve supply to the uterus is

Page 45: Elsevier Jan

(a) Presacral nerve

(b) Frankenhauser nerve

(c) Pudendal nerve

(d) Ovarian plexus

 Answer : Unattempted

Q 118.  Contraindication of vaginal delivery in female with previous cesarean section includes all

EXCEPT

(a) Previous classical cesarean section

(b) Breech presentation

(c) No history of vaginal delivery in the past

(d) Puerperal infection in previous pregnancy

Answer : Unattempted

Explanation :

Absolute indications for cesarean section are:Central placenta previaContracted pelvis or

cephalopelvic disproportion absoluteInaccessible lower segment as in bladder, fibrosis,

fibroid.Advanced carcinoma cervixVaginal stenosisCommon indications are:Fetal distressCPDPrevious

cesarean sectionComplication breachAntepartum hemorrhage (specially placenta previa with bleeding

or a central placenta)Malpresentation → like direct mentoposterior, brow, deep transverse arrest

etc.Failure to progress (incoordinate uterine action)Hypertensive disorders like sever pre-eclampsia

and uncontrolled seizures in eclampsiaDisease like coarctation of aorta, pelvic, tumors, cancer cervix,

previous VVF repair etc.Note:Previous Classical Cesarean section presents with increased risk of scar

rupture during subsequent labor and one can permit vaginal delivery with greater confidence after

lower segment section than after classical sectionBreech presentation per se in a patient is not an

indication for elective cesarean section, if the baby is average in size and pelvis adequate.Puerperal

infection in previous pregnancy may result in bad healing and hence a weak scar.References[Inputs

from Williams Obstetrics and International Guidelines

Q 119.  Which of the following is the mechanism of abortion sticks?

(a) Stimulation of uterine contraction

(b) Menstrual bleeding

(c) Irritation of the GIT

(d) Irritation of the genito-urinary tract

 Answer : Unattempted

Explanation :

The main mechanism of the use of abortion sticks is to produce an irritant action on the uterus so as to

induce uterine contractions and expulsion of the fetus.Abortion stick is an object used to procure illegal

abortion by unskilled interference by either self or by someone else (a dai/ midwife).It usually consists

Page 46: Elsevier Jan

of a specially made wooden or bamboo stick about 15–20 cm long or a twig of similar length with some

irritant plant such as Madar (Calotropis), Chitra (Plumbago zeylanica) or Kaner (Nerium odorum)

applied at one of its ends.The irritant substance is applied to some cotton wool or a piece of rag at one

end of the stick.The abortion sick is introduced into the os of the uterus thus causing abortion with or

without rupture of the membranes.Excoriation, bruising or perforation of the vagina or uterus may occur

as a result of the irritant action of the substance.References[1. Textbook of Forensic Medicine and

Toxicology, Reddy, (16th Ed.), Pg. 344–45[2. Textbook of Forensic Medicine and Toxicology, Krishan

Vij, (3rd Ed.), Pg. 551–52

Q 120.  Endosalpingitis is best diagnosed by

(a) Hystero laparoscopy

(b) X-Ray abdomen

(c) Hysterosalpingography

(d) Sono salpingography

Answer : Unattempted

Explanation :

Hystero-Laparoscopy also called laparo-hysteroscopy is a procedure in which the tubes can be

examined under direct vision with an endoscope and if the pelvis looks inflamed then the best place to

take a biopsy for culture is the tubal end. Same holds true for the diagnosis of endo-salpingitis.X-Ray

abdomen is too gross a procedure and informs mostly anatomical problems of the pelvis.HSG is a

procedure which will inform regarding the tubal patency and outlines the uterine anatomy along with

uterine polyps and fibroids.There may be some filling defects in the tube recognized with a good HSG if

there is Endosalpingitis but it does not prove the diagnosis as is done by a direct tubal culture.Sono

salpingography is an assessment of the tubal patency by perfusing the uterus and therefore the tubes

while observing the uterus and the pouch of Douglas by an ultrasound. This procedure is good for the

diagnosis of tubal patency but not for infections.References[Shaw's Textbook of Gynecology, (13th

Ed.), Pg. 420

Q 121.  Chorionic villus biopsy is done in all of the following EXCEPT

(a) Neural tube defects

(b) Sickle cell disease

(c) Myotonic dystrophy

(d) Down syndrome

 Answer : Unattempted

Explanation :

Indications of Chorionic villus biopsy:Karyotyping is the most common indication—Down

syndromeHemoglobinopathies —Sickle cell disease, Thalassemia, etc.Biochemical studies—

Gaucher's disease, Niemann-Pick disease, Tay-Sach's disease, etc.Single gene defects—Myotonic

dystrophy, Cystic fibrosis, Huntington's disease, Phenylketonuria, etc.Neural tube defects are

diagnosed by ultrasound and estimation of alpha-fetoprotein in maternal serum/ amniotic

Page 47: Elsevier Jan

fluid.Chorionic villous biopsy can be done after 10th week (earlier than amniocentesis)Earlier results

reduce maternal stress by going for a possible therapeutic abortion (if indicated) at an early

stage.Initially a trans-cervical route is acceptable to take the sample.A trans-abdominal route is taken if

the pregnancy exceeds 12th week.References[1. Langman's Medical Embryology, (10th Ed.), Pg 120,

294[2. Review of Medical Physiology, Ganong, (21st Ed.), Pg. 422

Q 122.  True or false regarding levator ani muscles?

(a) These muscles support the genital tract

(b) They maintain the sphincteric control of faeces.

(c) They cause stress incontinence if they are torn or damaged

(d) The muscles are responsible for internal rotation of fetal head during labour

Answer : Unattempted

Explanation :

The pelvic floor muscles comprising mainly the two levator ani muscles support the genital organs.

Denervation, laceration or weakness after menopause causes genital prolapse, stress incontinence of

urine and faeces.

Q 123.  Normal vaginal pH is

(a) 3 to 4

(b) 4 to 5

(c) 5 to 6

(d) More than 6

 Answer : Unattempted

Q 124.  A 35-year-old female presented with postcoital bleeding, it can be managed by

(a) Clinical examination and PAP smear

(b) Smear taken after treatment with Lugol's iodine

(c) Smear taken after treatment with acetic acid

(d) Smear taken after proper vaginal examination

Answer : Unattempted

Explanation :

PAP smear should be obtained prior to vaginal examination because the fingers may remove the

desquamative cervical cells and give a false negative report, lubricant may prevent detection of

organisms and any vaginal bleeding during examination may preclude proper visualization of the

cervix.The patient is placed in the dorsal position, the labia parted and the Cusco's self retaining

speculum is gently introduced without the use of lubricant and jelly. The cervix is exposed; the

squamocolumnar junction is now scraped with the Ayre's spatula. Scrapings are spread on the shell

and fixed with equal parts of 95% ethyl alcohol and ether.Papain-colaou classification:Grade I–

Page 48: Elsevier Jan

Normal cellsGrade II–Slightly abnormal, suggestive of inflammatory change, repeat smear after

treating the infection Grade III–A more serious type of abnormality need biopsy Grade IV–Distinctly

abnormal, possibly malignant, definitely require biopsyGrade V–Malignant cells seenHence, before

taking the smear if iodine/acetic acid/jelly are used or examination is done, false negative results are

generated.References[Shaw's Textbook of Gynecology, (13th Ed.), Pg. 77

Q 125.  A female patient has epithelial tumor. On screening a high level of CA-125 was found, further

management will be

(a) CT

(b) MRI

(c) PET scan

(d) Examination of the serial CA-125 levels

 

Answer : Unattempted

Explanation :

Tissue marker suggests the histologic nature of the tumor as well as decides the duration of

postoperative chemotherapy or a need for radiotherapy.CA-125 is a glycoprotein and surface cell

antigen and is secreted by the malignant epithelial tumors.CA-125 >35 Umb/mL suggests residual

tumor and indicates the need of chemotherapy.CA-125 is also raised in abdominal

tuberculosis andendometriosis. It should be noted that CA-125 is raised in only 50% of cases in stage I

and 90% in stage II ovarian cancer.In malignant tumors, CT and MRI recognize the spread of the

tumors and enlargement of pelvic and para-aortic lymph nodes >1 cm. This helps in learning about

postoperative radiotherapy or chemotherapy.References[Shaw's Textbook of Gynecology, (13th Ed.),

Pg. 372, 403

Q 126.  In ectopic pregnancy, termination of conception is mainly caused by

(a) Vascular accidents

(b) Maternal antibodies

(c) Hormonal cause

(d) Nutritional inadequacy

Answer : Unattempted

Explanation :

Fate of ectopic pregnancy:Tubal abortion–it is most often seen in cases of ampullary implantation.

Chorio-decidual hemorrhage occurs around the ovum, which gets detected and is expelled into the

tubal lumen. The blood may collect in the pouch of Douglas to form a pelvic hematocele.Tubal

rupture–when the fertilized ovum implants in the isthmic or interstitial portion of the tube then tubal

rupture is more common and the patients come in an acute presentation with shock. If rupture occurs

along the antimesenteric border, then there is profuse intraperitoneal hemorrhage and a pelvic

hematocele form.Incidence 1:300 normal pregnancy; world wideIncreased chance in PID, IUCD,

Endometriosis, IVF, GIFT, surgical process at tube, broad ligament myoma and ovarian tumor;

congenital tube anomalyOccurrence:fallopian tube 95–99%isthmic 25–30%ampullary 50–60%fimbrial

Page 49: Elsevier Jan

17–20%interstitial 1–1.5%tubal stump-rareMedical treatment offered to patients whose HCG level is

<2000 IU and size of mass <3 cmReferences[Manual of Obstetrics, Holland and Brews, (2nd Ed.), Pg.

206

Q 127.  Endometrial thickness in a postmenopausal woman should not exceed

(a) 2 mm

(b) 4 mm

(c) 8 mm

(d) 10 mm

 

Answer : Unattempted

Explanation :

More than 4 mm is considered abnormal and requires curettage to rule out premalignant and malignant

lesions of the endometrium.

Q 128.  Ipsilateral painful ophthalmoplegia with deviation of the eye associated with cavernous sinus

syndrome is seen in

(a) Gradenigo syndrome

(b) Tolosa-Hunt syndrome

(c) Weber's syndrome

(d) Benedikt's syndrome

Answer : Unattempted

Explanation :

Tolosa-Hunt syndromeOrbital apex or superior orbital fissure or cavernous sinus syn drome with optic

nerve involvement and varying degree of ophthalmoplegic migraineCharacterized by painful, acute

ophthalmoplegic migraine with or without involvement of optic nerve and ophthalmic domain of

the trigeminal nerve,and it responds promptly to steroid treatment.The patient should be investigated to

eliminate infraclinoid aneurysms, carotid cavernous fistula, pituitary tumor, meningioma, orbital tumor;

diagnosis should be made often by arteriography and venography.References[Parson's Diseases of

the Eye, (20th Ed.) Pg. 459, 488

Q 129.  Autosomal dominant inheritance is seen in

(a) Gyrate atrophy

(b) Best disease

(c) Laurence–Moon–Biedl syndrome

(d) Bassen–Kornzweig's disease

 Answer : Unattempted

Explanation :

All other diseases are autosomal recessive.References[1. Parson's Diseases of the Eye, (19th Ed.),

Page 50: Elsevier Jan

Pg. 574[2. Clinical Ophthalmology: A Systematic Approach, Kanski, (6th Ed.), Pgs. 667, 672,

694[3.Ophthalmology, Yanoff M, Duker JMD. (2nd Ed.), Pgs. 820, 825, 836

Q 130.  Where is hyaluronic acid seen?

(a) Vitreous humor

(b) Cartilage

(c) Cornea

(d) Aqueous humor

Answer : Unattempted

Explanation :

References[Parson's Diseases of the Eye, (20th Ed.), Pg. 321

Q 131.  Diabetic macular edema is caused by all of the following EXCEPT

(a) Oxidative stress

(b) Increased protein kinase-C

(c) Increased VEGF

(d) Retinal pigment epithelial dysfunction

 

Answer : Unattempted

Explanation :

Diabetic retinopathyThe retinal changes probably originate from a state of anoxia, which causes an

increased permeability of the capillaries, the formation of multiple microaneurysm, and local

degenerative changes in the tissues of the retina.The MC change is due to swelling of the nerve fibers,

giving the appearance of cloud like “Soft aggregates with ill defined margins-soft exudation.”Cotton

wool patchesare seen in superficial layer of retina. They are formed by the arrest of axoplasmic flow at

the edge of an ischemic area where nerve fibers cross the boundary.Other degenerative changes in

the neural elements result from the extravasation of fluid. Extravasated fluid is rich in fibrin and

proteins, which readily coagulates and a characteristic retinal manifestation ishard exudates as

deposition masses.The MC complication of diabetic macular (DM) edema is diabetes retinopathy.The

MC factor associated is duration > 10 years. Earliest feature–microaneurysms.MC change is soft

exudates' or ‘cotton wool patch’.MC cause of spontaneous vitreous hemorrhage is proliferative

diabetes retinopathy (PDR).TOC for PDR is panretinal photocoagulation and LASER.Kimmelstiel-

Wilson syndrome is diabetic retinopathy with glomerulosclerosis.References[Parson's Diseases of the

Eye, (20th Ed.) Pg. 292

Q 132.  A patient presented with left-sided tilt of the head and right-side hypertropia while walking and

it persists while on turn ing head straight and then to left side. It increases on left gaze; the

muscle paralyzed is

(a) Right superior oblique palsy

(b) Left superior oblique palsy

Page 51: Elsevier Jan

(c) Right side superior rectus palsy

(d) Left side superior rectus palsy

Answer : Unattempted

Explanation :

The following lines of Parson have been taken to answer the question:Paralysis of the superior

obliqueThere is hypoesthesia of the affected eye with limitation of the movements downwards and

towards the same side.The face is turned downwards towards the sound side with a slight tilt of the

head towards the shoulder of the sound or the normal side homonymous diplopia occurs on looking

down.The false image is lower and its upper end is tilted towards the trueimage (the image is intorted

because the eye is extorted when thesuperior oblique is paralyzed).The distance between the images

increases on looking down and towards the sound side and inclination of the false image increases on

looking down to the paralyzed side.The patient has great difficulty in going downstairs and vertigo is

normally a premium symptom.A congenital left superior oblique palsy produces a compensatory head

tilt to the right with chin depression and face turned slightly to the right. Thus binocular vision is

maintained in the primary position.When the head is forcibly tilted to the left, reflex intortion is excited.

This can only be undertaken by the left superior rectus, which is stimulated to overact. The result is an

unopposed over elevation of the left eye (Bielschowsky head tilt test).Paralysis of lateral

rectusThere is an esotropia or convergent squint with limitation of movement outwards and the face is

turned towards the paralyzed side.References[Parson's Diseases of the Eye, (20th Ed.) Pg. 416

Q 133.  Global vision 2020 does not include

(a) Cataract

(b) Onchocerciasis

(c) Epidemic conjunctivitis

(d) Trachoma

 

Answer : Unattempted

Explanation :

A global initiative has been launched to reduce avoidable blindness by 2020. The main features are as

follows:Target diseases are identified; the list includes:cataractrefractive errorschildhood

blindnessglaucomadiabetic retinopathytrachomaonchocerciasisHuman resource development–it

involves augmenting the middle level ophthalmic personnel and increasing the capacity and skills of

available human resources in various areas of ophthalmology.Infrastructure development–the

infrastructure proposed consists of a four-tier organization comprising of 20 centers of excellence, 200

training centers, 2000 services centers and 20,000 vision centers.References[1. Parson's Diseases of

the Eye, (20th Ed.) Pg. 528[2. Foundation of Community Medicine, Dhaar and Robbani, (1st Ed.) Pg.

962

Q 134.  Retinitis pigmentosa is associated with all of the following EXCEPT

(a) Marfan's syndrome

(b) Refsum syndrome

Page 52: Elsevier Jan

(c) Usher syndrome

(d) Cockaynae syndrome

Answer : Unattempted

Explanation :

Common syndromes with which retinitis pigmentosa associated are as follows:Laurence-Moon-Biedl,

i.e., Barth syndrome–Obesity, hypogo nadism, mental defects and Polydactyly-AR.Usher's

syndrome–AR, sensorineural deafness.It is pyruvate carboxylase deficiency.Subcente necrotizing

encephalomyelopathy is the main feature.Other names are–MELAB, Rud's syndrome, Hallgren's

syndrome, Usher-Hallgren syndrome, Dystrophy retinal dysacousia syndromeKearns-Sayre

syndrome–also called as Hallervorden-Spitz diseaseIt is due to carbohydrate-deficient glycoprotein

syndrome.Main features are chronic progressive external ophthalmoplegia, heart block, i.e.,

oculocraniosomatic syndrome. Maternally inherited other features are dysphagia, proximal weakness,

cerebellar ataxia cardiac conduction defect, and hearing loss.Cockayne's syndrome–AR, bird-like

face, MR, premature ageing, ataxia, dwarfism, flexion contracture of limbs.Refsum's syndrome–AR,

cerebellar ataxia, deafness, elevated CSF proteins, peripheral neuropathy.Bassen–Kornzweig

syndrome (abetalipoproteinemia) AR, ataxia, acanthocytosis, fat malabsorption.Also known as

HAKD syndrome, Alström syndrome, Stargardt's disease, Sjögren-Larson syndrome, medullary cystic

renal disease, tapetochoroidal dystrophy, Shwachman-Diamond syndrome.It is caused by deficiency of

beta lipoproteins, resulting in intes tinal malabsorption.Friedrich ataxia–ataxia, nystagmus.Marfan

syndrome–it is not associated with retinitis pigmentosaAD, disproportionately long limbs, long thin

fingers, a typicallytall stature, cardiovascular abnormalities. Heart valve aorta, lung, eye, and hard

palate defectsReferences[Parson's Diseases of the Eye, (20th Ed.) Pg. 309

Q 135.  Choroidal neovascularization is seen under retina in all of the following EXCEPT

(a) Trauma

(b) Angioid streak

(c) Myopia

(d) Hypermetropia

 

Answer : Unattempted

Explanation :

Virtually any pathological condition that involves the RPE and dam ages the brush membrane can be

complicated by choroidal neovascularization like:Angioid

streaksARMDMyopiaHistoplasmosisSarcoidosisMelanomaHemangiomaOsteomaTraumaChoroidal

ruptureIdiopathicLaser photocoagulationOsteomaDegenerative conditionsInfectious and inflammatory

conditionsMultifocal choroiditisNeviPICVascular and degenerative choroidal lesions elsewhere in the

fundus, particularly a choroidal neovascular membrane at the macula, are common developments in

the angioid streaks.Trauma to the choroids, a late complication, can cause a choroidal or

suprachoroidal hemorrhage.References[Parson's Diseases of the Eye, (20th Ed.) Pg. 310, 369

Q 136.  Claw hand is due to the involvement of

Page 53: Elsevier Jan

(a) Ulnar nerve

(b) Median nerve

(c) Radial nerve

(d) Posterior interosseous nerve

Answer : Unattempted

Explanation :

The classic claw hand is due to lesion to ulnar nerve.Ulnar paradox—higher the lesion, lesser the

deformity because the ulnar half of flexor digitorum profundus is paralyzed and the fingers are,

therefore, less “clawed.”ReferencesApley's System of Orthopedics and Fractures, (8th Ed.), Pg. 242,

342

Q 137.  Most common force involved in fractures of the spine is

(a) Flexion

(b) Extension

(c) Rotation

(d) Compression

 

Answer : Unattempted

Explanation :

Compression injuries are by far the most common force in vertebral fracture and are due to spinal

flexion.ReferencesApley's System of Orthopedics and Fractures, (8th Ed.), Pg. 659

Q 138.  A 15-year-old boy presented with a mass in the distal femur. X-ray from the lesion showed

features of Codman's triangle and sunray appearance. The diagnosis is

(a) Osteosarcoma

(b) Ewing's sarcoma

(c) Osteoclastoma

(d) Chondroblastoma

Answer : Unattempted

Explanation :

Both the Codman's triangle and sunburst appearance are characteristic features of osteosarcoma.Age

—predominantly in children and adolescents.Long bone metaphyses, especially around the knee and

proximal end of the humerus are commonly involved.ReferencesApley's System of Orthopedics and

Fractures, (8th Ed.), Pg. 185

Q 139.  Pollicization is best described as

(a) Amputation of thumb

(b) Equalization of fingers

Page 54: Elsevier Jan

(c) Toe to thumb transplantation

(d) Reconstruction of thumb

 

Answer : Unattempted

Explanation :

Reconstruction of thumb is called pollicization. Typically this consists of surgically migrating the index

finger to the position of the thumb in patients who are either born without a non-functional thumb (most

common) or in patients who have lost their thumb traumatically and are not amenable to other

preferred methods of thumb reconstruction such as toe-to-hand transfers.During pollicization the index

finger metacarpal bone is cut and the finger is rotated approximately 120–160 degrees and replaced at

the base of the hand at the usual position of the thumb.The arteries and veins are left attached. If

nerves and tendons are available from the previous thumb, these are attached to provide sensation

and movement to the new thumb (“neopollux”).If the thumb is congenitally absent, other tendons from

the migrated index finger may be shortened and rerouted to provide good movement.Loss of thumb

function seriously impairs the entire upper limb and a deficient thumb carries a high priority for

reconstruction. Despite this, some patients accommodate well to thumb amputation, and so

reconstruction is indicated for selected cases.Strong contraindications to elective thumb reconstruction

include:Compromising vascular diseaseBasis for short life expectancy such as:Metastatic

diseaseChronic pain syndrome, andDisuse of the limbUnreconstructable sensory lossUnrealistic

patient expectationsOther contraindications dictated by the common sense of the surgeonReferences1.

Grab and Smith's Plastic Surgery, (5th Ed.), Pg. 9292. Campbell's Orthopaedic Surgery, (11th Ed.), Pg.

660, 662, 664

Q 140.  A 30-year-old HIV positive male who is on antiretroviral therapy has pain in the right hip joint for

2 months. He has difficulty in abduction and internal rotation. Which of the following is most

likely diagnosis?

(a) Septic arthritis

(b) Osteoarthritis

(c) Avascular necrosis

(d) Tubercular arthritis

Answer : Unattempted

Explanation :

All the feature can be seen in tubercular arthritis, osteoarthritis or in avascular necrosis of head of

femur, but here clue for the diagnosis in patient is on antiretroviral therapy which cause the AVN of

hipVarious side effects of antiretroviral drugs are:Gastrointestinal-nausea, Vomiting,

DiarrheaNephrolithiasis, Frank Renal NephropathyDyslipidemiasLipdystrophyAVN-avascular

necrosisHeadache, dizziness, rashes, liver damage etc.TB arthritis of the hip: The disease is of

insidious in onset and runs a chronic course. Pain may be absent in the early stages, or if present, it

may be referred to the knee. The patient complaints of ‘night cries’, caused by the rubbing of the two

diseased surfaces when the movement occurs as a result of the muscle relaxation during sleep. Gross

deformities may be obvious on inspection. Minimal deformities are compensated for by pelvic tilt and

Page 55: Elsevier Jan

can be made obvious by tests. Commonly it is flexion, adduction and internal rotation deformity

of the hip. There is generally a true shortening of limb in TB of the hip, except in Stage I, in which

an apparent lengthening occurs.Stages of TB of the hip:Stage I (stage of synovitis): There is

effusion into the joint which demands the hip to be in a position of maximum capacity.This is a position

of flexion, abduction and external rotation. This stage is also called the stage of apparent

lengthening.It lasts for a very short period.Stage II (stage of the arthritis): Hip takes the attitude of

flexion, adduction and internal rotation. This is also called the stage of apparent shortening.Stage III

(stage of erosion): The attitude of the limb is the same as that in Stage II, i.e., flexion, adduction and

internal rotation except for the fact that the deformities are exaggerated. There is true shortening of

the limb because of the actual destruction of the bone.References1. 1. Essentials of Orthopedics,

Maheshwari, (3rd Ed.), Pg. 181–832. Current Diagnosis and Treatment of Pain, 2006, Pg, 305

Q 141.  Meralgia paresthetica is due to the involvement of which of the following nerve?

(a) Medial cutaneous nerve of thigh

(b) Lateral cutaneous nerve of thigh

(c) Sural nerve

(d) Femoral nerve

 

Answer : Unattempted

Explanation :

The lateral cutaneous nerve of thigh can be compressed as it runs through the inguinal ligament just

medial to the ASIS. The patient complains of numbness, tin gling, or burning discomfort over the

anterolateral aspect of the thigh (meralgia paresthetica).Morton's metatarsalgia—compression of digital

nerves in foot.ReferencesApley's System of Orthopedics and Fractures, (8th Ed.), Pg. 252

Q 142.  Bohler's angle is associated with fracture of

(a) Calcaneum

(b) Talus

(c) Navicular

(d) Cuboid

Answer : Unattempted

Explanation :

Flattening of angle subtended by the pos terior articular surface and the upper surface of body pos

terior to the joint (Bohler's angle) occurs in fracture of calcaneum. It is 25°–40° normally. It is also

known as tuber joint angle.ReferencesApley's System of Orthopedics and Fractures, (8th Ed.), Pg.

750–751

Q 143.  It is wise to keep and repair the meniscus rather than removing it when the injury is to which of

the following?

(a) Medial part of meniscus

Page 56: Elsevier Jan

(b) Mid part of meniscus

(c) Lateral part of meniscus

(d) Associated with collateral ligament injury

 

Answer : Unattempted

Explanation :

Meniscal tear is the most common injury to the knee that requires surgery. The medial meniscus is

torn approximately three times as often as the lateral meniscus.TreatmentPartial Meniscectomy–

Tears that are not amenable to repair are best treated by partial meniscectomy. In general, complex,

degenerative, and central/radical tears are resected with minimal normal meniscus being

resected.Meniscal Repair – Should be done for all peripheral/lateral tears, especially in young

patients and in conjunction with an ACL reconstruction.Meniscal Repair Results – Several studies

report 80–90% success rates with meniscal repairs. However, this depends on location, type of tear

and chronicity. It is accepted that the results of meniscal repair are best with acute, peripheral

tears in young patients undergoing concurrent ACL reconstruction.References1. Review of

Orthopaedics, Miller, (4th Ed.), Pg. 2112. Campbell's Orthopaedic Surgery, (11th Ed.), Pg. 2427–28

Q 144.  A 12-year-old male present with extensive heterotopic ossification over the neck back and

shoulder and decreased chest movements. He gives history of progressive immobility since the

age of 4 years. Which of the following is a false statement about his condition?

(a) Pneumonia is common

(b) Life longevity is normal

(c) Most common site involved is the spine

(d) Onset is before 6 year

Answer : Unattempted

Explanation :

Myositis ossificans progressivaAlso known as Fibrodysplasia Ossificans ProgressivaThis is a

rare, progressive, autosomal dominant inherited disease in which the connective tissues of skeletal

muscles, tendons, ligaments and fasciae become edematous, calcify and ossify.The onset of the

development of the lesion may be at any age but typically before the sixth year.The most common

site of involvement is in the spine and neck where huge sheets of bone are formed fusing the neck

and back into a solid mass; in some, the thigh muscles may be affected.The condition needs to be

differentiated from traumatic myositis ossificansNo treatment is permanently

effective.References1. Pediatric Orthopaedics and Fractures, (2nd Ed.), Pg. 846–472. Tachdjian's

Pediatric Orthopaedics, (3rd Ed.), Pg. 1632

Q 145.  Tympanic membrane develops from:

(a) Ectoderm

(b) Mesoderm

(c) Endoderm

Page 57: Elsevier Jan

(d) All the three germinal layers

#Num# (e)  Only (a) and (c)

 

Answer : Unattempted

Explanation :

Tympanic membrane develops from all the three germinal layers.Outer epithelial layer develops from

epithelium lining the first branchial cleft.Inner mucosal layer develops from the lining of tubotympanic

recess which is a derivative of first pharyngeal pouch and partly of the second pouch.Intermediate

fibrous layer is derived from mesoderm.

Q 146.  Citelli's angle is:

(a) Solid angle

(b) Cerebellopontine angle

(c) Sinodural angle

(d) Genu of facial nerve

Answer : Unattempted

Explanation :

Sinodural angle is also called the Citelli's angle. It is formed at the site where dura of middle cranial

fossa meets the dura of sigmoid sinus. It forms an important landmark in mastoid surgery.McEwen's

triangle is bounded by temporal line, posterosuperior segment of the external auditory canal and a line

drawn as a tangent to the external canal. It is an important landmark to open the mastoid antrum—an

initial step for mastoidectomy. Solid angle is the place where three semicircular canals meet.

Q 147.  Number of centres from which bony labyrinth ossifies

are:

(a) 6

(b) 8

(c) 10

(d) 14

 

Answer : Unattempted

Explanation :

Bony labyrinth, also called the otic capsule, develops from cartilage which later ossifies to form bone.

There are 14 centres of ossification. Ossification starts at 16th week when first centre appears near the

cochlea. The last centre appears at 20th week at the posterolateral part of the posterior semicircular

canal.

Q 148.  Area of stapes footplate is:

(a) 1.5 sq mm

(b) 2.2 sq mm

Page 58: Elsevier Jan

(c) 3.0 sq mm

(d) 3.2 sq mm

Answer : Unattempted

Q 149.  Which of the following is TRUE regarding Bradykinin?

(a) Pain in inflammation

(b) Decreases vascular permeability

(c) Vasoconstriction

(d) Bronchodilatation

 

Answer : Unattempted

Explanation :

Bradykinin is a physiologically and pharmacologically active peptide of the kinin group of proteins,

consisting of nine amino acids.Activation of the kinin system results in the release of the vasoactive

nonapeptide bradykinin. Bradykinin increases vascular permeability and causes contraction of smooth

muscle, dilation of blood vessels, and pain when injected into the skin.Bradykinin is a potent

endothelium-dependent vasodilator, causes contraction of non-vascular smooth muscle,

increases vascular permeability and also is involved in the mechanism of pain. In some aspects, it

has similar actions to that of histamine, and like histamine it is released from venules rather than

arterioles.Bradykinin raises internal calcium levels in neocortical astrocytes causing them to release

glutamate.Bradykinin is also thought to be the cause of the dry cough in some patients on angiotensin

converting enzyme (ACE) inhibitor drugs. This refractory cough is a common cause for stopping ACE-

inhibitor therapy.Over activation of bradykinin is thought to play a role in a rare disease called

Hereditary Angioedema, also known as Hereditary Angio-Neurotic Edema.References1.Robbin's

Pathological Basis of Disease, (7th Ed.), Pg. 65]] 2. Review of Medical Physiology, Ganong, (22nd

Ed.), Pg. 601

Q 150.  Degeneration of the basement membrane is mediated by

(a) Oxidases

(b) Elastase

(c) Hydroxylases

(d) Metalloproteinase

Answer : Unattempted

Explanation :

The replacement of granulation tissue with a scar involves transitions in the composition of the ECM

(Extracellular Matrix).The balance between ECM synthesis and degradation results in remodeling of

the connective tissue framework - an impor-tant feature of both chronic inflammation and wound

repair.Degradation of collagen and other ECM proteins is achieved by a family of matrix

Metalloproteinases (MMPs), which are dependent on zinc ions for their activity.MMPs are secreted as

proenzymes and activated extracellularly. They are:– Interstitial collagenases that clave fibrillar

Page 59: Elsevier Jan

collagen types I, II and III.– Gelatinases degrade amorphous collagen as well as fibronection–

Stromelysins act on a variety of ECM components including proteoglycans, laminin, fibronectin and

amorphous col-lagens– ADAMS (disintegrin and metalloproternase-domain) are membrane bound

MMPs that release extracellular domains of cell surface proteins– Activated MMPs are rapidly inhibited

by tissue inhibitors of metalloproteinases (TIMPs)ReferencesRobbin's Pathological Basis of Disease,

(7th Ed.), Pg. 110

Q 151. Phenomenon by which certain genes are expressed in a parent-of-origin-specific manner is

called as

(a) Trisomy

(b) Mosaicism

(c) Genomic imprinting

(d) Aneuploidy

 

Answer : Unattempted

Explanation :

Explanation Genomic ImprintingWe all inherit two copies of each gene, carried on the homologous

maternal and paternal chromosomes.It is generally assumed that there is no functional difference

between the genes derived from the mother or the father. Recent studies have challenged this notion,

and there is accu mulating evidence that, at least with respect to some genes, there are functional

differences between the paternal gene and the maternal gene.These differences result from an

epigenetic process, called imprinting. In most cases, imprinting selectively inactivates either the

maternal or paternal allele. Thus, mater nal imprinting refers to transcriptional silencing of the mater nal

allele, whereas paternal imprinting implies that the paternal allele is inactivated.Imprinting occurs in the

ovum or the sperm, before fertilization, and then is stably transmit ted to all somatic cells through

mitosis. As is often the case in medicine, genomic imprinting is best illustrated by Prader-Willi syn

drome and Angelman syndrome.ReferencesRobbin's Pathological Basis of Disease, (7th Ed.), Pg. 186

Q 152. In granulocyte sarcoma the marker is

(a) CD-38

(b) CD-33

(c) CD-117

(d) CD-137

Answer : Unattempted

Explanation :

Granulocytic sarcoma:Granulocyte sarcomas (chloroma) or myeloblastomas are discrete tumors of

leukemic cells that form in skin and soft tis-sues, periostium and bone, lymphnodes, mediastinum, GIT,

pleura, gonads, uterus, CNS and other sites.The develop in patients with AML or accelerated phase of

CML and occasionally, may be the first manifestation of AML, preceeding the onset in the marrow and

blood by months or years.Granulocyte sarcomas can been misdiagnosed as malignant lymphomas

Page 60: Elsevier Jan

because of similarity in histopathology in biopsy specimens from soft tissues.Traweek et al described

the use of commercially available panel of monoclomal antibodies, against myeloperoxidase, CD68,

CD43 and CD20, to accurately diagnose chloroma via immunohistochemitry and differentiate it from

lymphoma. Nowadays, immunohistochemical staining using monoclonal antibodies against CD34 and

CD117 would be the mainstay of diagnosis.ReferencesWilliamus Hematology, (7th Ed), Pg. 1152

Q 153. A diabetic patient is undergoing dialysis. Aspiration done around the knee joint would show

(a) β-2 microglobulin

(b) AA

(c) AL

(d) Lactoferrin

 

Answer : Unattempted

Explanation :

Diabetic patients may develop chronic renal failure for which they may have to undergo dialysis.Patient

on long-term hemodialysis for renal failure develop amyloidosis owing to deposition of β-2

microglobulin.This protein is present in high concentration in the serum of patients with renal failure and

is retained in circulation because it cannot be filtered through the cuprophane dialysis membranes.In

some series, as many as 60% to 80% of the patients on long-term dialysis developed amyloid deposits

in the synovium, joints, and tendon sheaths.ReferencesRobbin's Pathological Basis of Disease, (7th

Ed.), Pg. 261

Q 154. Which of the following oncogene is a growth factor?

(a) Fos

(b) Jun

(c) Myc

(d) Sis

Answer : Unattempted

Explanation :

Many cancer cells develop growth self-sufficiency by acquiring the ability to synthesize the same growth

factors to which they are responsive.The protooncogenesis, which encodes the β chain of platelet-

derived growth factor (PDGF), is overproduced in many tumors, especially low-grade astrocytomas

and osteosarcomas.Furthermore, it appears that the same tumors also express receptors for PDGF

and are hence responsive to autocrine stimu-lation.Although an autocrine loop is considered to be an

important element in the pathogenesis of several tumors, in most instances the growth factor gene itself

is not altered or mutated.More commonly, products of other oncogenes such as RAS (that lie along

many signal transduction pathways) cause over-expression of growth factor genes, thus forcing the

cells to secrete large amounts of growth factors, such as transforming growth factor-α.Myc, Fos, Myb

and Jun oncogenes are nuclear regulatory proteins of these Myc is most commonly involved in human

tumors.ReferencesRobbin's Pathological Basis of Disease, (7th Ed.), Pg. 293

Page 61: Elsevier Jan

Q 155. Which of the following is the most specific marker for Hodgkin's lymphoma?

(a) CD 15 and CD 68

(b) CD 15 and CD 30

(c) CD 15 and CD 45

(d) CD 30 and CD 68

 

Answer : Unattempted

Explanation :

The term “Hodgkin's Lymphoma (HL) previously known asHodgkin disease, encompasses a group of

lymphoid neoplas-mas that differ from NHL in several respectsIt is characterized morphologically by the

presence of distinctive neoplastic giant cells called Reed-Sternberg cells that induce the accumulation

of reactive lymphocytes, histiocytes (macrophages) and granulocytes.The neoplastic Reed-Sternberg

cells typically make up a minor fraction (1% to 5%) of the total tumor cell mass, making HL more difficult

to study than typical NHS.The WHO classification recognizes 5 subtypes of HL:– Nodular sclerosis–

Mixed cellularity– Lymphocyte rich– Lymphocyte depletion– Lymphocyte predominanceIn the first four

subtypes the Reed-Sternberg cells have a similar immunophenotype (CD 15 +ve, CD 30 +ve, CD45 -

ve); as a result these are often lumped together as classical forms of HLIn lymphocyte predominant HL,

the Reed-Sternberg cells have a characteristic B-cell immunophenotype (CD20 +ve, CD15 -ve, CD 30 -

ve) distinct from that of the classical HL subtypesReferencesRobbin's Pathological Basis of Disease,

(7th Ed.), Pg. 686–688

Q 156. All of the following are antigen presenting cells EXCEPT

(a) Langerhans cells

(b) Dendritic cells

(c) T cell

(d) Activated B cell

Answer : Unattempted

Explanation :

Antigen presenting cells (APC) are cells that can process antigen, break it into peptides, and present it

in conjunction with class II MHC molecules on the surface where it may interact with appropriate T-cell

receptor.Professional APCs include:Dendritic cells (interstitial and follicular)MacrophagesB

cells.Nonprofessional APCs (function as APCs for only brief periods).Thymic epithelial cellsVascular

endothelial cells.Among the three major APCs, dendritic cells are the only ones that can present

antigen via class IMHC molecule and class II MHC molecules.References1. Immunology Guide Book,

2004, Pg. 267]] 2. Robbin's Pathological Basis of Disease, (7th Ed.), Pg. 199

Q 157. Pick's bodies stain positive for all of the following except

(a) Ubiquitin

(b) Alpha-synnuclein

(c) Tau

Page 62: Elsevier Jan

(d) Alz-50

 

Answer : Unattempted

Explanation :

Pick disease (lobar atrophy) is a rare distinct progressive dementia characterised clinically by early

onset of behavioral changes together with alterations in personality (frontal lobe signs) and language

disturbances (temporal lobe signs)The brain invariably shows a pronounced, frequently asymmetric

atrophy of frontal and temporal lobes with conspicious sparing of the posterior two thirds of the superior

temporal gyrus and only rare involvement of either the parietal or the occipital lobe.The atrophy can be

severe reducing the gyri to a thin wafer (“knife-edge appearance). This pattern of lobar atrophy is often

prominent enough to distinguish Pick disease from Alzheimer disease on macroscopic examinationIn

addition to the localised cortical atrophy, there may also be bilateral atrophy of caudate nucleus and

putamen.On microscopic examination, neuronal loss is most severe in the outer three layers of the

cortex. Some of the surviving neurons show a characteristic swelling (Pick cells) or contain Pick bodies

which are cytoplasmic, round to oval filamentous inclusions that stain strongly with silver-

methods.Ultrastructurally these are composed of straight filaments, vesiculated ER and powered helical

filaments that are immunocytochemically similar to these found in Al;zheimer disease and contain 3

repeat tau, ubiquitin and Alz-50α-synnuclein is a component of Lewy bodies. Lewy bodies are

composed of fine filaments, densely packed in core but loose at the rim. These filaments are composed

of α-synnuclein, as was realized after the gene for this protein was linked to familial PD; neurofilament

antigens, Parkin and ubiquitin are also present in the Lewy bodies.References1. Robbin's Pathological

Basis of Disease, (7th Ed.), Pg. 1390,1391, 1392]] 2. Harrison's Principles of Internal Medicine, (16th

Ed.), Pg. 2402

Q 158. Worst prognosis is seen in which type of FSGN?

(a) Tip lesion variant

(b) Collapsing variant

(c) NOF

(d) Perihilar

Answer : Unattempted

Explanation :

Columbia classification of the morphological variants of FSGS.Collapsing type– it has highest frequency

to pass to end-stage renal disease.Glomerular tip lesion– least chances to pass to end-stage

disease.Cellular typePerihilar typeNot otherwise specified (NOF)–MC type

Q 159. In mitral valve prolapse, the histological finding is

(a) Hyalinization of the valve

(b) Fibrinoid necrosis

(c) Myxomatous degeneration of the valve

(d) None of the above

Page 63: Elsevier Jan

 

Answer : Unattempted

Explanation :

Mitral value prolapse is valvular abnormality where one or both mitral leaflets are “floppy”

and prolapse, or balloon back into the left atrium during systole.It is pathologically known as

“myxomatous degeneration” of the mitral value.The affected leaflets are often enlarged, redundant,

thick, and rubbery.Frequently involved, the tendinous cords are elongated, thinned and occasionally

ruptured.Annular dilation is characteristics, a finding that is rare in other causes of mitral

insufficiency.Concomitant involvement of the tricuspid value is present in 20% to 40% of cases, and the

aortic or pulmonary value (or both) may also be affected.Commissural fusion that typifies rheumatic

heart disease is absent.Historically, there is attenuation of the fibrosa layer of the value accompanied

by focally marked thickening of the spongiosa layer with deposition of mucoid (myxomatous) material.

The collagenous structure of the cords is attenuated.ReferencesRobbin's Pathological Basis of

Disease, (7th Ed.), Pg. 591

Q 160. Histopathological finding in Whipple's disease is

(a) Histiocytic infiltration in lamina propria

(b) Nongranulomatous lesion

(c) Proliferation

(d) Eosinophilic infiltration in mucosa

Answer : Unattempted

Explanation :

Whipple disease (intestinal lipodystrophy) is a rare disease caused by the bacterium Tropheryma

whippelii. It is a systemic condition that may involve any organ of the body, but principally affects the

intestine, central nervous system and joints.The causal organism T. Whipple is a gram-positive

actinomycete, named on the basis of molecular phylogenetic analysis.The histopathological hallmark

of Whipple disease is a small-intestinal mucosa laden with distended macrophages in the

lamina propria. The macrophages contain PAS-positive, diastase-resistant granules (which are

lysosomes stuffed with partially digested micro-organisms) and rod-shaped bacilli on electron

microscopy.Expansion of the villi by the dense infiltrate of macrophages imparts a shaggy gross

appearance to the intestinal mucosal surfaces. Edema of the mucosa thickens the intestinal

wallAccompanying these changes is involvement of mesenture lymph nodes by the same process and

lymphatic dilation, sug-gesting lymphatic obstruction. The lymphatic blockage is believed to be

responsible for lipid deposition in the villi thus the original impression of intestinal

lipodystrophy.ReferencesRobbin's Pathological Basis of Disease, (7th Ed.), Pg. 844

Q 161. HMB-45 is immunohistochemistry marker for

(a) Melanoma

(b) Schwannoma

(c) Neurofibroma

(d) Rhabdomyosarcoma

Page 64: Elsevier Jan

 

Answer : Unattempted

Explanation :

HMB-45 is immunohistochemical marker for melanomaHBN-45 is a monoclonal antibody originally,

obtained from extract of malignant melanoma, which identifies an oncofetal glycoconjugate associated

with immature melanosomes and probably related to the tyrosinase engymatic systemOriginally though

to be specific for activated/neoplastic melanocytes, it is now known to be expressed by other neural

crest derived tumors, angiomyolipomas of the kidney and other sites, other components of tuberosus

sclerosis complex (“PEComas”) and occassional carcinomas and leneral other neoplasma.Other

melanocytic marker frequently used is S–100ReferencesAckerman's Surgical Pathology, (9th Ed), Pg.

54

Q 162. Biondi ring tangles are seen in

(a) Lateral ventricle

(b) Cerebral cortex

(c) Lining of the choroid plexus

(d) Cerebellum

Answer : Unattempted

Explanation :

Biondi ring tangles are intracellular inclusions found in epithelial cells of choroid plexus cells of

Alzheimers disease/aging brains.They are homogenous globules covered with a thin layer of amyloid

fibrilsBy electron microscopy these tangles are made of tightly packed bundles of long filaments with

diameter around 10 nm that are morphologically distinct.Pathological changes in choroid plexus in

Alzheimer's diseaseEpithelial atrophy (↓ cell size by 1/3)↑ Biondi ring tanglesLipofuchsin

vacuolesBasement membrane thickening (3-fold ↑)↑ Stromal fibrosisIgG and IgM

depositionsReferences1. Robbin's Pathological Basis of Disease, (7th Ed.), Pg. 1386–88]] 2. Illustrated

Histopathology of Nervous System, (2nd Ed.), Pg. 235

Q 163. Which of the following protein defects does not cause the hereditary spherocytosis?

(a) Ankyrin

(b) Palladin

(c) Glycophorin C

(d) Anion transport protein

 Answer : Unattempted

Explanation :

HS is a disorder characterized by intrinsic defect in red cell membrane, leading to loss of membrane

and hence decreased surface area to volume ratio and consequently spherocytosis.Molecular

abnormality involves defect in proteins that are responsible for anchoring the lipid bilayer to the

underlying cytoskeletal network.Defective proteins may be:Ankyrin– defective in 50% patientsBand 3

(anion transport channel)– defective in 25% patientsSpectrin– defective in another 25%

patientsPallidin (band 4.2) – rare defectReferences1. Harrison's Principles of Internal Medicine, (16th

Page 65: Elsevier Jan

Ed.), Pg. 608]] 2. Robbin's Pathological Basis of Disease, (7th Ed.), Pg. 625

Q 164. Acinic cell carcinomas of the salivary gland is most commonly seen in

(a) Parotid salivary gland

(b) Minor salivary glands

(c) Submandibular salivary gland

(d) Sublingual salivary gland

Answer : Unattempted

Explanation :

Acnic cell tumors:Are composed of cells resembling the normal serous acinar cells of salivary

glands,Are relatively uncommon and constitute 2% to 3% of salivary gland tumors,most arise in parotid

glands; the small remainders arise in the submandibular glands,rarely involve minor salivary glands,

which normally have only a scant number of serous cells, andsometimes can be bilateral and

multicentric.ReferencesRobbin's Pathological Basis of Disease, (7th Ed.), Pg. 794.

Q 165. In a mother with the first baby is suffering from 21-hydroxylase deficiency; when would you

administer dexamethasone in next pregnancy?

(a) Started before pregnancy in planned conception

(b) As soon as diagnosis of pregnancy is confirmed

(c) Give after determining the sex of the baby

(d) After CVS done and confirmed

 

Answer : Unattempted

Explanation :

Prenatal diagnosis of congenital adrenal hyperplasia can be done by estimation of 17 ketosteroid,

pregnanetriol and 17-OHP in aminotic fluid or by HLA typing of aminotic cells. It is also possible by

detecting the mutations of the CAH gene by molecular genetic studies in chronic villous biopsy done as

early as 9th week of gestation.Now, prenatal treatment is given to prevent sexual ambiguity in newborn

girls by administering dexamethasone 0.5 mg slowly to pregnant mothers starting from 5th week of

gestation (OP Ghai). If started by 6 weeks of gestation, this amelio-rates the virilization of the external

genitalia in affected females (Nelson).References1 Synopsis, Pg. 952 Ghai's Essentials of Pediatrics,

Pg. 4903 Textbook of Pediatrics, Nelson, Pg. 1912

Q 166. MC inherited malignant tumor in childhood is

(a) Leukemia

(b) Retinoblastoma

(c) Neuroblastoma

(d) Wilm's tumor

Answer : Unattempted

Page 66: Elsevier Jan

Explanation :

Inherited cancer syndromes include several well-defined cancers in which inheritance of a single

mutant gene greatly increases the risk of developing a tumor.The predisposition of these tumors shows

an autosomal dominant pattern of inheritance.Childhood retinoblastoma is the most striking example in

this category.Approximately 40% of retinoblastoma are inherited.Carriers of a mutant of the RB tumor

suppressor gene have a 10,000 fold increased risk of developing retinoblastoma usually

bilateral.Retinoblastoma tumor predisposition is inherited as an autosomal dominant pattern with a high

penetrance rate (80–100%) of individuals with the mutated gene developing retinoblastoma.Features

that characterize inherited cancer syndrome are as follows:In each syndrome tumors involve specific

sites and tissues.Tumors within this group are often associated with a specific marker phenotype.Both

incomplete penetrance and variable expressively are noted.References1 Robbin's Basic Pathology,

(7th Ed.), Pg. 2842 Ghai's Essentials of Pediatrics, Pg. 577

Q 167. A 6-year-old child has urinary retention and constipation, the diagnosis is

(a) Malignant sacral teratoma

(b) Rectal duplication of cyst

(c) Pelvic neuroblastoma

(d) Anterior meningomyelocele

 

Answer : Unattempted

Explanation :

Sacrococcygeal teratoma: The tumor, which arises between the sacrum and the rectum, is firmly

attached to the last seg ment of the sacrum.At the time of birth, some of these tumors are huge, and in

20% of cases the infant is stillborn.The tumor tends to be large, but it can be small enough to pass

unnoticed until it enlarges or a complication ensues.It is this (large) variety that is prone to become

malignant, usually at about 10 months of age.Treatment: removal should be soon after birth.Delay is

liable to result in fatal ulceration, infection, rectal or urinary obstruction, or malignant change.Pelvic

neuroblastoma:Apart from pressure effect neuroblastoma should have other features also. About 90%

of neuroblastoma regardless of location, produce catecholamines, which are important diagnostic

features.Myelomeningocele:Occurs most commonly in the lumbo sacral region and presents with

flaccid paralysis of the lower limbs, absent deep tendon reflex, lack of response to touch and pain and a

high incidence of postural abnormalities of the lower limbs (club feed and subluxation of hips).Constant

urinary dripping and a relaxed and sphincter may be evident. (Not retension and

constipation).Myelomeningocele is always associated with neurological deficits and urinary problems

develop in 90% of cases. But the question talks about urinary retention and constipation, hence the

Answer will be (a).References1 Bailey and Love's Short Practice of Surgery, (24th Ed.), Pg.

12482Robbin's Basic Pathology, (7th Ed.), Pg. 5023 Textbook of Pediatrics, Pg. 1985

Q 168. Best indicator of long-term nutritional status is

(a) Mid arm circumference

(b) Height for age

Page 67: Elsevier Jan

(c) Weight for age

(d) Weight for height

Answer : Unattempted

Explanation :

Measurement of the velocity of growth or increment in a unit of time is better for an early identifica tion

of factors affecting growth. Growth parameters includes weight, height, head circumference, and chest

circumference, and so on.Weight is a sensitive measure of the nutritional status.Decreased weight for

height is Wasting = signifying acute onset malnutrition.Decreased height for age is Stunting = signifying

chronic course of malnutrition.References1 Synopsis, Pg. 132 Ghai's Essential Pediatrics, (6th Ed.), Pg.

4–53 Textbook of Social and Preventive Medicine, Park, (18th Ed.), Pg. 474

Q 169. Commonest tumor in children is

(a) Leukemia

(b) Lymphoma

(c) Wilms' tumor

(d) Neuroblastoma

 

Answer : Unattempted

Explanation :

Leukemia is the most common form of child hood malignancy. It accounts for 30% of all childhood

causes. This is followed by brain tumor (19.1%), lymphoma (12.3%), neuroblastoma (8.1%), Wilm's

tumor (6.5%), soft tissue sar coma (6.3%), tumors of the bone (4.8%), and retinoblastoma

(2.3%).Lymphomas are the third most common group of malignancies found in

children.Neuroblastomais the most common intra-abdominal solid tumor that occurs in

childhood.Around 50% occurs within 2 years of life. Most frequently diagnosed neoplasm occurs in

infants.Wilm's tumor is the 2nd most common malignant abdominal tumor in childhood. Around 80%

pres ents in less than 5 years of age.References1. Synopsis, Pg. 492. Ghai's Essential Pediatrics, (6th

Ed.), Pg. 5613. Textbook of Pediatrics, Nelson, Pg. 1709, 1711

Q 170. Most common cause of seizures in neonates is

(a) Hypoxic

(b) Metabolic

(c) Trauma

(d) Fever

Answer : Unattempted

Explanation :

References1. Synposis, Pg. 382. Manual of Neonatal Care, Cloherty, (5th Ed.), Pg, 512

Q 171. Tumor associated with heterozygosity is

Page 68: Elsevier Jan

(a) Hypoxic

(b) Metabolic

(c) Trauma

(d) Fever

 

Answer : Unattempted

Explanation :

Both normal alleles of the RB locus must be inactivated (two hits) for the development of

retinoblastoma.A child carrying an inherited mutant Rb allele in all somatic cells is perfectly normal

(except for increased risk of developing cancer). Because such a child is heterozygous at the Rb locus;

it implies that heterozygosity for the Rb gene does not affect cell behavior.Cancer develops when with

cell becomes homozygous for the mutant allele or put another way, when the cell loses heterozygosity

for the normal Rb gene (a condition known as LOH, for loss of heterozygosity).Because the Rb gene is

associated with cancer, when both normal copies are lost, it is sometimes referred to as a “Recessive

Cancer Gene”.ReferencesRobbin's Basic Pathology, (7th Ed.), Pg. 299

Q 172. A baby has recently developed mouthing but has not developed stranger anxiety, likes and

dislikes for food. Most appropriate age of this baby is

(a) 3 months

(b) 5 months

(c) 7 months

(d) 9 months

Answer : Unattempted

Explanation :

The mouthing occurs at 4 months; stranger's anxiety at 7–8 months; likes and dislikes for food at 4–5

months. So child age is less than 7–8 months but more than 4–5 months.References1. Synopsis, Pg, 4,

52. Textbook of Pediatrics, Nelson, (17th Ed.), Pg. 343. Ghai's Essential Pediatrics, (6th Ed), Pg. 46–7

Q 173. Out of the following which milestones develops first?

(a) Mirror play

(b) Crawling

(c) Creeping

(d) Pincer grasp

 

Answer : Unattempted

Explanation :

Mirror play occurs at 6 months; crawling at 9 months; creeping at 10 months; prince grasp at 9 months;

unassisted pincer grasp at 12 months.References1. Synopsis, Pg. 4, 52. Ghai's Essential Pediatrics,

(6th Ed.), Pg. 44, 46

Page 69: Elsevier Jan

Q 174. A 6 years old child has an IQ of 50. Which of the following tasks should the child be able to

perform?

(a) Identify colors

(b) Read a sentence

(c) Ride a bicycle

(d) Copy a triangle

Answer : Unattempted

Explanation :

IQ = mental age/chronological age × 100.Mental age = IQ × chronological age/100 = 50 × 6/100 = 3

years.Child can identify color at the age of 2½ years (names one color at the age of 2½ years; names 2

color at 3 years); reads a sentence at 5 years; rides bicycle at age 5 years and after; copies a triangle

at 5 years. Rides tricycle at 3 years. Remem ber, in choices it is bicycle not

tricycle.References1 Textbook of Social and Preventive Medicine, Park, (18th Ed.), Pg. 4982 Child

Development, Elingworth, (9th Ed.), Pg. 124

Q 175. Most common mode of treatment of a 1-year-old child with asthma is

(a) Inhaled short acting beta-2 agonist

(b) Oral short-acting theophylline

(c) Oral ketotifen

(d) Leukotriene agonist

 

Answer : Unattempted

Explanation :

Treatment of acute moderate and severe asthma:Inhaled B2 agonist: Repeated every 20 minutes for 1

hour with O2.Oral dose prednisolone. Then assess for improvement.Treatment of mild acute

asthma:B2agonist nebulisation or metered dose inhaler (MDI) with spacer with or without facemask.

Then assess for improvement.In both the above mentioned conditions B2 against inhalation is used for

the initial treatment.*MDI: Metered Dose inhalers require co-ordination so it cannot be used before 5–6

years.*All other drugs mentioned in choices are used for maintenance therapy along with

B2 against.References1.Synopsis, Pg. 1132. Ghai's Essential Pediatrics, (6th Ed.), Pg. 356

Q 176. Component of pentalogy of Fallot is

(a) Atrial septal defect (ASD)

(b) Patent ductus arteriosus (PDA)

(c) Coarctation of Aorta (CoA)

(d) Left ventricular hypertrophy (LVH)

Answer : Unattempted

Explanation :

Pentalogy of fallots includes four components of tetralogy of fallot and ASD/patent Foramen ovale, that

Page 70: Elsevier Jan

is, ASD or patent foramen ovale, VSD, RVH, PS and over-riding of aorta.

Q 177. A newborn has congenital heart failure, which is not improving on treatment. He has bulging

anterior fontanelles with a bruit on auscultation. On trans-fontanelle USG a hypoechoic midline

mass is seen with dilated lateral ventricles. What is the most likely diagnosis in this patient?

(a) Vein of Galen malformation

(b) Arachnoid cysts

(c) Medulloblastoma

(d) Encephalocele

 

Answer : Unattempted

Explanation :

An arteriovenous malformation of the vein of galen during infancy can cause: high cardiac output

congestive cardiac failure secondary to shunting of large volumes of blood or progressive

hydrocephalus and increased intracranial pressure, secondary to obstruction of C.S.F. pathways.They

rarely bleed. The presence of focal neurological deficits depends on the location of the malformation

and may include hemiparesis, sensory disturbances, and oculomotor palsies.A cranial bruit is invariably

present.Diagnosis: On USG-Hypoechoic midline mass in seen. On CT (Con trast enhanced study):

visualization of malformation. On MRA (magnetic resonance angiography) → definitive diagnosis can

be made.Treatment: Microsurgical excision; Embolization of vein.* The other three choices in question

does not present with cranial bruit.References1. Pediatric Neurology, (4th Ed.), Fenichel, Pg. 360,

1052.Textbook of Pediatrics, (17th Ed.), Nelson, Pg. 2037

Q 178. Premature baby of 34 weeks was delivered. Baby developed bullous lesion on the skin and X-

ray shows periostitis. Next investigation should be

(a) VDRL for mother and baby

(b) ELISA for HIV

(c) PCR for TB

(d) Hepatitis surface antigen for mother

Answer : Unattempted

Explanation :

This baby is a case of congenital syphilis. So investigation to diagnose is VDRL for mother and

baby.References1. Synopsis, Pg. 302. Manual of Neonatal Care, Cloherty, (5th Ed.), Pg. 322

Q 179. In an autosomal recessive (AR) disorder, one parent is normal and the other is carrier and the

child is also affected. Best explanation is

(a) Germline mosaicism

(b) Genomic imprinting

(c) Penetration

Page 71: Elsevier Jan

(d) Uniparental disomy

 

Answer : Unattempted

Explanation :

Uniparental disomy:It occurs when both chromosomes of a pair of chro mosomes, present in a person

with a normal number of chromo somes have been inherited from only one parent.Uniparental

isodisomy:In this both chromosomes in a pair are identical; consequently, the genes on both

chromosomes are also identical.When a parent is a carrier of AR disorder and the offspring of a carrier

parent has uniparental disomy with isodisomy for a chromosome that carries an abnormal gene, the

abnormal gene will be present in two copies and the phenotype will be that of the AR disorder. In others

worlds, the child has an autosomal recessive disorder when only one parent is actually a carrier of that

recessive disorder.* The AR disorders spinal muscular atrophy, cystic fibrosis, cartilage-hair hypoplasia,

α and β thalassemias, and blooms syndrome have occurred because of uniparental disomy.Germ line

mosaicismThis refers to the presence of mosaicism in the germs cells found in the gonads.This is

suspected when there is more than one affected offspring with the some genetic abnormality (usually

inherited as a chromosomal or autosomal dominant disorder) with phenotypically normal parents.There

is an increased risk of recurrence of an affected child.Penetration:Penetrance refers to the proportion of

individuals with a mutant genotype that expresses the phenotype.If all carriers of a mutant express the

phenotype, penetrance is complete.Some individuals do not have any features of the phenotype it is

said to be incomplete or reduced penetrance.Dominant conditions with incomplete penetrance are

characterized by skipping of generations with unaffected carriers transmit ting the mutant gene. For

e.g., hypertrophic obstructive cardiomyopathy, caused by mutations in the myosin binding protein C

gene is a dominant disorder with clinical features in only a subset of patients who carries the

mutation.Patient who have the mutation but no evidence of the disease can still transmit the disorder to

subsequent generation.References1. Synopsis, Pg. 622. Textbook of Pediatrics, Nelson, (17th Ed.), Pg.

3893.Harrison's Principles of Internal Medicine, (16th Ed.), Pg. 371, 375, 385

Q 180. All of the following can establish a diagnosis of congenital CMV infection in a neonate EXCEPT

(a) Urine culture of CMV

(b) IgG CMV antibodies in blood

(c) Intra-nuclear inclusion bodies in hepatocytes

(d) CMV viral DNA in blood by polymerase chain reaction

Answer : Unattempted

Explanation :

The definitive method for diagnosis of con genital CMV infection is virus isolation (spin enhanced cul

ture or shell vial) or PCR, which should be performed at or shortly after birth. Urine and Saliva are best

specimens for culture.Infants with congenital CMV infection may excrete CMV in the urine for several

years.An IgG antibody test is of little diagnostic valve because a positive result also reflects maternal

antibodies, although a negative result excludes the diagnosis of congenital CMV infection.In general,

IgM tests lack sensitivity and specificity and are unreliable for diagno sis of congenital CMV infection.

Antibodies to CMV anti gen: pp 65 antigen can be used for diagnosis.Cytomegalic cells are found in a

wide variety of organs including salivary gland, lung, liver, kidney, intestine, pancreas, adrenal gland,

Page 72: Elsevier Jan

and the CNS.References1 Synopsis, Pg. 302 Manual of Neonatal Care, Cloherty, (5th Ed.), Pg.

2573Textbook of Pediatrics, Nelson, (17th Ed.), Pg. 10684 Harrison's Principles of Internal Medicine,

(16th Ed.), Pg. 1050–1

Q 181. Which of the following is false about the side effect of tetracycline?

(a) Can cause pseudotumor cerebri

(b) Not teratogenic if used during pregnancy

(c) Discoloration of teeth may occur

(d) Superinfection can occur

 

Answer : Unattempted

Explanation :

Adverse effects of tetracyclineIrritative effects: Tetracyclines can cause epigastric pain, nausea,

vomiting, and diarrhea by their irritant property.Dose-related toxicityLiver damage: Fatty infiltration of

liver and jaundice occurs occasionally.Kidney damage: A reversible Fanconi

syndromePhototoxicityTeeth and bones: Given during late pregnancy or childhood, tetracyclines can

cause temporary suppression of bone growth. The ultimate effect on stature is mostly insignificant, but

deformities and reduction in height are a possibility with prolonged use.Antianabolic effectIncreased

intracranial pressureDiabetes insipidusVestibular toxicityHypersensitivity Superinfection-

Tetracyclines are the MC antibiotics responsible for superinfection. Intestinal infection with candida

albicans is most prominentReferencesEssentials of Medical Pharmacology, Tripathi KD, (6th Ed.), Pg.

107-110

Q 182. Which of the following is an absolutely contradiction of ipratropium bromide?

(a) Asthma

(b) Hypertension

(c) Peptic ulcer

(d) Urinary retention

Answer : Unattempted

Explanation :

Anticholinergic drugs have relaxant action on ureter and urinary bladder leading to urinary retention in

older males with prostatic hypertrophy. However, the same can be beneficial in neurogenic bladder

enuresis.Relaxation of biliary tract is less marked and effects on uterus are also

minimal.ReferencesEssentials of Medical Pharmacology, Tripathi KD, (6th Ed.), Pg. 110.

Q 183. Flumazenil is

(a) Agonist of diazepam

(b) Diazepam antagonist

(c) SSRI

Page 73: Elsevier Jan

(d) Anticonvulsant

 

Answer : Unattempted

Explanation :

Antagonism: When one drug decreases or inhibits the action of another, they are antagonistic.

Usually, in an antagonistic pain, one drug is inactive as such but decreases the effect of the

other.Flumazenil: It is an imidazobenzodiazepine, and a specific and competitive antagonist of

benzodiazepines (BZD) at BZD receptors.Flumazenil: It is a BZD analog, which has little intrinsic

activity (practically no effect on normal subjects) but competes with BZD receptor and reverses their

depressant or stimulant effects, respectively.It abolishes the hypnogenic, psychomotor, cognitive, and

electroencephalogram (EEG) effects of BZDs. At higher doses, it has some weak BZD agonist-like as

well as an inverse agonist-like activity, but these are of no clinical significance.On IV injection, action of

flumazenil starts in seconds and lasts for 1-2 hours; elimination t1/2 is 1 hour due to rapid

metabolism.UsesTo reverse BZD anesthesia:It allows early discharge of patients after diagnostic

procedures and facilitates postanesthetic management.BZD overdose:Majority of patients of BZD

overdose require only supportive measures, like patent airway, maintenance of BP, cardiac, and renal

function. In addition, flumazenil 0.2 mg/minute may be injected IV till the patient regains

consciousness.Adverse effectsFlumazenil is safe and well tolerated. Agitation, discomfort, tearfulness,

anxiety, coldness, and withdrawal seizures are the occasional side effects.References1. Essentials of

Medical Pharmacology, Tripathi KD, (6th Ed.), Pg. 399-400.]] 2. Clinical Anesthesiology, (4th Ed.), Pg.

286.]] 3. Basic & Clinical Pharmacology, Katzung, (10th Ed.), Pg. 354

Q 184. Ergotamine is contraindicated in all of the following EXCEPT

(a) Vascular Headache

(b) Pregnancy

(c) Ischemic heart disease

(d) Patient on Erythromycin

Answer : Unattempted

Explanation :

ErgotamineIt acts as a partial agonist and antagonist at a adrenergic and all subtypes of 5HT1 and

5HT2receptors, but does not interact with 5HT3 or dopamine receptorsProduces sustained

vasoconstriction, visceral smooth muscle contraction, vasomotor center depression and antagonizes

the action of NA and 5HT on smooth muscles.The compound also has the properties of serotonin

antagonism.IndicationIndicated as therapy to abort or prevent vascular headache, e.g., migraine,

migraine variants or so-called “histaminic cephalalgia”.Adverse effectsNausea, vomiting, abdominal

pain, muscle cramps, weakness, paresthesias, coronary and other vascular spasm, chest pain are the

frequent side effects.These drugs are contraindicated in presence of sepsis, ischemic heart disease,

peripheral vascular disease, hypertension, pregnancy, liver and kidney disease.Drug InteractionsCo-

administration of ergotamine with potent CYP 3A4 inhibitors (ritonavir, nelfinavir, indinavir,

erythromycin, clarithromycin, and troleandomycin) has been associated with acute ergot toxicity

(ergotism) characterized by vasospasm and ischemia of the extremities with some cases resulting in

amputation.There have been rare reports of cerebral ischemia in patients on protease inhibitor

Page 74: Elsevier Jan

therapyErgot Alkaloids include:Ergotamine/dihydroergotamineErgonovine

(Ergometrine)DihydroergotoxineBromocriptineLSDMethysergideReferences1 Essentials of Medical

Pharmacology, KD Tripathi, (6th Ed.), Pg. 1682 Basic & Clinical Pharmacology, Katzung, (10th Ed.),

Pg. 274

Q 185. Which of the following drugs is not used in erectile dysfunction?

(a) Phenylephrine

(b) Apomorphine

(c) Alprostadil

(d) PGE1 analogs

 

Answer : Unattempted

Explanation :

Phenylephrine is related structurally to adrenaline and is a vasoconstrictor. Rest of the drugs is used in

erectile dysfunction. Apomorphine, a D1 stimulant is an investigational drug. Alprostadil, a synthetic

PGE1 analog was introduced in 1986 by Daiken. It is particularly useful in impotence induced by spinal

cord injury. Drugs of choice for impotence are phosphodiesterase inhibitors. There is no compelling

evidence that one phosphodiesterase inhibitor is better than the other one.

Q 186. Leucovorin is used as rescue therapy for patients who are on one of the following drug

therapies

(a) Asparagine

(b) Methotrexate

(c) 6-mercaptopurine

(d) Cyclophosphamide

Answer : Unattempted

Explanation :

Methotrexate toxicity: Folinic acid (Leucovorin, citrovorum factor, 5-formyl-THF) is an active coenzyme

form which does not need to be reduced by DHFRase before it can act.Methotrexate is a DHFRase

inhibitor; its toxicity is not counteracted by folic acid, but antagonized by folinic acid.Folinic acid is

expensive and not needed for the correction of simple folate deficiency for which folic acid is good

enough.References1.Essentials of Medical Pharmacology, Tripathi KD, (6th Ed.), Pg. 592.2. Lippincot's

Pharmacology, (3rd Ed.), Pg. 458.

Q 187. Deafness causing drug is

(a) Alcohol

(b) Valproic acid

(c) Chloroquine

(d) Warfarin

Page 75: Elsevier Jan

 

Answer : Unattempted

Explanation :

Both alcohol and chloroquine are mentioned in the list given, but if we have to choose one option then

we will go with the choice (c) chloroquine.Others are as follows:Antibiotics

(Aminoglycoside)StreptomycinGentamycinTobramycinNeomycinDiureticsFrusemideEthacrynic

acidAntimalarialsQuinineChloroquineCytotoxic

drugs

N2mustardCisplatinCarboplatinAnalgesicsSalicylatesIndomethacinIbuprofenChemicalsAlcoholTobacco

CO poisoningMiscellaneousErythromycinAmpicillinPropranololDeferoxamineReferenceDisease of Ear,

Nose, and Throat, Dhingra, (3rd Ed.), Pg. 44

Q 188. Following drug is used for subungual hyperkeratosis of nail

(a) Oxytetracycline

(b) Silver sulfadiazine

(c) Clofazimine

(d) Terbinafine

Answer : Unattempted

Explanation :

Collection of friable debris under the nail is called subungual hyperkeratosis caused by tinea

unguium.The treatment is as follows:FingernailsTerbinafine (8 weeks)Itraconazole (2

pulses)Griseofulvin (24 weeks)Toe nailsTerbinafine (12-16 weeks)Itraconazole (3-4 pulses)Griseofulvin

(36 weeks)ReferenceIllustrated Synopsis of Dermatology and Sexually Transmitted Disease, Neena

Khanna, (1st Ed.), Pg. 242, 244

Q 189. Which of the following is not associated with beta-2 agonist treatment?

(a) Hyperkalemia

(b) Hyperglycemia

(c) Detrusor relaxation

(d) Relaxation of gut and bronchial muscles

 

Answer : Unattempted

Explanation :

Stimulating beta-2 adrenergic receptors opens up potassium channels and this could lead to increase in

movement of potassium from plasma to muscles.

Q 190. Which of the following is not a nonselective beta-blockers with additional actions?

(a) Carvedilol

(b) Betaxolol

(c) Carteolol

Page 76: Elsevier Jan

(d) Labetalol

Answer : Unattempted

Explanation :

Betaxolol is a cardioselective beta-blocker with no additional alpha blocking action. The drug is used for

patients with glaucoma with bronchial asthma or hypertension. It has greater affinity for beta-1 than

metoprolol. It was approved as a 0.5% ocular solution for use in glaucoma in 1985. Later, another

formulation appeared in 1995 (0.25%). Though a long acting drug, it is almost completely devoid of

systemic effects upon topical use in eye.

Q 191. Which of the following does not have receptors which are transcription factors?

(a) Insulin

(b) Estrogen

(c) Glucocorticoids

(d) Vitamin D

 

Answer : Unattempted

Explanation :

Insulin receptors are coupled to autophosphorylation of beta subunit rather than regulation of gene

expression as in the case with rest of the receptors.

Q 192. Nesiritide is a

(a) Brain natriuretic peptide analog

(b) Endothelin R antagonist

(c) Gp IIb/IIIa antagonist

(d) TNF alpha antagonist

Answer : Unattempted

Explanation :

Nesiritide is a brain natriuretic peptide and works in a manner similar to that of frusemide. It is a short

acting drug and is given intravenously. Therefore, it is not useful in chronic congestive heart failure. It is

only used in acute heart failure. Mechanism of action is similar to that of nitrates (i.e., this peptide too

acts by increasing cAMP). Side effect, therefore, is predictably hypotension. This is a first recombinant

peptide to be used for CHF and was approved in 1987.

Q 193. Heparin in pregnant female if given, should be

(a) Topoisomerase inhibitor

(b) Inhibition of microtubule formation

(c) Mitotic cell inhibitor

(d) Exaggerates polymerization and causes the stabili-zation of the microtubules

 Answer : Unattempted

Page 77: Elsevier Jan

Explanation :

Long-term use of heparin is associated with significantly lowered bone density.Note: Various adverse

effects of Heparin areHypocalcemia or osteoporosisBleeding due to

overdoseThrombocytopeniaAlopecia (Transient and reversible)Liver damage (serum transaminases

rises)Hypersensitivity reactionsReferences1 Harrison's Principles of Internal Medicine, (16th Ed.), Pg.

22632 Essentials of Medical Pharmacology, Tripathi KD, (6th Ed.), Pg, 598, 599

Q 194. Paclitaxel acts by

(a) Topoisomerase inhibitor

(b) Inhibition of microtubule formation

(c) Mitotic cell inhibitor

(d) Exaggerates polymerization and causes the stabilization of the microtubules

Answer : Unattempted

Explanation :

It is a complex diterpine taxane obtained from bark of the western yew tree, which exerts cytotoxic

action by a novel mechanism.It enhances polymerization of tubulin, a mechanism opposite to that of

vinca alkaloids.The microtubules are stabilized and their depolymerization is prevented.This stability

results in inhibition of normal dynamic reorganization of the microtubule network that is essential for

vital interphase and mitotic functions.Abnormal arrays or “bundles” of microtubules are produced

throughout the cell cycle.Cytotoxic action of paclitaxel emphasizes the importance of tubulin-

microtubule dynamic equilibrium.The approved indications of paclitaxel are metastatic ovarian and

breast carcinoma after failure of first-line chemotherapy and relapse cases.It has also shown efficacy in

advanced cases of head and neck cancer, small cell lung cancer, esophageal adenocarcinoma, and

hormone refractory prostate cancer.Note:Anti cancer drugs that acts by inhibition of microtubules

(Tubulin) are vinca alkaloids (Vincristine/vinblastine)Anticancer drugs that act by stabilization of

microtubules are Taxanes - (Paclitaxel/Docetaxel)ReferencesEssentials of Medical Pharmacology,

Tripathi KD, (6th Ed.), Pg. 824, 825

Q 195. Tiotropium is used for the treatment of which of the following?

(a) Ptyalism

(b) Asthma

(c) Urinary retention

(d) Ileus

 Answer : Unattempted

Explanation :

Semisynthetic Anticholinergic Drug derivatives are: Homatropine, Atropine methonitrate, Hyoscine butyl

bromide, Ipratropium bromide, Tiotropium bromide.Tiotropium bromide is a recently developed

congener of ipratropium bromide which binds tightly to bronchial M1 / M3 muscarinic receptors

producing long lasting bronchodilation. Binding to M2 receptors is less tight conferring relative M1 / M3

selectivity. Like ipratropium it exhibited bronchial selectivity in action.Inhaled ipratropium / tiotropium are

Page 78: Elsevier Jan

the bronchodilators of choice in COPD. They produce slower response than inhaled sympathomimetics

and are better suited for regular prophylactic use.References1. Essentials of Medical Pharmacology,

KD Tripathi, (6th Ed.), Pg. 106, 110, 222]] 2. Basic & Clinical Pharmacology, Katzung, (10th Ed.), Pg.

114

Q 196. All of the following are useful in relieving Post-op urinary spasm EXCEPT

(a) Darifenacin

(b) Ipratropium

(c) Tolterodine

(d) Bethanechol

Answer : Unattempted

Explanation :

Tolterodine is an antimuscarinic drug that is used to treat urinary incontinence.Tolterodine: This

relatively M3 selective muscarinic antagonist has preferential action on urinary bladder; less likely to

cause dryness of mouth and other anticholinergic side effects. It is indicated in overactive bladder with

urinary frequency and urgency. Since it is metabolized by CYP3A4, dose should be halved in patients

receiving CYP3A4 inhibitors (erythromycin, ketoconazole, etc.)Bethanechol is a parasympathomimetic

choline ester that selectively stimulates muscarinic receptors without any effect on nicotinic receptors.

Bethanechol is used to treat urinary retention resulting from general anesthetic or diabetic neuropathy

of the bladder, etc.ReferencesEssentials of Medical Pharmacology, KD Tripathi, (6th Ed.), Pg. 97, 111

Q 197. Which of the following is not a true statement about clonidine?

(a) Increases parasympathetic outflow

(b) Decreases sympathetic outflow by blocking central alpha receptor

(c) Used in hypertension

(d) Prazosin is used to antagonize side effects of clonidine

 

Answer : Unattempted

Explanation :

This is a centrally acting alpha-2 agonist that decreases the central sympathetic outflow. This is a

popular drug and is used for diabetic diarrhea and for reducing manifestations of opioid withdrawal and

mitigating manifestations of menopause.

Q 198. Facilitated diffusion

(a) Requires energy

(b) Is an uphill process

(c) Occurs with the help of a protein carrier and does not require energy

(d) None of the above

Answer : Unattempted

Page 79: Elsevier Jan

Explanation :

Facilitated diffusion is a carrier-mediated diffusion occuring along the electrochemical gradient, and

hence it does not require energy.

Q 199. Gibbs—Donnan effect states that the presence of nondiffusible ions on one side of

semipermeable membrane

(a) Causes unequal distribution of diffusible ions across the membrane

(b) Is responsible for the development of transmembrane potential

(c) Causes solutions of both the compartments to be electrically neutral at equilibrium

(d) All of the above

 

Answer : Unattempted

Explanation :

According to the Gibbs–Donnan equilibrium, the presence of nondif-fusible ions inside the cell causes

the following effects at equilibrium:Solutions on either side are electrically neutral at equilibrium.The

product of diffusible ions on one side of membrane is equal to the product of diffusible ions on the other

side. Thus, there is unequal distribution of diffusible ions, and hence, ions tend to diffuse along their

gradient causing the development of diffusion potential across the membrane. Therefore, at equilibrium,

transmembrane potential develops.

Q 200. Water molecules pass through

(a) Pores of cell membrane

(b) Lipid bilayer

(c) Both of the above

(d) None of the above

Answer : Unattempted

Explanation :

Water molecule is small in size and has very high kinetic energy; therefore, it can pass through lipid

bilayer like a bullet. It can also pass through the pores lined by proteins through which water as well as

water-soluble substances can pass.

Q 201.  Ions cannot pass through lipid bilayer because

(a) They are large

(b) They have charges

(c) They are not water soluble

(d) None of the above

 Answer : Unattempted

Page 80: Elsevier Jan

Explanation :

Ions have electrical charges, and therefore are repelled by opposite charges present in polar lipids of

the bilayer.Because ion has a charge, it becomes hydrated (water molecules are bonded to it).

Hydrated ion has a larger size, and therefore ion cannot pass through the lipid bilayer.

Q 202.  Pinocytosis requires following

(a) Sodium

(b) Potassium

(c) Calcium

(d) Hydrogen

Answer : Unattempted

Explanation :

Calcium reacts with contractile protein filaments beneath the coated pits, and provides force for

pinching the pinocytic vesicle into the interior of the cell from the cell membrane.

Q 203.  Four weeks residence at high altitude results in

(a) Metabolic acidosis

(b) Metabolic alkalosis

(c) Respiratory alkalosis

(d) Respiratory acidosis

 

Answer : Unattempted

Explanation :

At high altitude when one stays for more than 1 week (when breaking effect of CO2is lost) pulmonary

ventilation increases to five to six times. This causes washing out of CO2 resulting in reduced

CO2 content of blood leading to respiratory alkalosis.

Q 204.  Viscosity of blood

(a) Rises when plasma protein concentration rises

(b) Is more when measured in vivo than in vitro

(c) Increases in severe anemia

(d) None of the above

Answer : Unattempted

Explanation :

Viscosity of blood is directly proportional to the number of cells and concentration of plasma proteins.

Q 205.  Specific gravity

(a) Of RBCs is 1000

Page 81: Elsevier Jan

(b) Of plasma is 1030

(c) Of plasma is 1090

(d) None of above

 Answer : Unattempted

Q 206.  Which of the following does not includes the mechanism of learning and memory?

(a) Changes in the level of neurotransmitter at synapse

(b) Increasing protein synthesis

(c) Recruitment by multiplication of neurons

(d) Spatial reorganization of synapse

Answer : Unattempted

Explanation :

Learning involves such processes as habituation and sensitization. In these, there are changes in the

release of neurotransmitter at the synapses.The key to memory is alteration in the strength of selected

synaptic connections. There is activation of genes and increased protein synthesis. It may take one of

these forms: (i) increased number of neurotransmitter vesicles, (ii) increased number of release sites,

(iii) increase in the number of dendritic spines, and (iv) changes in dendritic geometry and spatial

reorganization of synapses.Addition of new nerve cells after birth and their role in memory has not

been established.ReferencesReview of Medical Physiology, Ganong, (21st Ed.), Pg. 273–4.

Q 207.  Which of the following is not true about endothelin-1?

(a) Bronchodilatation

(b) Vasoconstriction

(c) Decreased GFR

(d) Has inotropic effect

 

Answer : Unattempted

Explanation :

Endothelins cause bronchoconstriction.Biologic effects of endothelins:Hemodynamic effectsContracts

vascular smooth muscle; veins more sensitive than arteriesCardiac effectsPositive inotropic and

chronotropic effects on myocardiumIntense vasoconstriction of coronary arteriesNeuroendocrine

effectsIncreased plasma levels of ANP, renin, aldosterone, and catecholaminesModulates synaptic

transmissionRenal effectsIncreases renal vascular resistanceDecreases GFR, renal blood

flowIncreases Na+ reabsorption through hemodynamic actions, but decreases Na+ reabsorption

through inhibition of Na+ – K+ pump.ReferencesReview of Medical Physiology, Ganong, (21st Ed.), Pg.

602

Q 208.  Not true about blood pressure measurement is

(a) Cuff width should be 40% of arm circumference

Page 82: Elsevier Jan

(b) Diastolic blood pressure is indicated by fourth Korotkoff sound

(c) Small cuff measures spuriously elevated diastolic blood pressure

(d) Mönckeberg's sclerosis causes pseudohypertension

Answer : Unattempted

Explanation :

American Heart Association recommends that the pneumatic bag within the cuff should be 20% wider

than the diameter of the limb or its width should be 40% of the arm circumference.It is a debatable

point whether phase IV of Korotkoff sound (“muffling”) or phase V (“disappearance”) be taken as the

diastolic BP. First 3 phases have sound only during systole because cuff pressure is lower than the

systolic but still higher than diastolic BP, hence no flow during diastole. Muffling means the sound

becomes continuous, during systole as well as diastole. Sound during diastole means flow during

diastole; cuff pressure has reached the diastolic pressure. Disappearance of sound only means that

the flow has become laminar. Since disappearance of sound is more reliably reproducible, it is

commonly taken as diastolic BP.In situations of greater turbulence of blood (e.g., exercise), difference

between phase IV and phase V is increased, phase V is much below the diastolic pressure.Small cuff

may not occlude the artery completely. Artery would open completely at an earlier point. Hence,

diastolic pressure may be recorded falsely high.Monckenberg sclerosis is a degenerative disease

characterized by dystrophic calcification of tunica media. Since arterial walls become stiff, it gives

higher pressure by cuff measurement—pseudohypertension.References1 Textbook of Medical

Physiology, Boron and Boulpaep, Pg. 439, 440.2 Kumar and Clark, (3rd Ed.), Pg. 626

Q 209.  Normal rate of excretion of H+ ions in kidneys is

(a) 40 mEq/day

(b) 4400 mEq/day

(c) 400 mEq/day

(d) 1000 mEq/day

 Answer : Unattempted

Q 210.  Most common symptom of alcohol withdrawal is

(a) Body ache

(b) Tremor

(c) Diarrhea

(d) Rhinorrhea

Answer : Unattempted

Explanation :

The most common symptom in alcohol withdrawal is tremor. Nausea, vomiting, weakness, irritability,

insomnia, and anxiety are the other common withdrawal symptoms. The other choices given are more

commonly found in opioid withdrawal.ReferencesA Short Textbook of Psychiatry, Neeraj Ahuja, (6th

Ed.), Pg. 41

Page 83: Elsevier Jan

Q 211.  Naltrexone is used in a case of opioid dependence to

(a) Prevent respiratory depression

(b) To treat withdrawal symptom

(c) To prevent relapse

(d) To treat overdose of opioid

 

Answer : Unattempted

Explanation :

An overdose of opioid can be treated with opioid antagonists (e.g., naloxone and naltrexone).

Naltrexone is an orally available opioid antagonist, which block the opioid receptors. When an opioid-

dependent person takes opioids, there are no pleasurable experiences. Thus, he does not continue to

take opioids, thereby preventing relapse.ReferencesA Short Textbook of Psychiatry, Neeraj Ahuja, (6th

Ed.), Pg. 47

Q 212.  Altered perception of real objects is

(a) Illusion

(b) Delusion

(c) Hallucination

(d) Delirium

Answer : Unattempted

Explanation :

Illusion is the altered perception of the real objects, whereas hallucination is perception in absence of

any stimulus.Disorientation to time, place, and person is pathognomonic symptom

of delirium.Delusion is a disorder of thought in which person has a fixed, firm, and false belief not

shared by others and not in keeping with sociocultural background of the person.ReferencesA Short

Textbook of Psychiatry, Neeraj Ahuja, (5th Ed.), Pg. 14

Q 213.  A patient presented to the department with acute psychosis, he has history of haloperidol

intake since last 2 days in doses of 20 mg/day. He has his-tory of an episode before presenting

of the casualty with tongue protrusion, stiffness, oculogyric crisis, and abnormal posture of

limbs and trunk without loss of consciousness. Administration of diphenhydramine HCl

releived symptoms with a minute. Most probable diagnosis is

(a) Acute dystonia

(b) Akathisia

(c) Tardive dyskinesia

(d) Neuroleptic malignant syndrome

 Answer : Unattempted

Explanation :

Page 84: Elsevier Jan

Acute dystonia most commonly occurred within the first week of start of haloperidol. Symptoms are

facial grimacing, torticollis, oculogyric crisis, and opisthotonus (abnormal contraction of spinal

muscles). Treatment is diphenhydramine hydrochloride and benztropine mesylate.References1. A

Short Textbook of Psychiatry, Neeraj Ahuja, (6th Ed.), Pg. 190–1952. Essentials of Medical

Pharmacology, Tripathi KD, (5th Ed.), Pg. 397

Q 214.  A 2-year-old child with normal mile stones, playing on his own and looking at his hands does

not respond to the father and mother; the diagnosis

(a) Autism

(b) ADHD

(c) Depression

(d) Schizophrenia

Answer : Unattempted

Explanation :

Autism (i.e., marked impairment in reciprocal social and interpersonal interactions)Absent social

smile.Lack of eye to eye contact.Lack of awareness of others existence or feeling; treats others as

furniture.Lack of attachment to parents and absence of separation anxiety.No or abnormal social play;

prefers solitary games.Lack of initiative behavior.ADHDPoor attention span with distractibility,

hyperactivity, and impul-sivity.Onset before 7 years of age.Male:female: 6–8:1.Depression: Sadness of

mood or loss of interest and/or pleasure in almost all activities (pervasive sadness), hopelessness, and

helplessness.Insomnia, loss of appetite and weight.General body ache, pain (common in 3rd and 4th

decade).The only important differential diagnosis is childhood onset schizophrenia.Delusion and

hallucinations are present in schizophrenia but absent in autism.Age of onset is ≪ two and half years

in infantile autism, while it is after 5–6 years in childhood onset schizophrenia.Moderate to severe MR

and epilepsy are common in infantile autism, while rare in childhood schizophrenia.ReferencesA Short

Textbook of Psychiatry, Neeraj Ahuja, (6th Ed.), Pg. 177–79

Q 215.  In India most common substance of abuse is

(a) Cannabis

(b) Tobacco

(c) Alcohol

(d) Opium

 

Answer : Unattempted

Explanation :

Cannabis (Indian hemp) is the most commonly used illicit drug. However, among all substances (illicit

and licit), tobacco is the most common substance of abuse.References1. A Short Textbook of

Psychiatry, Neeraj Ahuja, (6th Ed.), Pg. 482. Kaplan & Sadock's Comprehensive Textbook of

Psychiatry, (9th Ed.), Pg. 444

Page 85: Elsevier Jan

Q 216.  4-year-old child presents with delayed speech and poor concentration. He is not interested in

friends and has difficulty in communication most probable diagnosis in this patient is

(a) Autism

(b) ADHD

(c) Specific learning disability

(d) Mental retardation

Answer : Unattempted

Explanation :

Autism typically the onset occurs before the age of two and half years. In some cases, the onset may

occur later in the childhood. The characteristic features of autism are as follows:Marked impairment in

reciprocal social and interpersonal interaction, e.g., impairment in making friends and prefers solitary

games.Marked impairment in language and nonverbal communication.Abnormal behavioral

characteristics, especially repetitive behaviors.ReferencesA Short Textbook of Psychiatry, Neeraj

Ahuja, Pg. 176–177

Q 217.  Learning does not include

(a) Modeling

(b) Catharsis

(c) Exposure

(d) Sensitization

 

Answer : Unattempted

Explanation :

Catharsis is another name of ventilating one's emotions. It is used in psychotherapies for bringing out

the emotional content. The other choices are different forms of learning. Learning is the acquisition and

development of behaviors, including skills, knowledge, understanding, values, and wisdom. It is the

product of experience and education. Learning ranges from simple forms such as habituation and

classical conditioning seen in many animal species to more complex activities such as ~ seen only in

relatively intelligent animals.Types of learning:Simple non-associative

learningHabituationSensitizationAssociative learningOperant conditioningClassical

conditioningImprintingObservational learning, modelingPlayElectronic learningReferencesSciences

Basic to Psychiatry, Puri, Tyrer, (2nd Ed)

Q 218.  Which of the following is a basis for Psychodynamic theory of mental illness?

(a) Unconscious conflict

(b) Maladjusted reinforcement

(c) Organic neurological problem

(d) None of the above

Answer : Unattempted

Page 86: Elsevier Jan

Explanation :

Psychodynamic theory models of psychiatric illness are based on the unresolved conflict in the mind

and, especially of the past responsible for the current problem. The psychodynamic models are

interested in how childhood relationships and experiences affect future mental health.Key features of

thepsychodynamic approach are as follows:Our behavior and feelings as adults are rooted in our

childhood experiences.Relationships (particularly parenting) are of primary importance in determining

how we feel and behave.Our behavior and feelings are powerfully affected by the meaning of events to

the unconscious mind.Information can be obtained from dreams, irrational behavior, and what patients

in therapy say.The personality is made up of three distinct structures—id, ego, and super ego.Defense

mechanisms are used to protect the ego, e.g., repression.Children develop through a series of fixed

stages—oral, anal, and phallic.ReferencesA Short Textbook of Psychiatry, Neeraj Ahuja, (5th Ed.), Pg.

211–212

Q 219.  MRI is an absolute contraindication in patients with

(a) Pace maker

(b) Prosthetic heart valve

(c) Middle ear prosthesis

(d) First trimester pregnancy

 

Answer : Unattempted

Explanation :

Contraindications are:AbsoluteCardiac pacemakerFerromagnetic vascular clips and Intracranial

clipsInternal hearing aids or metallic orbital foreign bodyClaustrophobiaRelative1st trimester

pregnancyMiddle car prosthesisIndications are:PIVD or degenerative disk diseasesSpinal cord

tumorSpinal cord abscesses (spinal osteomyelitis)Spinal canal stenosisReferencesCampbell's

Operative Orthopedics, (10th Ed.), Pg. 124, 125

Q 220.  Hair on end appearance on X-ray is seen in

(a) Thalassemia

(b) Sickle cell anemia

(c) Rickets

(d) Scurvy

Answer : Unattempted

Q 221.  Radiocontrast is not a contraindicated in

(a) Renal failure

(b) Patient on metformin

(c) Dehydration

(d) Obesity

Page 87: Elsevier Jan

 

Answer : Unattempted

Explanation :

Radio contrast is excreted through kidney, so various conditions—CRF, CCF, MM, DM, and patient on

metformin—are C/I for radiocontrast.ReferencesHarrison's Principles of Internal Medicine, (16th Ed.),

Pg. 1647

Q 222.  GFR is measured with which of the following?

(a) Iodohippurate

(b) Tc99m-DTPA

(c) Tc99m-MAG3

(d) Tc99m-DMSA

Answer : Unattempted

Explanation :

Renal ScanI-131 01H (Orthoiodohippurate) has been largely replaced by Tc99m-MAG3 for the

evaluation of renal tubular function/ effective renal plasma flowTc99m-DTPA (Diethylene triamine

pentaacetic acid). It is the agent of choice for the assessment ofPerfusionGFR - it is the agent of

choice of GFRObstructive uropathyVesicourethral refluxTc99m-DMSA (dimercaptosuccinic acid). It is

suitable for the imaging of the functioning cortical mass pseudotumor versus the lesionTc99m-

Mercaptoacetyltriglycine (MAG3). True renal plasma flow is detected. It has replaced

DTPA.ReferencesGrainger and Allison's Diagnostic Radiology: A Textbook of Medical Imaging (4th

Ed.), Pg. 1500; (5th Ed.), Pg. 141, 150, 846

Q 223.  Egg on side appearance on chest X-ray is seen in

(a) Uncorrected TGA

(b) TOF

(c) TAPVAC

(d) Ebstein's anomaly

 

Answer : Unattempted

Explanation :

The following points about the different radiological features related to heart should be known:Egg in

cup–constrictive pericarditisEgg on side (egg-shaped heart)–D-TGAConvex left side heart–L-TGABoot-

shaped heart–TOFSnowman heart or figure of 8–TAPVACStraight left cardiac border–Ebstein's

anomalyBox-shaped heart–ASDPear-shaped heart/water bottle heart–pericardial effusionJug handles

appearance/cottage loaf sign–primary pulmonary hypertension

Q 224.  Snow man heart is seen in

(a) Picture archiving and communicating system

(b) Portal-archiving common system

Page 88: Elsevier Jan

(c) Planning-archiving corbor scheme

(d) Photo-archiving computerized system

Answer : Unattempted

Explanation :

Radiographic results are pathognomonic in older children if the anomalous pulmonary veins enter the

innominate vein and persistent left superior vena cava (SVC). A large supracardiac shadow together

with the normal cardiac shadow may form the snowman appearance.

Q 225.  Which of the following is not seen on bronchoscopy?

(a) True vocal cords

(b) Bifurcation of trachea

(c) Terminal bronchus

(d) Subcarinal lymph nodes

 

Answer : Unattempted

Explanation :

Bronchoscopy is the process of direct visualization of the tracheobronchial tree.Flexible fiberoptic

bronchoscope is passed though either mouth or nose, between vocal cords, and into trachea.The

ability to flex the scope makes it possible to visualize virtually all airways to the level of subsegmental

bronchi.The bronchoscopist is able to identify endobronchial pathology, including tumors, granulomas,

bronchitis, foreign bodies, and sites of bleeding samples from airway lesion that can be taken by

several methods including, washing, brushing, and biopsy.ReferencesHarrison's Principles of Internal

Medicine, (15th Ed.), Pg. 1544

Q 226.  Investigation of choice for interstitial lung disease is

(a) Chest X-Ray

(b) HRCT

(c) CECT

(d) MRI

Answer : Unattempted

Explanation :

HRCT is of proven value in the diagnosis of diffuse lung disease (like interstitial lung diseases),

particularly in the early stages when the chest radiograph is normal and for follow-up.HRCT clearly

depicts distribution and higher definition of appearances of pulmonary parenchymal disease.Nowadays

HRCT is used for detection of bronchiectasis, and surgery is undertaken without preoperative

bronchography. Severity and extent of bronchiectasis is demonstrated.HRCT is excellent for the

detection and evaluation of emphysema also.HRCT can identify regions most suitable for biopsy at a

time when the chest radiograph is normal.References1 Grainger and Allison's Diagnostic Radiology: A

Textbook of Medical Imaging, (4th Ed.), Pg. 278, 2792 Textbook of Radiology and Imaging, Sutton,

(7th Ed.), Pg. 33

Page 89: Elsevier Jan

Q 227.  PACS stands for

(a) Picture archiving and communicating system

(b) Portal-archiving common system

(c) Planning-archiving corbor scheme

(d) Photo-archiving computerized system

 

Answer : Unattempted

Explanation :

In medical imaging, picture archiving and communication systems (PACS) are computers or networks

dedicated to the storage, retrieval, distribution, and presentation of images. The medical images are

stored in an independent format. The most common format for image storage is DICOM (digital

imaging and communications in medicine).

Q 228.  Which of the following is TRUE about alpha fetoprotein?

(a) Half-life is 5–7 days

(b) Always increased in Wilson's disease

(c) Maximum level is reached at the 20th week of preg-nancy

(d) Yolk sac is the predominant source during the fetal period

Answer : Unattempted

Explanation :

AFP is produced initially by fetal yolk sac in small quantities and then in larger quantities by fetal liver

on yolk sac degeneration.Tumors that secrets AFP are:Principles tumor is endodermal sinus tumor

(yolk sac tumor)NeuroblastomaHepatoblastomaHepatocellular carcinomaRare tumors are malignant

mixed mullerian tumor and Wilms' tumor. AFP is rarely elevated but when it is elevated it may serve as

a marker of disease and progression of recurrence.Increase in AFP is seen teratomas (rarely), serum

levels of AFP which are often correlated with tumor size. Resection is usually associated with a fall in

serum levels. Serum levels are useful in assessing response to treatment.AFP is detected only in

minute amounts in healthy men (less than 25 mg/mL; 25 wg/c).It is never produced by pure

seminomas or choriocarcinomas.Histologically reported pure seminoma with an elevated AFP level

should be treated as a nonseminoma germ cell tumor.Half-life is 5–7 days and the level should return

to normal within 25–35 days or tumor removal.References1 Teitz Textbook of Clinical Chemotherapy,

Pg. 17552 5-minute Urology Consult, Leonard G Gomello, Pg. 621

Q 229.  Anal fissure can be best diagnosed by

(a) Clinical examination and visualization

(b) DRE

(c) Anoscopy

(d) Endosonography

Page 90: Elsevier Jan

 

Answer : Unattempted

Explanation :

In long standing anal fissure a sentinel skin tag can usually be displayed. This together with a typical

history and a tightly closed, puckered anus is almost pathognomonic of the condition.By gently parting

the margins of the anus, the lower end of the fissure can be seen.Because of the intense pain it

causes, digital examination of the anal canal should not be attempted. Digital and proctoscopic

examinations will require general anesthesia.Anal fissure is longitudinal split in the anodern (pain

sensitive skin) of the distal anal canal.Ecotpic site (other than midline posterior) can be due to Crohn's

disease, TB, HIV or STD (like chancroid, LGV, syphilis, chlamydia), CMV, Kaposi sarcoma, B cell

lymphoma, HSV), squamous cell carcinoma.The proximal limit of and fissure is dentate lineMost

common symptom is painMost common cause associated is ischemiaChronic anal fissures commonly

present with a deep canoe shaped ulcer, a hypertrophied papilla at the upper end and, a skin

tag.TreatmentRelaxation of internal sphincter (mainstay i.e. by using topical substances like nitric

oxide, GTN, diltiazem)Operative–lateral anal sphincterotomy.References[1. Bailey and Love's Short

Practice of Surgery, (24th Ed.), Pg. 1253[2. Sabiston's Textbook of Surgery, (18th Ed.), Pg. 1444

Q 230.  A New born child presented with distended abdomen with bilateral gas under diaphragm. Most

likely cause is

(a) Perforation of stomach

(b) Necrotizing enterocolitis/pneumatosis cystosis intestinalis

(c) Duodenal atresia

(d) Intestinal atresia

Answer : Unattempted

Explanation :

Necrotizing enterocolitis is a disease characterized by crepitant necrosis of the gut and is the most

common surgical emergency affecting neonatal gut, occurring particularly in premature newborns.A

multifactorial etiology is likely.Immaturity of the gut and immune system, formula feeds (breast milk is

protective), bacterial infection and impaired gut blood flow have been implicated.The baby typically

develops abdominal distension, bloody stools and bilious aspirates, and rapidly deteriorates.Patchy or

extensive pneumatosis intestinalis progresses to necrosis and perforation of the gut–gas under

diaphragm.Milder cases of necrotizing enterocolitis respond to antibiotics, gut rest and delayed

reintroduction of feeds. However, perforated necrotizing enterocolitis requires resection/ excision of

perforated segment followed by temporary stoma or primary anastomosis.References[Newborn

Surgery, Prem Puri, (2nd Ed.), Pg. 501–08

Q 231.  A 40-year-old diabetic presents with an ischiorectal abscess and high fever, and requires

incision and drainage under general anesthesia. He has been on treatment with metformin for

long. Investigation shows a blood sugar of 350 mg/dl, and urinary ketones are positive. Surgery

should be done

(a) Immediately

Page 91: Elsevier Jan

(b) After control of ketosis with metformin

(c) After control of ketosis with insulin

(d) After control of ketosis, and after normalization of glucose

 

Answer : Unattempted

Explanation :

Diabetic ketoacidosis is a grave anesthetic risk, and should be controlled before general anesthesia.

Preoperative control is always achieved with insulin, not by oral hypoglycemics. Blood glucose levels

will not normalize till the pus is drained.

Q 232.  A 30-year-old severe asthmatic, smoker requires elective inguinal hernia repair. He is

adequately controlled on salbutamol inhalers and oral prednisolone. Sputum is sterile.

Preoperatively, which of the following measures is LEAST required?

(a) Continue salbutamol

(b) Continue steroids, but change to intravenous steroids

(c) Stop smoking for at least 2 weeks

(d) Start prophylactic antibiotics

Answer : Unattempted

Explanation :

Treatment for asthma should continue. Since this is an elective procedure, the patient should be made

to stop smoking for 4 weeks before surgery (and preferably for ever afterwards, although few surgeons

can achieve this miracle!). Many surgeons give prophylactic antibiotics, but these are not mandatory.

Q 233.  A patient was evaluated prior to a cholecystectomy. Which of the following findings represent

the greatest procedure risk?

(a) Age = 70 years

(b) Alkaline phosphatase = 1000 I.U. (normal < 200)

(c) Ejection fraction 25% on echocardiography

(d) Platelet count 80000/ml

 

Answer : Unattempted

Explanation :

Although all the factors listed here worsen the results from the procedure, patients with an ejection

fraction below 35% have a marked risk of operative mortality. Other conditions that are serious risk

factors are active hepatitis (but not obstructive jaundice), acidosis, and recent myocardial infarction,

and are virtual contraindications to surgery.

Q 234.  A 60-year-old male is to undergo a colectomy for cancer. Which of the following drug

combinations is most acceptable prophylaxis?

(a) Cefotaxime and metronidazole

Page 92: Elsevier Jan

(b) Clindamycin and erythromycin

(c) Ciprofloxacin and amikacin

(d) Cloxacillin and vancomycin

Answer : Unattempted

Explanation :

In general, Staphylococcus aureus is the commonest agent causing wound infection after any surgery.

Streptococcus is another important pathogen. In gastrointestinal surgery, E coli, other enterobacteria,

and anaerobes deserve special attention. Cefotaxime is a broad-spectrum antibiotic, and

metronidazole is excellent for anaerobic cover. Together, both will cover most pathogens likely to be

encountered in gastrointestinal operations. Clindamycin, erythromycin, cloxacillin and vancomycin are

narrow-spectrum agents (especially useful for gram positive bacteria), and are ineffective against the

enterobacteria. Ciprofloxacin and amikacin have a broad spectrum, but both are useless against

streptococcus and anaerobes. Note that cefotaxime/metronidazole, while adequate, is not necessarily

the best antibiotic combination in this case. Co-amoxyclav +amikacin+metronidazole is probably

superior. Some protocols use clindamycin (which provides anaerobic cover) along with a broad-

spectrum agent such as amikacin.

Q 235.  A patient with bile duct stones and marked jaundice requires surgery. Which is the LEAST

important concern?

(a) Infection

(b) Wound healing

(c) Clotting derangements

(d) Dehydration

 

Answer : Unattempted

Explanation :

Patients with jaundice are susceptible to infection and coagulopathy, which are serious concerns

during surgery. Dehydration can result in irreversible renal shutdown, and is the most dangerous

complication of all. Wound healing is impaired in jaundice, but this is a less serious problem.

Q 236.  Safe dosage limits for local anesthetic are

(a) Lignocaine: 40 ml of 2%

(b) Bupivacaine: 40 ml of 1%

(c) Ropivacaine: 40 ml of 1%

(d) Prilocaine: 40 ml of 1%

Answer : Unattempted

Explanation :

Safe doses of local anesthetic for an adult are lignocaine 400 mg, bupivacaine 150 mg, Ropivacaine

225 mg, and prilocaine 400 mg.

Page 93: Elsevier Jan

Q 237.  In parotid surgeries, the anatomical markers for localization of the facial nerve include all of the

following EXCEPT

(a) Posterior belly of digastric

(b) Mastoid process

(c) Retrograde dissection by tracing its branches

(d) Inferior belly of omohyoid

 

Answer : Unattempted

Explanation :

Landmark for facial nerve in parotid surgeryThe pointed lower end of the tragal cartilageThe bony

external auditory meatusThe mastoid processThe sternomastoid muscleThe posterior belly of the

digastricThe retracted lower pole of the parotid glandReferences[Bailey and Love's Short Practice of

Surgery, (20th Ed.) Pg. 570

Q 238.  A 45-year-old female has a solitary gall stone of 5 cm in size which was incidentally diagnosed

by ultrasound. She has no symptom. What should be the best line of management?

(a) Cholecystectomy only, if she develops biliary colic

(b) Lap cholecystectomy immediately

(c) Lap cholecystectomy even if she is asymptomatic after 2 months

(d) Open cholecystectomy even if she is asymptomatic after 2 months

Answer : Unattempted

Explanation :

Symptomatic gallstones are the main indication for cholecystectomy.Absolute contraindications for the

procedure are uncontrolled coagulopathy and end-stage liver disease.Various indication of

cholecystectomy in asymptomatic gall stone diseaseStone size > 3 cm in diameterGallstone in diabetic

patientCalcified in diabetic patientGall bladder cholesterosisAdenomyomatosisCongenitally anomalous

gall bladderGall stones in patient with immunocompromised or on immunosuppressant

therapyReferences[1. Schwartz's Principles of Surgery, (8th Ed.), Pg. 1204[2. Harrison's Principle of

Medicine, (17th Ed.), Pg. 1195[3. Bailey and Love's Short Practice of Surgery, (25th Ed), Pg. 1121

Q 239.  A patient with ITP has a platelet count of 50,000, there is a plan for splenectomy. What is the

best time for the infusion of platelets in this patient?

(a) Immediately after ligating splenic vein

(b) Immediately after removal of spleen

(c) After incision

(d) Next day of surgery

 Answer : Unattempted

Explanation :

Patients with extremely low platelet counts (≪10,000/mm3) should have platelets available for surgery,

Page 94: Elsevier Jan

but should not receive them preoperatively. This is because the transfused platelets will get

sequestered in the spleen and are lost with splenectomy.Once the splenic pedicle is ligated, platelets

are given to those who continue to bleed.References[1. Schwartz's Principles of Surgery, (8th Ed.), Pg.

1306[2. Baily and Love's Short Practice of Surgery, (24th Ed.), Pg. 1089, (25th Ed.) Pg. 1106

Q 240.  A Male presents with blunt injury to abdomen, hypotensive, responding to IV fluids, mild

tenderness in the epigastrium. What is the next line of management?

(a) Observation

(b) Further imaging of upper abdomen

(c) Exploratory laparotomy

(d) Laparoscopy

Answer : Unattempted

Explanation :

In blunt trauma to abdomen, it is necessary to determine whether an exploratory laparotomy is

necessary. Physical examination of the abdomen is unreliable in making this determination difficult.

However, most authorities agree that the presence of abdominal rigidity or gross abdominal distension

in a patient with truncal trauma is an indication for prompt surgical exploration. For the majority of

patients suffering blunt abdominal trauma, it is not clear whether exploration is needed and thus

detailed imaging of the abdomen should be done particularly if the patient is hemodynamically

stable.Blunt abdominal trauma is currently evaluated by ultrasound in most major trauma centers with

CT being done in selected cases.Indication for urgent-laparotomy in blunt injury abdomenUnexplained

hypovolemiasPeritonitisOther associated injuriesReferences[Schwartz's Principles of Surgery, (8th

Ed.), Pg. 141–42

Q 241.  Which of the following is NOT yet treated by intervention radiology?

(a) Portal hypertension

(b) Ileal stricture

(c) Bile duct stricture

(d) Renal stone

 

Answer : Unattempted

Explanation :

Intervention radiology has changed the treatment of many disorders. Portal hypertension caused by

liver disease can be treated by a transjugular intrahepatic portasystemic shunt (TIPS). Bile duct

strictures can be treated by stenting under radiological control. Renal stones are treated by shock wave

lithotripsy or nephrolithotripsy, both under radiological control. Several vascular disorders, notably

coronary artery disease, are amenable to interventional radiology. In contrast, ileal strictures require

open (or sometimes laparoscopic) surgery, without the requirement of radiology during the procedure.

Q 242.  While assessing a patient, the physician should observe all of the following dictums, EXCEPT:

Page 95: Elsevier Jan

(a) Do not make the patient completely naked during clinical examination

(b) Take specific permission before doing a socially sensitive investigation, such as a test for HIV

(c) If rectal examination is unlikely to contribute to the management, do not do it

(d) If examination of particular area causes pain, do not examine the area

Answer : Unattempted

Explanation :

Tenderness is a valuable sign, and therefore a gentle examination should be carried even if

examination causes pain. Avoid making a patient completely naked; it causes needless

embarrassment. It is only ethical to take permission for tests/procedures that may be refused. Rectal

and vaginal examinations are obviously unnecessary in a patient who presents with a scalp sebaceous

cyst or a finger fracture.

Q 243.  A middle-aged man will undergo surgery for a large ventral hernia. The expected procedure

time is 90 minutes, and the expected blood loss is 250 ml. You will give a preoperative blood

transfusion

(a) If the hemoglobin is below 12 G/dl

(b) If the hemoglobin is below 10 G/dl but not if it is between 10 and 12

(c) If the hemoglobin is below 8 G/dl but not if it is between 8 and 12

(d) Always, regardless of hemoglobin, in view of the high expected blood loss

 

Answer : Unattempted

Explanation :

Preoperative blood transfusion is required in the severely anemic so that they tolerate anesthesia

better. The expected blood loss is too low to justify preoperative transfusion as a routine. Blood should

be given during surgery if hemorrhage exceeds two units of blood. In the present case, if the patient's

hemoglobin is 9 and estimated bleeding is 300 ml (about a unit of blood), most persons would not

transfuse during surgery.

Q 244.  The following is true for pre- and intraoperative anesthesia management:

(a) Pre-anesthetic complete blood counts are essential even for medically fit patients about to undergo minor surgery

(b) Recent myocardial infarction is not by itself a contraindication for elective anesthesia

(c) Hepatitis does not affect recovery from anesthesia

(d) Bipolar diathermy is safer than monopolar for patients on pacemaker

Answer : Unattempted

Explanation :

The use of a bipolar diathermy during surgery is much safer than monopolar if the patient is on a

pacemaker. Blood counts are usually not required in minor surgery if the patient is otherwise fit. A

recent myocardial infarct is among the strongest contraindications to elective anesthesia. Anesthesia is

also very risky in patients who have hepatitis.

Page 96: Elsevier Jan

Q 245.  The protein value of food is often expressed as nitrogen. Approximately how much nitrogen is

present in 100 G protein?

(a) 5 – 10 G

(b) 15 – 20 G

(c) 25 – 30 G

(d) 35 – 40 G

 Answer : Unattempted

Explanation :

For most proteins, 6.25 G protein contains 1 G nitrogen.

Q 246.  Complications Hemithyroidectomy include all of the following EXCEPT

(a) Wound hematoma

(b) Injury to external branch of superior laryngeal nerve

(c) Recurrent laryngeal nerve injury

(d) Hypocalcemia

Answer : Unattempted

Explanation :

Hypocalcemia can occur following total thyroidectomy but not hemithyroidectomy.Complications of

thyroidectomy (frequently asked facts)Hemorrhage leading to wound hematoma.Respiratory

obstruction – it occurs due to laryngeal edema, hematoma, and rarely due to collapse and kinking of

trachea.Recurrent laryngeal nerve paralysis– mostly transient, permanent paralysis rarely

occurs.Hypocalcemia– common in 2nd and 5th day after surgery.Thyrotoxic crisis (storm)– occur in

inadequately prepared thyrotoxic patients.Thyroid insufficiency– long term complication.Rarely–

external branch of superior laryngeal nerve palsy.References[Bailey and Love's Short Practice of

Surgery, (24th Ed.), Pg. 796, (25th Ed.), Pg. 782

Q 247.  All of the following are TRUE about Volvulus of colon EXCEPT

(a) Sigmoid volvulus is more common than cecal

(b) Common in Psychiatric ill patients

(c) Past history of acute abdominal pain with passage of large amount of gas by flatus tube insertion

(d) Sigmoidoscopy is contraindicated

 Answer : Unattempted

Explanation :

Colon is the most common site of volvulus in colonSigmoid colon (65%)–most commonCecal volvulus

(30%)Transverse colon volvulus (3%)Splenic flexure volvulus (2%)Sigmoidoscopy in sigmoid volvulus

may prove therapeutic (and is not contraindicated).The rotation in sigmoid volvulus usually occurs

inanticlockwise direction, while in cecal volvulus, rotation occurs in clockwisedirection.Sigmoid

Page 97: Elsevier Jan

volvulus (frequently asked facts)Precipitating factorsLong mesentery of the pelvic colonNarrow

attachment at the base of pelvic micro colon.Long redundant, pendulous sigmoidLoaded colon due to

high residue dietDiverticulitis with a bend, or adhesions (Peri diverticulitis)Radiological signsCoffee

bean appearanceBent inner tube sign–on plain X-Ray abdomenOmega signBird beak appearance–on

contrast enemaManagementEndoscopic derotation–done in absence of gangrene or

perforation.Surgery–indications of surgeryIf gangrene or perforation occurElective after endoscopic

derotationProceduresIf gangrene or massive contamination–Hartmann procedureIf viable bowel–

sigmoidopexy, extra peritonealization of sigmoid resection and end to end

anastomosis.References[1. Bailey and Love's Short Practice of Surgery, (24th Ed.), Pg. 1197, Pg.

1191, 1192, 1194, 1200[2.CSDT, (12th Ed.), Pg 722

Q 248.  Which of the following is a nonabsorbable suture?

(a) Polypropylene

(b) Polyglyconate

(c) Polyglycolic acid

(d) Polydioxanone

Answer : Unattempted

Explanation :

Polypropylene is used as a monofilament suture with tensile strength of more than 1 year.It is non

absorbable and remains encapsulated in body tissues.It has low tissue reaction and is frequently used

in cardiovascular surgery, plastic surgery, subcuticular skin closure, etc.Other example of non-

absorbable sutures are Silk, Nylon, Polyester, Linen, surgical steel, etc.Absorbable

SuturesPolyglyconate and Polydioxanone are used as monofilament suture and get completely

absorbed at around 180 days.Polyglycolic acid is used as braided multifilament suture and gets

completely absorbed by 60–90 days.Other examples of absorbable sutures are catgut, polyglactin,

polyglecaprone, etc.References[Bailey and Love's Short Practice of Surgery, (24th Ed.), Pg. 98–99

Q 249.  Abbe Estlander Flap is based on which artery?

(a) Labial artery

(b) Facial artery

(c) Lingual artery

(d) Internal mammary artery

 Answer : Unattempted

Explanation :

Abbe flapIn 1898, Abbe first described the lip switch flap (Abbe flap) which was originally designed as

a complete philtral reconstruction for the relief of the bilateral cleft-lip deformity.Today it is used for both

upper and lower lip reconstruction.The Abbe flap is designed half of wide as the lip defect.During flap

elevation, the labial artery supply to the flap is carefully preserved.The flap pedicle can be safely

severed in 3 weeks to allow full mouth opening.Estlander flapFor those defects that extend laterally to

include the oral commissure, the Estlander flap (lateral lip switch flap) is a useful procedure.This is a lip

Page 98: Elsevier Jan

switch technique similar to the Abbe flap and typically uses a medially based full thickness upper

lip.Similar to the Abbe flap, the width of the flap is designed half as wide as the defect.Once the flap

pedicle is divided, the oral commissure needs to be reconstructed with a commissure

plasty.References[Skin Flaps in Facial Surgery, Daniel Buchen, (2007 Ed.), Pg. 130–135

Q 250.  Internal oblique, external oblique, and transverses abdominis are retracted laterally in which

approach?

(a) Classical renal access

(b) Lumbotomy

(c) Laparoscopic access

(d) Spigelian hernia

Answer : Unattempted

Explanation :

The surgical procedure starts with the patient in partial left lateral decubitus position (30 degrees)

under general anesthesia and continuous peridural anesthesia. The transperitoneal access was used

in all cases.The skin incision must be extended to the same size of the assistant surgical glove, for

allowing the insertion of the device designed for introducing the hand.The incision begins at the lateral

margin of the rectus muscle of the abdomen, 2 cm above the symphysis pubis and is obliquely

extended until the anterosuperior iliac spine.The external oblique, internal oblique, and transverses

muscle of the abdomen are retracted, the peritoneum is opened and the colon is displaced medially.

The ureter is identified, separated then 10 mm (camera) introduced into abdominal cavity through an

opening and guided by the surgical hand, which was introduced in the inguinal incision.

Q 251.  Fluid in body in which ABO antigen does not occur

(a) Serum

(b) Saliva

(c) Sweat

(d) CSF

 Answer : Unattempted

Explanation :

Blood group antigens are present in Blood Salivary gland

a. Saliva

b. Pancreas

c. Kidney

d. Liver

e. Lung

Testes

g. Semen

Page 99: Elsevier Jan

h. Amniotic fluid

1. Ganong 22nd Ed., Pg. 537.

Q 252.  7-year-old boy with h/o trauma 2 months back now presents with fever and acute pain over

thigh on X-ray femoral shaft shows lesion with multiple laminated periosteal reaction next line

of management is

(a) Core biopsy

(b) Tc99 MDP scan

(c) MRI

(d) Crp measurement

Answer : Unattempted

Explanation :

a. MRI is the amaging modality of choice for evaluating bone marrow involvement as well as non calcific

soft tissue lesion

b. TC bone sacn are used to access degree of osteoblastic activity best indicators are metastatic

carcinoma and lymphoma of bone

c. Biopsy should usually be the final staging procedure

d. As histological diagnosis has to be made before doing any management biopsy is the best next

procedure in this case

1. CPDT, 4th Ed., Pg. 303.

Q 253.  A child presents with chronic diarrhea since 1 year with abdominal pain. Stool sample reveals

100 micron ova in intestine which of the following can be the causative organism A/E

(a) G. discoidis not recalled

(b) Fasciola Gigantica

(c) O. vivireni

(d) Not recalled

 

Answer : Unattempted

Explanation :

O. vivireni is i liver fluke that causes cholangitis. The cyst size is 11—30mu.m.

1. Ichhpeujann, 3rd Ed., Pg. 253.

Q 254.  Carisoprodol activation forms which metabolite

(a) Amphetamine

Page 100: Elsevier Jan

(b) Meprobamate

(c) Doxylamine

(d) Dimethadione

Answer : Unattempted

Explanation :

a. Carisoprodol is a centrally-acting skeletal muscle relaxant.

b. Carisoprodol is a colorless, crystalline powder, having a mild characteristic odor and a bitter taste.

c. It is slightly soluble in water and freely soluble in alcohol, chloroform and acetone.

d. Its solubility is practically independent of pH.

e. It is marketed in the United States under the brand name Soma, and in the United Kingdom and other

countries under the brand names Sanoma andCarisoma.

Carisoprodol is available by itself or mixed with aspirin and in one preparation (Soma Compound With

Codeine) along with codeine and caffeine as well.

g. Carisoprodol has a rapid, 30 minute onset of action, with the aforementioned effects lasting for

approximately 2–6 hours.

h. It is metabolized in the liver via the cytochrome P450 oxidase isozyme CYP2C19, excreted by the

kidneys and has an approximate 8 hour half-life.

A considerable proportion of carisoprodol is metabolized to meprobamate, which is a known drug of

abuse and dependence.

1. Various Internet Sites

Q 255.  Which of the following is true regarding phenytoin

(a) Follows zero order kinetics

(b) No interaction with any other drug

(c) Not teratogenic

(d) Excreted unchanged

 Answer : Unattempted

Explanation :

a. Phenytoin (diphenylhydantoin, DILANTIN) is effective against all types of partial and tonic-clonic

seizures but not absence seizures

b. Phenytoin exerts antiseizure activity without causing general depression of the CNS. In toxic doses, it

may produce excitatory signs and at lethal levels a type of decerebrate rigidity.

c. This effect is mediated by a slowing of the rate of recovery of voltage- activated Na+ channels from

inactivation, an action that is both voltage-(greater effect if membrane is depolarized) and use-

dependent.

d. The effects on Na+ channels are selective, and no changes of spontaneous activity or responses to

iontophoretically applied GABA or glutamate are detected.

e. At concentrations five- to tenfold higher, multiple effects of phenytoin are evident, including reduction

Page 101: Elsevier Jan

of spontaneous activity and enhancement of responses to GABA; these effects may underlie some of

the unwanted toxicity associated with high levels of phenytoin.

The pharmacokinetic characteristics of phenytoin are influenced markedly by its binding to serum

proteins, by the nonlinearity of its elimination kinetics, and by its metabolism by cyps.

g. Phenytoin is extensively bound (about 90%) to serum proteins, mainly albumin.

h. Some agents, such as valproate, can compete with phenytoin for binding sites on plasma proteins;

when combined with valproate-mediated inhibition of phenytoin metabolism, marked increases in free

phenytoin.

Phenytoin is one of the few drugs for which the rate of elimination varies as a function of its

concentration (i.e., the rate is nonlinear).

The plasma half-life of phenytoin ranges between 6 and 24 hours at plasma concentrations below 10

μg/ml but increases with higher concentrations;

k. As a result, plasma drug concentration increases disproportionately as dosage is increased, even

with small adjustments for levels near the therapeutic range.

The majority (95%) of phenytoin is metabolized principally in the hepatic endoplasmic reticulum by

CYP2C9/10 and to a lesser extent CYP2C19.

m. The principal metabolite, a parahydroxyphenyl derivative, is inactive. Because its metabolism is

saturable, other drugs that are metabolized by these enzymes can inhibit the metabolism of phenytoin

and increase its plasma concentration.

n. Conversely, the degradation rate of other drugs that are substrates for these enzymes can be

inhibited by phenytoin; one such drug is warfarin, and addition of phenytoin to a patient receiving

warfarin can lead to bleeding disorders.

o. An alternative mechanism of drug interactions arises from phenytoin's ability to induce diverse cyps.

p. Coadministration of phenytoin and medications metabolized by these enzymes can lead to an

increased degradation of such medications.

q. Of particular note in this regard are oral contraceptives, which are metabolized by CYP3A4.

Treatment with phenytoin could enhance the metabolism of oral contraceptives and lead to unplanned

pregnancy. The potential teratogenic effects of phenytoin

s. The low aqueous solubility of phenytoin hindered its intravenous use and led to

production offosphenytoin, A water-soluble prodrug.

Toxicity

a. Emergency treatment of status epilepticus, the most notable toxic signs are cardiac arrhythmias, with

or without hypotension, and/or CNS depression. Although cardiac toxicity occurs more frequently in

older patients and in those with known cardiac disease, it also can develop in young, healthy patients.

These complications can be minimized by administering fosphenytoin

b. Acute oral over dosage results primarily in signs referable to the cerebellum and vestibular system;

c. High doses have been associated with marked cerebellar atrophy

d. Toxic effects associated with chronic treatment also are primarily dose-related cerebellar-vestibular

effects but also include other CNS effects,

e. Behavioral changes

Increased frequency of seizures

Page 102: Elsevier Jan

g. Gastrointestinal symptoms,

h. Gingival hyperplasia,

Osteomalacia, and

Megaloblastic anemia.

k. Hirsutism is an annoying untoward effect in young females

Serious adverse effects, including those on the skin, bone marrow, and liver, probably are

manifestations of drug allergy

m. Moderate elevation of the plasma concentrations of hepatic transaminases sometimes are observed;

n. Gingival hyperplasia occurs in about 20% of all patients during chronic therapy and is probably the

most common manifestation of phenytoin toxicity in children and young adolescents.

o. It may be more frequent in those individuals who also develop coarsened facial features.

1. KATZUNG 10th Ed., Pg. 357.

Q 256.  Anti TNF alpha is not used is

(a) RA with arthritis

(b) RA with HIV

(c) RA with HBV

(d) RA with pulmonary fibrosis

Answer : Unattempted

Explanation :

a. Recently, biologic agents that bind and neutralize TNF have become available

1. One of these is a TNF type II receptor fused to IgG1 (etanercept),

2. The second is a chimeric mouse/human monoclonal antibody to TNF (infliximab), and

3. The third is a fully human antibody to TNF (adalimumab).

b. These agents not only are effective in persistently controlling signs and symptoms of RA in a majority

of patients, but they have also been shown to slow the rate of progression of joint damage assessed

radiographically and to improve disability.

c. Particularly notable is the capacity of TNF blockade to

d. Increase the risk of developing reactivation of dormant tuberculosis.

e. It is prudent to carry out tuberculin skin testing and, if necessary, further evaluation with chest

radiographs before beginning therapy with an anti-TNF agent to limit the chance of inciting

reactivation of tuberculosis

Anti-TNF therapy also has the potential to increase the risk of lymphoma and possibly other

malignancies in treated patients.

g. TNF-neutralizing therapy can also induce the development of anti-DNA antibodies, but rarely is there

associated evidence of signs and symptoms of systemic lupus erythematosus

h. Development of antibodies to infliximab (ATI) is associated with an increased risk of infusion reactions

and a decreased response to treatment.

Acute infusion reactions

Page 103: Elsevier Jan

Severe serum sickness

k. Rarely, infliximab has been associated with optic neuritis,

Seizures,

m. Radiographic evidence of central nervous system demyelinating disorders, including multiple

sclerosis.

n. It may exacerbate symptoms in patients with New York Heart Association functional class III/IV heart

failure.

1. Harrison's 17th Ed., Pg. 1897, 2091.

Q 257.  HLAB27 is positive in

(a) Ankylosing spondylosis

(b) RA

(c) SLE

(d) Sjögren syndrome

 Answer : Unattempted

Explanation :

Significant HLA class I and class II associations with disease

Page 104: Elsevier Jan

1. Harrison's Principle of Internal Medicine, 18th Ed., Pg. 2051.

Q 258.  Presence of nephroblastomatosis in a biopsy secimen from wilms tumor indicates

(a) Denysh drash syndrome

(b) Mutation in insulin like growth factor

Page 105: Elsevier Jan

(c) Tumor may spread to contralateral kidney

(d) It is a predisposing factor for development of wilms tumor

Answer : Unattempted

Explanation :

a. Nephroblastomatosis refers to abnormal persistence of embryonal renal tissue.

b. Favourable survival and response rate of bilateral wilms tumor suggests a strong relation with this

c. It maintains the potential for malignant induction to wilm's tumor

d. May be perilobar (MC) or intra lobar

e. Treated by chemotherapy

1. Nelson, 18th Ed., Pg. 2143.2. Chapmen 5th Ed., Pg. 368.

Q 259.  Which of the following is example of instability of chromosome

(a) Ataxia telengectasia

(b) Downs syndrome

(c) Kliffel feil syndrome

(d) Not recalled

 Answer : Unattempted

Explanation :

Ataxia-Telangiectasia

a. Ataxia-telangiectasia (AT) is an autosomal recessive genetic disorder characterized by cerebellar

ataxia, oculocutaneous telangiectasia, and immunodeficiency.

b. The mutant ATM gene has sequence similarity to the phosphatidyl-inositol-3 kinases that are involved

in signal transduction.

c. The ATM gene belongs to a conserved family of genes that monitor DNA repair and coordinate DNA

synthesis with cell division.

d. The deleterious effects of the ATM gene are widespread. Truncal ataxia may become evident when

walking begins and is progressive.

e. Telangiectasia, primarily represented by dilated blood vessels in the ocular sclera, in a butterfly area

of the face, and on the ears, is an early diagnostic feature. Immunodeficiency may be clinically

manifest by recurrent and chronic sinopulmonary infection leading to bronchiectasis, although not all

patients have overt immunodeficiency.

Ovarian agenesis is a frequent occurrence.

g. Persistence of very high serum levels of oncofetal proteins, including-fetoprotein and

carcinoembryonic antigen, may be of diagnostic value.

h. Frequent causes of death are chronic pulmonary disease and malignancy. Lymphomas are most

common, although carcinomas also occur.

The immunologic abnormalities seem to be related to maldevelopment of the thymus.

The markedly hypoplastic thymus is similar in appearance to an embryonic thymus.

Page 106: Elsevier Jan

k. The peripheral T cell pool is reduced in size, especially in lymphoid tissue compartments. Cutaneous

anergy and delayed rejection of skin allografts are common.

B lymphocyte development is normal, most patients are deficient in serum ige and IgA, and a smaller

number have reduced serum levels of IgG, particularly of the IgG2, IgG4 subclasses.

m. The defect in DNA repair mechanisms in AT patients renders their cells highly susceptible to

radiation-induced chromosomal damage and resultant tumor development.

1. Harrison's Principle of Internal Medicine, 18th Ed., Pg. 2058.

Q 260.  Which of the following is true regarding NHL

(a) BCL6 is associated with Burkitt Lymphoma

(b) BCL2 is associated with mantle cell lymhoma and follicular lymhoma

(c) CD 34 associated with DPFL

(d) CD 10 is associated with mantle cell lymphoma

Answer : Unattempted

Explanation :

Burkitt Lymhoma

a. Tumor of B cell lineage

b. Express surface IGM

c. Monotypic may be K or Lamda type

d. CD 10, CD 19, CD20+

e. BCL6 +

C-MYC +translocation pattern IGH locus T(8;14), KT (2;8). Or lamda t(8;22)

Mantle Cell Lymhoma

a. CD 19, CD 20 +

b. With high level surface IGM or IGD either Kappa or Lamda type

c. CD5+

d. CD23−

e. Cyclin d1 protein+

T(11;14)

Follicular Lymphoma

a. CD 10, CD 19, CD20+

b. CD5−

c. BCL2+

d. BCL6+T(14;18)

Page 107: Elsevier Jan

1. Robbins Pathology, 7th Ed., Pg. 676.

Q 261.  Abrasions can be confused with

(a) Eczema

(b) Chemical injury

(c) Ant bite

(d) Joules burn

 

Answer : Unattempted

Explanation :

Abrasion is a wound caused by superficial damage to the skin, not deeper than the epidermis. It is

less severe than a laceration. Mild abrasions, also known asgrazes or scrapes, do not scar or bleed,

but deep abrasions may lead to the formation of scar tissue. A more traumatic abrasion that removes

all layers of skin is called an avulsion.Colloquially, abrasions caused by contact with textiles or

carpeting are referred to as rug burn or carpet burn. In vehicle accidents where the skin contacts the

road surface, it is known as road rash. Slipping on ropes or other surfaces is known as rope burn or

friction burn.Age of abrasions:Fresh—bright redIn around 12 to 24 hours—lymph and blood dries up

leaving a bright scab2 to 3 days-reddish brown scabAround 4 to 7 days—epithelium grows up and

covers defect under the scabAfter 7 days, scab dries, shrinks, and falls out.References1. The

Essentials of Forensic Medicine and Toxicology, Reddy (23rd Ed.), Pg. 145; (22nd Ed.), Pg.

151.2. Parikh's Textbook of Medical Jurisprudence, Forensic Medicine and Toxicology (6th Ed.), Pg.

43.3. Simpson's Forensic Medicine, 1997, Pg. 81.

Q 262.  Heat rupture due to high temperature is called

(a) Regular rupture

(b) Irregular rupture

(c) Blood clotted

(d) Vessels rupture

Answer : Unattempted

Explanation :

High temperature can cause rupture of the superficial skin due to the hardening of the subcutaneous

fat and it may resemble incised or lacerated wound.Heat rupture and the incised wound can be

differentiated by:Heat rupture has clear marginsHeat rupture is more superficialHeat rupture is seen in

antemortem burnsMultiple small wound involves multiple areas of bodyVessels can be seen running

along the base of heat rupture.References1. The Essentials of Forensic Medicine and Toxicology,

Reddy (22nd Ed.), Pg. 143; (23rd Ed.), Pg. 283.

Q 263.  False about H. Inflenza

(a) It needs factor X and Y

Page 108: Elsevier Jan

(b) Peptide capsule plays an important role in pathogenesis

(c) Rarely causes meningitis in child before 2 year

(d) Mc Invasive form is meningitis

 

Answer : Unattempted

Explanation :

a. The bacterium is a small (1- by 0.3-m) gram-negative organism of variable shape; it is often described

as a pleomorphic coccobacillus

b. H. Influenzae grows both aerobically and anaerobically.

c. Its aerobic growth requires two factors: hemin (X factor) and nicotinamide adenine

dinucleotide (V factor).

d. These requirements are used in the clinical laboratory to identify the bacterium

e. Six major serotypes of H. Influenzae have been identified; designated athrough f, they are based on

antigenically distinct polysaccharide capsules.

In addition, some strains lack a polysaccharide capsule and are referred to as nontypable strains

g. Type B and nontypable strains are the most relevant strains clinically

h. The most serious manifestation of infection with Hib is meningitis

The age of peak incidence varies somewhat among populations, depending in part on the use of

vaccine, but this infection primarily affects infants ≪2 years of age.

The clinical manifestations of Hib meningitis are similar to those of meningitis caused by other

bacterial pathogens.

k. Fever and altered central nervous system function are the most common features at presentation.

Nuchal rigidity may or may not be evident.

Subdural effusion, the most common complication, is suspected when, despite 2 or 3 days of

appropriate antibiotic therapy

1. Harrison's 17th Ed., Pg. 923.

Q 264.  About widal test A/E are true

(a) A single high titre is not enough for conclusive diagnosis

(b) O antibody last longer and hence is not indicative of recent infection

(c) Baseline titre differs depending on endemicity of disease

(d) H antigen can not differentiate between subtypes

Answer : Unattempted

Explanation :

a. Widal test is a test for measurement of H and O aglutinins for typhoid and Paratyphoidbacilli in the

patient sera

b. Two types of Tube are generally used for the test a narrow tube with conical bottom (Dreyer's tube)

for H Agglutinogen. H Agglutination forms loose Cotton wooly clumps.

c. A short round bottomed tube (Felix) for O agglutination. It forms disc like pattern at the bottom of the

Page 109: Elsevier Jan

tube

d. Antigen used to test are H and O Antigen of S. Typhi and H Antigen of S. Paratyphi A and B

e. The Paratyphoid O Antigens are not employed as they cross react with Thphoid O Antigen due to

sharing of factor 12

The strain used are S. Typhi 901, O and H antigen

g. Results of Widal test are Interpreted as follows:

1. The agglutination titres will depend on stage of the disease

2. Agglutinin will appear by end of 1st week

3. The titres will increase steasily till the 3rd to 4th week after which they will decline

4. Demonstration of rising titre of antibodies by testing two or more serum samples is more meaningful

then a single test

5. Titre of 1/100 or more for O Agglutinins and 1/200 or more for H Aggutinins are significant

6. Agglutinins may be present on account of prior disease, in apparent infection or immunisation

7. H agglutinins persists longer than O agglutinins

8. Serum from an individual immunised with tab vaccine will generally have antibodIes to S. Typhi and

S. Parathyphi

9. In case of infection antibodies will be seen only against the infecting species

10. Person who had prior infection or immunisation may develop anamnestic reaction during an

unrelated fever

11. This can be differentiated by repetation of test after a week

12. Anamnestic reaction is sustained in case of enteric fever

13. Cases treated with chloramphenicol will show poor agglutinin response

1. Anant 8th Ed., Pg. 296.

Q 265.  Necrotising Hystiocytic Lymphadinitis on Histopathology is seen in:

(a) Castelmans Disease

(b) Kikuchi's Disease

(c) Kimura Disease

(d) Hodgkins Disease

 

Answer : Unattempted

Explanation :

It is a differential diagnosis of tubercular lymphadenitis

1. Harrison's 17th Ed., Pg. 1011.

Q 266.  Effective leprosy control programme includes all except:

(a) Decreased disability rate

(b) Increased number of new children

Page 110: Elsevier Jan

(c) High new case detection

(d) Proportion of multibacillary case onto treatment

Answer : Unattempted

Explanation :

Two main types of indicators in leprosy control

Operational Indicator

a. Primarily managerial in character

b. Monitor function of ongoing control activities in the field

1. Related to case finding

2. Treatment

3. Relapses

4. Diability

c. Relapse rate is the best indicator of the efficacy of drug regimen

d. Case detection ratio (the number of cases registered to estimate number of cases)

e. Proportion of children (0–14 year) among newly detected cases

Proportion of multibacillary cases on regular treatment during the year

g. Proportion of female cases among the newly detected cases

h. Proportion of new cases presenting with grade 2 disability/impairment at the time of diagnosis

Proportion of treatment defaulters

Treatment completion /cure rate

k. Proportion of patients who develop new/additional disability during MDT

Epidermiological Indicators

Required to evaluate the effectiveness of the programme that is to access the impact of the action

taken with regard to problem reduction

Incidence:

a. Taken to calculate for different subgroups of the population

b. It is the most sensitive index of transmission of disease

c. Only index for measuring effectiveness of the measure taken

d. They are useful in monitoring the success of control of programme

Prevalance

a. Measure of the case load

b. Useful in the planning of the treatment of services

c. It is useful in calculating prevalance rates of different subgroups

1. Park 20th Ed., Pg. 289.

Page 111: Elsevier Jan

Q 267.  Not under NRHM

(a) Strengthening institutional delivery under Janani Surakhshya Yojana

(b) Constructs health and family welfare society

(c) Appointment of new ASHA to facilitate access to health service

(d) Constructs state and district health mission

 

Answer : Unattempted

Explanation :

Plan of action to strengthen infrastructure

a. Creation of cadre of accredited social health activist (ASHA)

b. Strengthening sub centres

c. Strengthening primary health centres

d. Strengthening of community centres for first referral unit (promotion of Rogi Kalyan Samiti for hospital

management)

e. District becomes the core unit of planning, budgeting and implementation of the programme.

All vertical health and family welfare programme at district level will merge into one common “district

health mission” and at the state level into “state health mission”

g. There will be provision of “mobile medical unit” at the district level for improved outreach service

Janani Suraksha Yojana: Launched in 12th April 2005, 100% centrally sponsored scheme, comes in

RCH phase II also included in NRHM.

Objectives of the Scheme

a. Reducing maternal mortality and infant mortality through encouraging delivery at health institutions

b. Focussing at the institutional care among woman in below poverty line families

Also know about Vandemataram Scheme which also comes under RCH II:

a. This is a voluntary scheme wherein any obstetric and gynae specialist, maternity home, nurshing

home, lady doctor/MBBS doctor can volunteer themselves for providing safe motherhood service

b. The enrolled doctor will display Vandemataram Logo at the clinic

c. Iron, folic acid tablets, oral pills, TT injection etc are provided by the respective district medical officers

to the vandemataram doctors /clinic for free distribution to beneficiaries

1. Park 20th Ed., Pg. 379; www.mohfw.nic.in/NRHM

Q 268.  About standard error of mean false is

Page 112: Elsevier Jan

(a) Increases with sample size

(b) Based on normal distribution

(c) Used to determine confidence limit

(d) Acurate estimation of standard deviation

Answer : Unattempted

Explanation :

Standard error of mean = standard deviation/square root of mean of sample

If sample size increases square root of mean of sample will increase so standard error of menan will

decrease.

1. Park 20th Ed., Pg. 753.

Q 269.  About growth chart false is

(a) Space between 2 lines defines road to health

(b) Dots position on graph is more important then direction

(c) Lowest line equals to 3rd percentile

(d) Develops tool for educating mother

 

Answer : Unattempted

Explanation :

a. Growth chart is a visible display of the childs growth and development

b. WHO growth chart has 2 reference curve

1. Upper reference curve represents the media (50 percentile) for boys, slightly higher for girls

2. Lower reference curve the 3rd percentile for girls slightly lower than that for boys

c. The chart can be used for both sexes

d. The space between the two growth curves (weight channel) has been called as road to health

e. This will include the zone for most population

If the child is growing normally its growth line will be above 3 rd percentile and will run parallel to the

road to health curve

g. Its direction of the curve that is more important than dots on the line

h. Flattening of the curve signals growth failure the objective in child care is to keep the child above 3rd

percenile

1. Park 20th Ed., Pg. 468

Q 270.  10000 person in a community 20% have disease with sensitivity of the test 95% and specificity

80% What will be the positive predictive value

(a) 54.3

Page 113: Elsevier Jan

(b) 45.7

(c) 98.5

(d) 15.3

Answer : Unattempted

Explanation :

Formula for positive predictive value

PPV = Sensitivity Prevalence × sensitivity/(prevalance × sensitivity) + (1-prevalance) × (1-

specificity)

Q 271.  Patient presents with eye opening with painful stimuli, inappropriate words, localize pain. What

is the score glasgow coma scale

(a) 8

(b) 10

(c) 12

(d) 14

 

Answer : Unattempted

Explanation :

Glasgow coma scale

1. Harrison's 17th Ed. Pg. 1674.

Q 272.  All of the following are true about choledochal cyst except

(a) Type 2 is most common

Page 114: Elsevier Jan

(b) Danger of rupture and causing biliary peritonitis

(c) Associated with pancreatic duct anomalies

(d) Mostly present in the infancy

Answer : Unattempted

Explanation :

Choledochal cysts are congenital conditions associated with benign cystic dilatation of bile ducts.

Classic triad of intermittent abdominal pain, jaundice, and a right upper quadrant abdominal mass is

found only in minority of patients. Most of them present in first year of life; adult presentation is rare and

usually at this stage is associated with complicationClassification based on site of the cyst or

dilatation.Type I: Most common variety involving saccular or fusiform dilatation of a portion or entire

common bile duct (CBD) with normal intrahepatic duct.Type II:Isolated diverticulum protruding from the

CBD.Type III or Choledochocele: Arise from dilatation of duodenal portion of CBD or where

pancreatic duct meets.Type IV: Dilatation of both intrahepatic and extrahepatic biliary duct.Type V or

Caroli's disease: Cystic dilatation of intrahepatic biliary ducts.Choledochal cysts are treated by

surgical excision of the cyst with the formation of a roux-en-Y anastomosis to the biliary duct. Future

complications include cholangitis and a 2% risk of malignancy, which may develop in any part of the

biliary tree.1. CSDT. 12th Ed., Pg. 313.

Q 273.  Regarding flail chest which of the following is false

(a) At least 3 rib fracture with fracture at two sites bilaterally

(b) Should be ventilated with fiO2 >60% when po2 decreases to 40%

(c) Paradoxical movement may not be found in conscious patient

(d) If severely overlapping is seen open surgery and fixation is done

 Answer : Unattempted

Explanation :

a. By definition, a flail chest occurs in the presence of two or more fractures in three or more

consecutive ribs and causes instability of the chest wall; however, it can also occur after

costochondral separation.

b. Flail chest is characterized by paradoxical motion of the chest wall (inward with inspiration and

outward with expiration).

c. Fractures can be located in the anterior, lateral, or posterior chest wall.

d. Flail chest occurs in 10–15% of patients sustaining major chest trauma, and the chance of having an

intrathoracic injury in this situation increases several-fold.

e. Closed head injury is the most frequently associated extrathoracic injury, and it contributes to higher

morbidity and mortality rates. Isolated flail chest carries a low mortality risk in younger patients.

The paradoxical motion increases the work of breathing, and the most important consequence of flail

chest is respiratory failure.

g. Underlying pulmonary contusion and pain during inspiration are the most important components in the

pathophysiology of the respiratory failure.

h. Sequential measurements of forced vital capacity, tidal volume, and inspiratory force are useful to

Page 115: Elsevier Jan

predict which patients will require ventilatory support.

The pathophysiologic effects may be present immediately or may progress over a period of several

hours and be manifested as late respiratory decompensation.

Care should be taken to not overload these patients with fluid because respiratory function may be

impaired even further.

1. Bailey and Love 25th Ed., Pg. 343.

Q 274.  Orchidopexy is done at the age of

(a) Neonate

(b) 1–2 year

(c) Before puberty

(d) 5–6 year

Answer : Unattempted

Explanation :

Anorchia and cryptorchidism:

a. A 46, XY male without palpable testes may have intra abdominal testes, which carry an increased risk

of malignant degeneration, anorchia (the “vanishing testes syndrome”) in which no testes are found at

laparotomy or laparoscopy, or retractile testes, a variation of normal.

b. Cryptorchidism is the most common abnormality of male newborns.

c. F there is a male phenotype and male internal ducts; functioning fetal testes capable of secreting

testosterone and antimüllerian hormone were present early during fetal life but degenerated

thereafter. Administration of 2000 microgram of HCG (3000 u/m2) intramuscularly usually evokes an

increased concentration of plasma testosterone after 72 hours when functional leydig cells are

present.

d. The lack of a rise in testosterone concentration, in conjunction with an increased plasma

concentration of FSH and LH or an augmented gonadotropin response to GnRh, is evidence for the

diagnosis of bilateral anorchia.

e. Alternatively, measurement of antimüllerian hormone will indicate the presence of testicular tissue in a

range of suspected conditions in prepubertal boys from presumed anorchia to male

pseudohermaphrodism and true hermaphrodism.

Serum inhibin B is a useful indicator of the presence of functional testicular tissue; values were

correlated with the testosterone response to HCG administration, values less than 15 pg/ml indicated

anorchia.

g. Cryptorchid testes may descend into the scrotum during more prolonged treatment with HCG.

h. Such descent occurs in retractile testes, it can occur in true cryptorchid testes if local anatomic factors

do not inhibit descent.

Recent studies suggest that hormonal treatment may decrease the number of spermatogonia

compared to individuals who had only orchiopexy.

Page 116: Elsevier Jan

While not yet contraindicated, HCG or GnRh treatment is not recommended for the treatment of

cryptorchidism.

k. Normally testicular descent has occurred by 1 year of age; although later descent is described, the

incidence is low.

Orchidopexy is recommended between 12 and 18 months of age in those testes not expected to

descend spontaneously.

m. Two critical steps in the maturation of germ cells are described in the normal prepubertal testis that do

not occur in the unilaterally undescended testes:

1. At 2 to 3 months of age, the gonocytes (primitive spermatogonia) in the fetal stem cell pool

transform into the adult dark spermatogonia, which become the adult stem pool (possibly related to

the early infancy surge in LH, FSH, and testosterone

2. At 4 to 5 years of age, meiosis begins and primary spermatocytes begin to appear.

n. The contralateral descended testis is affected as well but less so than the undescended testis.

o. Postpubertal orchidopexy is associated with a high (>85%) prevalence of azoospermia or

oligospermia.

p. Cryptorchid testes, even if replaced in the scrotum, may never have normal spermatogenic function

as a consequence of an early abnormality in germ cell maturation, vascular damage to the testicular

circulation during orchidopexy, or an intrinsic testicular defect.

q. Early orchidopexy reduces the risk of carcinoma of the testes

Although dysgenetic testes, even if located in the scrotum, carry an increased risk of malignant

transformation.

s. Undescended testes remain at a higher temperature than descended testes and undescended testes

have a maturation arrest at the conversion of the gonocyte to the spermatogonia; this appears to

direct the testes toward malignant degeneration.

One year is a useful age to consider orchiopexy for undescended testes as it is an age at which the

likelihood of spontaneous descent lessens but the benefits to the testes of orchiopexy remain.

u. Methods of orchidopexy:

1. Transabdominal Orchidopexy

2. Fowler-Stephens orchidopexy

3. Staged Fowler-Stephens orchidopexy

1. Williams Textbook of Endocrinology, 11th Ed.2. Campbell-walsh urology, 9th Ed. chapter 118.

Q 275.  Method for diagnosing whooping cough in child

(a) Tracheal aspirate

(b) Cough plate method

(c) Sputum smear

(d) Nasopharyngeal swab

 Answer : Unattempted

Explanation :

Page 117: Elsevier Jan

a. Culture of nasopharyngeal secretions remains the gold standard of diagnosis.

b. Although DNA detection by polymerase chain reaction (PCR) is replacing culture in many laboratories

because of increased sensitivity and quicker results.

c. The best specimen is collected by nasopharyngeal aspiration, in which a fine flexible plastic catheter.

d. Is passed into the nasopharynx.

e. Since B. Pertussis is highly sensitive to drying, secretions for culture should be inoculated without

delay onto appropriate medium (Bordet-Gengou or Regan-Lowe), or the catheter should be flushed

with a phosphate-buffered saline solution.

For culture and/or PCR. An alternative to the aspirate is a Dacron or rayon nasopharyngeal swab;

again, inoculation of culture plates should be immediate or an appropriate transport medium (e.g.,

Regan-Lowe charcoal medium) should be used.

1. Harrison's 17th Ed., Pg. 935.2. CPDT 19th Ed. Pg. 1165.

Q 276.  Hypertrophic pyloric stenosis is associated with

(a) Hypokalemic metabolic alkalosis

(b) Hypokalemic metabolic acidosis

(c) Hyerkalemic metabolic alkalosis

(d) Hyperkalemic metabolic alkalosis

Answer : Unattempted

Explanation :

1. CPDT 19th Ed., Pg. 580.

Q 277.  About Kostmann syndrome treatment of choice is

(a) Atg+cyclosporin

(b) Atg+cyclosporin+GMCSF

(c) Testosterone+GMCSF

(d) GMCSF

 Answer : Unattempted

Explanation :

Kostmann syndrome

a. Type of congenital neutropenia

b. Due to abnormality of antiapoptotic gene HAX1 and the elastage gene

ELA 2 treated by recombinant G-CSF OR GMCSF

c. Prophylactic antibiotic therapy not indicated

d. Can progress to leukemia

Page 118: Elsevier Jan

1. CPDT 19th Ed., Pg. 827.

Q 278.  Reflex hallucination is a morbid type of

(a) Kinesthesia

(b) Paresthesia

(c) Synesthesia

(d) Hyperesthesia

Answer : Unattempted

Explanation :

Synesthesia is when a veridical perception in one modality produces a hallucination in another, for

example, seeing a doctor writing (visual) and then feeling him writing across one's stomach

(tactile).Synesthesia is the experience of a stimulus in one sense modality producing a sensory

experience in another. For example feeling of cold in one's spine on hearing a fingernail scratch on

blackboard. some hallucinogenic drugs like LSD, Mescaline can cause it.Reflex Hallucination is a

morbid form of synesthesia. In a reflex hallucination, a stimulus in one sensory field produces a

hallucination in another, for example, a patient felt pain in his head (somatic hallucination) while she

heard other person sneeze (the stimulus) and was convinced that sneezing causes the

pain.Neurologist Richard Cytowic identifies the following diagnostic criteria of synesthesia:Synesthesia

is involuntary and automatic.Synesthetic perceptions are spatially extended, meaning they often have a

sense of “location.” For example, synesthetes speak of “looking at” or “going to” a particular place to

attend to the experience.Synesthetic percepts are consistent and generic (i.e., simple rather than

pictorial).Synesthesia is highly memorable.Synesthesia is laden with affect.

Q 279.  6 year old child presents with boggy swelling and discharging sinus from several orifice with

pluck able hair. Multiple lymph nodes swelling present in occipital region diagnosis can be

established by

(a) Bacterial culture

(b) Biopsy

(c) KOH

(d) Patch test

 Answer : Unattempted

Explanation :

It is tinea capitis infection of child

It can be diagnosed by

a. KOH scrapping: produce branching hyphae, endothrix/exothrix

b. Fungal culture

c. Woods light: shine woods, light used (a type of UV light) green fluroscence produced

Page 119: Elsevier Jan

1. Neena Khanna 3rd Ed., Pg. 247.

Q 280.  6 year old child presents with multiple flaccid bulla on trunk and few erosions on buccal

mucosa. DFA will show

(a) Fishnet IgG in epidermis

(b) Linear IgG on dermoepidermal junction

(c) Linear IgA on dermoepidermal junction

(d) Granular IgA in dermal papilla

Answer : Unattempted

Explanation :

Pemphigus

a. Age: middle age may involve children

b. No gendre prevelance

c. Flaccid bulla develops on normal skin

d. Nikolky sign positive

e. Bulla spread sign positive

Lesions predominently present on face, groin, axilla, trunk

g. 50% patients lesions begin in oral mucosa

h. Presence of acantholytic cells

DIF shows intracellular deposits of IgG giving fish net appearance in epidermis

Tzank smear positive

Bullous pemphigoid

a. Autoimmune disorder

b. Itchy, tense, persistent bullae, some times large hemorhagic bulla on normal skin or on erythematous,

edematous base

c. Bulla usually does not rupture

d. Associated with urticarial plaque

e. Healing with milia formation

Infrequent oral lesions

g. Age prediction 60–80 years

h. Equal incidence in males and females

Bulla spread and nikolsky sign negative

Self limiting condition

k. Involve lower part of body

DIF shows linear deposit of c3 and IGG along the dermoepidermal junction

Chronic bulous disorder of childhood

Page 120: Elsevier Jan

a. Age below 5 years

b. Slight female preponderance

c. Itchy

d. Tense bulla sometimes on erythematous base

e. String of pearl appearance

Lesions are often grouped

g. Occurs around orifices

h. Also involve lower abdomen knees and elbows

DIF shows linear deposit of IGA at dermoepidermal junction

Dermatitis herpetiformis

a. Gluten sensitive enteropathy is always present

b. Age prediction 20–60 year

c. Male to female ratio 2:1

d. Extremly itchy

e. Show microabcess on tip of dermal papilla

DIF shows granular deposit of IGA in the tip of dermal papilla

1. Neena Khanna 3rd 63.

Q 281.  Rate of mineralization of newly formed osteoid can be best estimated by the following

(a) von Kossa staining for calcium

(b) Alzarin red stain

(c) Labeled tetracycline

(d) Immunofluorescence

 

Answer : Unattempted

Explanation :

Tetracycline labeling methods have been used to measure the rate of growth in length and the rate of

growth in width of the tibia of the normal rat.The main limitations of the tetracycline labeling methods

are that in very young animals the bands of labeled bone are indistinct and remodeling occurs quickly;

in animals nearing maturity, the growth in width is very slow and periods of at least 14 days are

required to give reliable results.The tetracycline labeling methods can be used also to determine

changes in the basic processes of bone formation and bone resorption.Tetracycline when administered

in vivo becomes fixed in new forming mineralizing bone and exhibits a characteristic fluorescence

when viewed by ultraviolet light. In this manner, new bone formation can be accurately

measured.Tetracycline labeling is a method of determining the effect of various factors on bone

growth.References1. Turek's Orthopaedics (4th Ed.), Pg. 178.

Page 121: Elsevier Jan

Q 282.  Allopurinol used in organ preservation as

(a) Antioxidant

(b) Free radical destruction

(c) Preservative

(d) None of the above

Answer : Unattempted

Explanation :

AIM of preservation – To minimize cold ischemic injury by inhibiting cell swelling; providing high

energy substrates, reducing ROS formation on reperfusion.Solution may be hypertonic or contain

lactobionate and raffinose to prevent cell swelling; Adenosine and GSH are added to aid ATP

generation;Allopurinol is added to inhibit formation of ROS, has been shown to be protective in

cadaveric renal and liver transplantation; Calcium channel blockers and flushing of organs before

implantation have been utilized to limit ROS generation; have shown to reduce its I/R injury.Most

common solution used is university of Wisconsin (UW) solution.Contents – Lactobionate,

Raffinose, and Adenosine (a precursor of ATP generation).Magnesium sulfate (to retard calcium

influx).Glutathione (as an antioxidant).Allopurinol (to inhibit Xanthine oxidase reducing ROS

formation).Recently added to it – prostaglandin E1; SOD conjugation to ethylene glycol; free radical

scavenger N-acetyl cysteine and NO donor Na Nitroprusside – resulted significant decreased graft

rejection.1. David S. Wilkes, William J. Burlingham. Immunobiology of Organ Transplantation, Pg. 558.

Q 283.  Radiotherapy is the primary modality of treatment in

(a) T3N0 glottic cancer

(b) T3N1 nasopharyngeal cancer

(c) T3N1 supraglottic cancer

(d) T3N0 subglottic

 Answer : Unattempted

Explanation :

Irradiation is treatment of choice in all stage of nasopharyngeal cancer

Glottic cancer

Carcinoma in situ: transoral endoscopic CO2 laser

a. T1 carcinoma: radiotherapy

b. T1: carcinoma

a. With extension to anterior commisure : radiotherapy

b. In the absence of extension to anterior commisure frontolateral laryngectomy is done with regular

follow up

c. If it fails total laryngectomy is performed

Page 122: Elsevier Jan

T2n0 carcinoma:

a. If cord is mobile and anterior commisure not involved :radiotherapy

b. If tumor recurs: total laryngectomy

c. If anterior commisure involved /arytenoid involved/vocal cord mobility impaired: vertical laryngectomy

or fronto lateral laryngectomy done

T3/T4 carcinoma : total laryngectomy

Sub glottic cancer

a. T1/T2 carcinoma: radiotherapy

b. T3/T4 carcinoma: total laryngectomy with post op radiotherapy

Supraglottic cancer

a. T1 carcinoma :radiotherapy

b. T2: carcinoma supraglottic laryngectomy with or without neck dissection

c. T3/T4 carcinoma : total laryngectomy with neck dissection and post op radiotherapy.

1. PL Dhingra 4th Ed., Pg. 235.

Q 284.  Caustic poison erodes mucosa because

(a) Hygroscopic in nature

(b) It has glue like action

(c) It is being programmed to stick

(d) It has affinity for mucosa

Answer : Unattempted

Explanation :

a. Caustic action is due to local action on alimentary tract or respiratory tract

b. No systemic action except shock

Their action is characterized by

1. Extraction of Water from the Tissue (hygroscopic action)

2. Coagulation of cellular protein

3. Conversion of hemoglobin to haematin

1. Parikh 6th Ed., Pg. 8.30.

Q 285.  Features of antemortem burn are a/e

Page 123: Elsevier Jan

(a) Vesicle with hyperemic base

(b) Red line

(c) Vesicle with air

(d) Presence of pus

 

Answer : Unattempted

Explanation :

1. K.S. Narayan Reddy 20th Ed., Pg. 287.

Q 286.  Female with RHD is on prosthetic valve replacement. The ideal time for heparin

(a) 32 wk

(b) 36 wk

(c) 40 wk

(d) Onset of labor

Answer : Unattempted

Explanation :

a. Unfractionated heparin is used between 6–12 weeks and again after 36 weeks and stopped just

before delivery.

b. LMW heparin should not be used as it results in valvular thrombosis.

c. Warfarin cannot be used in early pregnancy as it leads to still births. Abortins and warfarin

embryopathy,

d. And if continued after 36 weeks it causes excessive bleeding as it decreases production of clotting

factors.

1. Williams 22nd Ed., Pg. 1023.

Q 287.  23 year lady with previous h/o caesarian section presents with hematuria during labor. What

could be the possibility?

(a) Prolonged labor

Page 124: Elsevier Jan

(b) Impending scar rupture

(c) Urethral trauma

(d) Membrane rupture

 

Answer : Unattempted

Explanation :

Features of CS scar rupture are FHR decelerations- most significant and earliest to come.

Tachycardia, fall in BP, fresh vaginal bleeding, superficially palpated fetal parts and absent FHS, loss

of uterine contour and loss of station of head on p/v examination. Hematuria occurs when the scar

ruptures into the bladder.

1. Williams 22nd Ed., Pg. 613.

Q 288.  Which of the following is associated with >20% risk of congenital anomaly

(a) Omphalocele

(b) Gastroscisis

(c) Spina Bifida

(d) Cleft Palate

Answer : Unattempted

Explanation :

Omphalocele

a. Midline abdominal wall defect

b. Viscera may include liver small and large intestine, spleen gall bladder covered by amnion

c. Due to failure of bowel to return to the body cavity from its physiological herniation during 6th -10th

week

d. Associated anomalies occur in 30–70% cases

e. In descending order of frequency associated with

1. Chromosomal Abnormality (Trisomy 13, 18, 21))

2. Congenital Heart Disease (Tetralogy of Fallot, ASD)

3. Beckwith Wiedemann Syndrome (Large for Gestational Age Baby, Hyperinsulinism, Visceromegally

of Kidney, Adrenal Gland and Pancreas, Macroglossia, Hepatorenal Syndrome)

4. Pentalogy of Cantrel and Prune Belly Syndrome

1. Langmen 10th Ed., Pg. 221, 294, 278.2. CSDT 12th Ed., Pg. 1318.

Q 289.  67 year male smoker presents with hemoptysis and cough, broncoscopic biopsy shows

Undifferentiated carcinoma on immuno-histochemistry will show

Page 125: Elsevier Jan

(a) Cytokeratin

(b) Viamentin

(c) Calretinin

(d) Leucocyte common antigen

 

Answer : Unattempted

Explanation :

Repeated several times in previous years

Q 290.  Dyslipidemia is diagnosed by

(a) Cholesterol

(b) LDL/HDL

(c) Cholesterol/tg

(d) APO A1

Answer : Unattempted

Explanation :

LDL: HDL cholesterol ratio is the best predictor of dyslipidemia in atherosclerosis and coronary heart

disease.

1. Harper 28th Ed., Pg. 230.

Q 291.  Spindle cell tumors in GIT can be diagnosed having marker

(a) CD 117

(b) CD 34

(c) Not recalled

(d) Not recalled

 Answer : Unattempted

Explanation :

a. Leiomyosarcomas and gists make up 1—3% of gastric neoplasms

b. They most frequently involve the anterior and posterior walls of the gastric fundus and often ulcerate

and bleed.

c. Even those lesions that appear benign on histologic examination may behave in a malignant fashion.

These tumors rarely invade adjacent viscera and characteristically do not metastasize to lymph

nodes, but they may spread to the liver and lungs.

MORPHOLOGY

a. May be solitary or multiple

Page 126: Elsevier Jan

b. Cut surface of the tumor is tan and usually lacks whirling smooth muscle pattern of leiomyoma or

leiomyosarcoma. microscopically the tumor can exhibit spindle cells

c. Most tumors are quite cellular

d. Mitotic activity is variable

e. Majority of the tumor shows positivity for C-KIT(CD117)

85% of GIST are C-KIT+35% have normal C-KIT but positive for PDGFRA

TREATMENT

a. The treatment of choice is surgical resection.

b. Combination chemotherapy should be reserved for patients with metastatic disease.

c. Gists are unresponsive to conventional chemotherapy; ~50% of patients experience objective

response and prolonged survival when treated with imatinib mesylate, a selective inhibitor of the c-

kit tyrosine kinase.

d. Many patients with GIST whose tumors have become refractory to imatinib subsequently benefit from

sunitinib, another inhibitor of the c-kit tyrosine kinase.

1. Harrison's 17th Ed., Pg. 573.2. Robbins Pathology, 7th Ed., Pg. 826.

Q 292.  Sudden thuderclap headache with unconciousness with ipsilateral dilated pupil is due to

(a) Acute ischemia of mid brain

(b) Acute aneurismal hemorrhage

(c) Meningitis

(d) Tubercular lesion

Answer : Unattempted

Explanation :

a. Most unruptured intracranial aneurysms are completely asymptomatic. Symptoms are usually due to

rupture and resultant SAH, although some present with mass effect on cranial nerves or brain

parenchyma.

b. At the moment of aneurysmal rupture with major SAH, the ICP suddenly rises. This may account for

the sudden transient loss of consciousness that occurs in nearly half of patients

c. Sudden loss of consciousness may be preceded by a brief moment of excruciating headache, but

most patients first complain of headache upon regaining consciousness. In 10% of cases, aneurysmal

bleeding is severe enough to cause loss of consciousness for several days.

d. The patient often calls the headache “the worst headache of my life”;however, the most important

characteristic is sudden onset.

e. Sudden headache in the absence of focal neurologic symptoms is the hallmark of aneurysmal

rupture, focal neurologic deficits may occur.

Anterior communicating artery or MCA bifurcation aneurysms may rupture into the adjacent brain or

subdural space and form a hematoma large enough to produce mass effect.

Page 127: Elsevier Jan

g. The common deficits that result include hemiparesis, aphasia, and abulia.

h. A third cranial nerve palsy, particularly when associated with pupillary dilatation, loss of ipsilateral

(but retained contralateral) light reflex, and focal pain above or behind the eye, may occur with an

expanding aneurysm at the junction of the posterior communicating artery and the internal carotid

artery

1. Harrison's 17th Ed.

Q 293.  ABO and RH grouping is not required in

(a) Fresh frozen plasma

(b) Cryoprecipitate

(c) PRP

(d) Single donor platelet

 

Answer : Unattempted

Explanation :

1. CPDT 19th Ed., Pg. 848.

Q 294.  Immunofluroscence method to detect ANCA in fresh tissue of rat

(a) Liver

(b) Kidney

(c) Stomach

(d) Brain

Answer : Unattempted

Explanation :

http://www3.interscience.wiley.com/journal/119263339/abstract?cretry=1&sretry=0

Q 295.  Type of anemia caused by ileocaecal TBb

(a) Megaloblastic anemia

(b) Normocytic normochromuc

(c) Sideroblastic

(d) Iron deficiency anemia

 Answer : Unattempted

Explanation :

a. Intestinal tuberculosis:

b. Intestinal tuberculosis is usually seen in underdeveloped countries.

Page 128: Elsevier Jan

c. It occurs mostly in patients with immunocompromised states and impaired host defenses and

malnutrition.

d. Organisms involved are Mycobacterium tuberculosis and Mycobacterium Bovis.

e. It can affect any part of GIT from mouth to anus. The sites affected most are ileum, ileocecal region

and

Proximal colon.

g. Primary focus is in intestine and mesenteric lymph nodal involvement causes Ghon complex

formation.

h. Tuberculous granulomas are initially formed in the mucosa or the peyer's patches. These granulomas

are seen

Just beneath the ulcer bed mainly in the submucosa. Tuberculous ulcers are superficial and usually

transversely

Oriented. Cicatrical healing of these ulcers may produce strictures. More over occlusive arterial

changes

k. Associated with tuberculosis may produce ischemia and perforation of ulcers as well as aid in

development of

Strictures.

m. It is of two types depending upon route of infection, and immune response

1. Primary intestinal T.B

2. Secondary intestinal T.B

n. It can also be classified into types depending upon type of tuberculous lesion.

1. Ulcerative tuberculosis.

2. Hyperplastic tuberculosis.

Primary tuberculosis:

a. Primary T.B. is the form of disease that develops in a previously unexposed and therefore

unsensitized person.

b. The source of infection is exogenous i.e. Taking milk infected with mycobacterium Bovis. In case of

elderly or

c. Immunocompromized patients who had been infected previously can also show the primary pattern of

response

d. When re-infected because their immunity is not able to store information for the next exposure.

e. The primary response in subjects with intact immune system is conversion to latent T.B. and in

Immunocompromized subjects it is of primary progressive type. The latent form then can be

reactivated later in life

When the immunity is weakened.

Secondary tuberculosis:

a. Secondary or post-primary/reactivation tuberculosis is the pattern of disease that arises in a

previously sensitized

b. Host. It may follow shortly after primary tuberculosis, but more commonly it arises from reactivation of

Page 129: Elsevier Jan

dormant

c. Primary lesion many decades after initial infection. The source of infection may be blood born,

swallowing coughed

d. Up infected material, direct spread from adjacent organ or through lymph channels of infected lymph

nodes.

e. In 25–50% of cases there is accompanying active pulmonary tuberculosis. 10.14.

Ulcerative tuberculosis:

a. It is mostly secondary to pulmonary tuberculosis and arises as a result of swallowing tubercle bacilli.

There are

b. Multiple ulcers in the terminal ileum, lying transversely, and the overlying serosa is thickened,

reddened, and

c. Covered in tubercles

d. Tubercle is a granuloma surrounded by fibrous tissue that has undergone central caseation necrosis.

Tubercles

e. Heal by fibrosis and calcifications

Hyperplastic tuberculosis:

a. This variety usually occurs in the ileocecal region, although solitary or multiple lesions in the distal

ileum are

b. Sometimes seen. The infection establishes itself in lymphoid follicles, and the resulting chronic

inflammation causes

c. Thickening of intestinal wall and narrowing of lumen. There is early involvement of regional lymph

nodes which may

d. Caseate. Patients of this variety presented with sub-acute intestinal obstruction, mass in right iliac

fossa, perforation

e. And some times complete intestinal obstruction. 10.14.

Some times ileocecal tuberculosis presents as acute abdomen without any lump in right lower

abdomen

g. Days of fever and pain abdomen. Patient explored with a suspicion of enteric perforation and found a

lot of puss in

h. Peritoneal cavity and 2 perforations in the terminal ileum with enlarged local mesenteric lymph nodes.

There was no

Stricture distal to perforations and no tubercles anywhere in the abdomen. Biopsy of mesenteric

lymph node

Revealed tuberculous lesions. Patient gave a good response to anti-tuberculosis therapy.

As vitamine B12 absorption occurs primarily in ileum region and ileocaecal tuberculosis presents with

obstructive symptoms it will cause vitamin B12malabsorption.

1. CSDT 12th Ed., Pg. 674.

Page 130: Elsevier Jan

Q 296.  17 year old girl with primary amenorrhea has features of absent breast on examination found to

have hypoplastic uterus the cause could be:

(a) Turner syndrome

(b) MRK syndrome

(c) Mixed gonadal dysgenesis

(d) Androgen insensitivity syndrome

Answer : Unattempted

Explanation :

Turner syndrome

a. 45, X or 45, X/46, XX

b. Streak gonad or immature ovary

c. Genitalia: external: Female, Internal: Hypoplastic female

d. Breast Development

e. Infancy: lymphedema, web neck, shield chest, low set hair line, cardiac defects and coarctation of

the aorta, urinary tract malformations & horseshoe kidney

Childhood: short stature, cubitus valgus, short neck, short 4th metacarpals, hypoplastic nails,

micrognathia, scoliosis, otitis media & sensori-neural hearing loss, ptosis & amblyopia, multiple nevi &

keloid formation, autoimmune thyroid disease, visuo-spatial learning difficulties

g. Adulthood: pubertal failure & primary amenorrhea, hypertension, obesity, dyslipidemia, impaired

glucose tolerance & insulin resistance, autoimmune thyroid disease, cardiovascular disease, aortic

root dilatation, osteoporosis, inflammatory bowel disease, chronic hepatic dysfunction, increased risk

of colon cancer, hearing loss

Mixed Gonadal Dysgenesis (45, X/46, XY)

a. Mixed gonadal dysgenesis typically results from 45, X/46, XY mosaicism.

b. The phenotype of patients with this condition varies considerably.

c. Although some patients have a predominantly female phenotype with somatic features of TS, streak

gonads, and müllerian structures, most 45, X/46, XY individuals have a male phenotype and testes,

and the diagnosis is made incidentally after amniocentesis or during investigation of infertility.

Androgen Insensitivity Syndrome

a. Mutations in the androgen receptor (AR) cause resistance to androgen (testosterone, DHT) action or

the androgen insensitivity syndrome (AIS).

b. Androgen receptor is X-linked, only males are affected and maternal carriers are phenotypically

normal. XY individuals with complete AIS (formerly called testicular feminization syndrome) have a

female phenotype, normal breast development (due to aromatization of testosterone), a short vagina

but no uterus (because MIS production is normal), scanty pubic and axillary hair, and female

Page 131: Elsevier Jan

psychosexual orientation. Gonadotropins and testosterone levels can be low, normal, or elevated,

depending on the degree of androgen resistance and the contribution of estradiol to feedback

inhibition of the hypothalamic-pituitary gonadal axis.

c. Most patients present with inguinal herniae (containing testes) in childhood or with primary

amenorrhea in adulthood.

d. Gonadectomy is usually performed, as there is a low risk of malignancy, and estrogen replacement is

prescribed. Alternatively, the gonads can be left in situ until breast development is complete.

e. The use of graded dilators in adolescence is usually sufficient to dilate the vagina and permit sexual

intercourse.

Partial AIS (Reifenstein syndrome) results from less severe AR mutations. Patients often present in

infancy with perineoscrotal hypospadias, small undescended testes, and with gynecomastia at the

time of puberty.

g. More severely under and orgenized patients present with clitoral enlargement and labial fusion and

may be raised as females.

h. Azoospermia and male-factor infertility has also been described in association with mild loss-of-

function mutations in the androgen receptor. Trinucleotide (CAG) repeat expansion, from a mean of

22 repeats to >40 repeats, within a highly polymorphic region of the androgen receptor is associated

with spinal and bulbar muscular atrophy (also known as Kennedy disease).

These patients may show evidence of partial androgen insensitivity in adolescence or adulthood (e.g.,

gynecomastia).

Congenital Absence of the Uterus (Uterine Agenesis, Uterovaginal Agenesis, Rokitansky–

Küster–hauser syndrome).

a. This disorder is the second most frequent cause of primary amenorrhea.

b. Accounts for about 15% of individuals with primary amenorrhea

c. Individuals with complete uterine agenesis have normal ovaries with regular cyclic ovulation and

normal endocrine function.

d. Women with this disorder have normal breast and pubic and axillary hair development but have a

shortened or absent vagina in addition to absence of the uterus

e. Congenital renal abnormalities occur in about one third of these individuals and skeletal abnormalities

in about 12%.

Cardiac and other congenital abnormalities also occur with increased frequency.

g. Occasional defects in the bones of the middle ear can also occur, resulting in some degree of

deafness.

1. Kliegman: Nelson Textbook of Pediatrics, 18th Ed.2. Harrison's 17th Ed., Pg. 2342.

Q 297.  A hypertensive female shows ascending aorta aneurism. On histopathological examination.

Shows giant cell granuloma in vessel wall .which one can be the cause

(a) Tubercular aortoarteritis

Page 132: Elsevier Jan

(b) Wegner granulomatosis

(c) Pan

(d) Non specific aortoarteritis

 

Answer : Unattempted

Explanation :

a. The vascular lesion in PAN is a necrotizing inflammation of small and medium-sized muscular

arteries. The lesions are segmental and tend to involve bifurcations and branchings of arteries.

b. The histopathologic hallmarks of Wegener's granulomatosis are necrotizing vasculitis of small

arteries and veins together with granuloma formation, which may be either intravascular or

extravascular.

c. Takayasu's arteritis is an inflammatory and stenotic disease of medium- and large-sized arteries

characterized by a strong predilection for the aortic arch and its branches. For this reason, it is often

referred to as the aortic arch syndrome.

d. The disease involves medium- and large-sized arteries, with a strong predilection for the aortic arch

and its branches; the pulmonary artery may also be involved.

1. Harrison's 17th Ed., Pg. 2127.

Q 298.  57 year old man presents with cough with productive sputum with pus culture of the sputum

shows gram positive cocci with surrounding hemolysis. Which of the following test can be

done to differentiate this organism from surrounding commensals?

(a) Bile solubility test

(b) Bacitracin test

(c) Coagulase

(d) None

Answer : Unattempted

Explanation :

a. The organism shows gram positivity that can be possible in

b. Enterococci/pneumococci/streptococcus pyogenes but hemolysis can be found in streptococcus

pyogenes

c. Other two will show alpha or no hemolysis in culture

d. So the differentiating feature to isolate this organism is bacitracin test

1. Ananth 8th Ed., Pg. 209.

Q 299.  Ayoung patient presents with headache with bitemporal hemianopia with 6/6 vision this

condition is seen with

(a) Trauma

Page 133: Elsevier Jan

(b) Chiasmal lesion

(c) B/1 cavernous sinus thrombosis

(d) Optic neuritis

 

Answer : Unattempted

Explanation :

Optic neuritis

a. Produce sudden onset progressive and profound visual loss

b. Dark adaptation lowered

c. Visual obscuration in the bright light

d. Impairement of color vision

e. Movement phophenesand sond phosphenes: phosphenes are glowing sensation produced by non

photic stimuli

Episodic transient obscuration of vision (uhthoff symptoms)

g. Depth perception impaired (pulfrich phenomena)

h. Pain

Chaismal lesion:

Central chiasmal lesion

a. Bitemporal hemianopia

b. Bitemporal hemianopic palsy of pupillary reflex

Lateral chiasmal lesion

a. Binasal hemianopia

b. Binasal hemianopic palsy of pupillary reflex Psychae

1. Parson 20th Ed., Pg. 338.2. A.K. Khurana 294

Q 300.  Young girl with repeated bouts of pain in leg, nausea, vomiting and physical examination was

normal. Most likely she is suffering from:

(a) Generalised anxiety disorder

(b) Conversion disorder

(c) Somatoform pain disorder

(d) Somatisation disorder

Answer : Unattempted

Explanation :

Page 134: Elsevier Jan

1. Kaplan and Sadock 634